Is it true that $0.999999999dots=1$?












260












$begingroup$


I'm told by smart people that
$$0.999999999dots=1$$
and I believe them, but is there a proof that explains why this is?










share|cite|improve this question











$endgroup$












  • $begingroup$
    Comments are not for extended discussion; this conversation has been moved to chat.
    $endgroup$
    – quid
    Jan 6 at 13:43
















260












$begingroup$


I'm told by smart people that
$$0.999999999dots=1$$
and I believe them, but is there a proof that explains why this is?










share|cite|improve this question











$endgroup$












  • $begingroup$
    Comments are not for extended discussion; this conversation has been moved to chat.
    $endgroup$
    – quid
    Jan 6 at 13:43














260












260








260


114



$begingroup$


I'm told by smart people that
$$0.999999999dots=1$$
and I believe them, but is there a proof that explains why this is?










share|cite|improve this question











$endgroup$




I'm told by smart people that
$$0.999999999dots=1$$
and I believe them, but is there a proof that explains why this is?







real-analysis algebra-precalculus decimal-expansion






share|cite|improve this question















share|cite|improve this question













share|cite|improve this question




share|cite|improve this question








edited May 30 '18 at 21:40


























community wiki





26 revs, 17 users 36%
Michael Hardy













  • $begingroup$
    Comments are not for extended discussion; this conversation has been moved to chat.
    $endgroup$
    – quid
    Jan 6 at 13:43


















  • $begingroup$
    Comments are not for extended discussion; this conversation has been moved to chat.
    $endgroup$
    – quid
    Jan 6 at 13:43
















$begingroup$
Comments are not for extended discussion; this conversation has been moved to chat.
$endgroup$
– quid
Jan 6 at 13:43




$begingroup$
Comments are not for extended discussion; this conversation has been moved to chat.
$endgroup$
– quid
Jan 6 at 13:43










26 Answers
26






active

oldest

votes


















347












$begingroup$

What does it mean when you refer to $.99999ldots$? Symbols don't mean anything in particular until you've defined what you mean by them.



In this case the definition is that you are taking the limit of $.9$, $.99$, $.999$, $.9999$, etc. What does it mean to say that limit is $1$? Well, it means that no matter how small a number $x$ you pick, I can show you a point in that sequence such that all further numbers in the sequence are within distance $x$ of $1$. But certainly whatever number you choose your number is bigger than $10^{-k}$ for some $k$. So I can just pick my point to be the $k$th spot in the sequence.



A more intuitive way of explaining the above argument is that the reason $.99999ldots = 1$ is that their difference is zero. So let's subtract $1.0000ldots -.99999ldots = .00000ldots = 0$. That is,



$1.0 -.9 = .1$



$1.00-.99 = .01$



$1.000-.999=.001$,



$ldots$



$1.000ldots -.99999ldots = .000ldots = 0$






share|cite|improve this answer











$endgroup$













  • $begingroup$
    Comments are not for extended discussion; this conversation has been moved to chat.
    $endgroup$
    – quid
    Jan 6 at 13:41



















255












$begingroup$

Suppose this was not the case, i.e. $0.9999... neq 1$. Then $0.9999... < 1$ (I hope we agree on that). But between two distinct real numbers, there's always another one (say $x$) in between, hence $0.9999... < x < 1$.



The decimal representation of $x$ must have a digit somewhere that is not $9$ (otherwise $x = 0.9999...$). But that means it's actually smaller, $x < 0.9999...$, contradicting the definition of $x$.



Thus, the assumption that there's a number between $0.9999...$ and $1$ is false, hence they're equal.






share|cite|improve this answer











$endgroup$













  • $begingroup$
    Comments are not for extended discussion; this conversation has been moved to chat.
    $endgroup$
    – quid
    Jan 6 at 13:43



















164












$begingroup$

What I really don't like about all the above answers, is the underlying assumption that $1/3=0.3333ldots$ How do you know that? It seems to me like assuming the something which is already known.



A proof I really like is:



$$begin{align}
0.9999ldots × 10 &= 9.9999ldots\
0.9999ldots × (9+1) &= 9.9999ldots\
text{by distribution rule: }Space{15ex}{0ex}{0ex} \
0.9999ldots × 9 + 0.9999ldots × 1 &= 9.9999ldots\
0.9999ldots × 9 &= 9.9999dots-0.9999ldots\
0.9999ldots × 9 &= 9\
0.9999ldots &= 1
end{align}$$



The only things I need to assume is, that $9.999ldots - 0.999ldots = 9$ and that $0.999ldots × 10 = 9.999ldots$ These seems to me intuitive enough to take for granted.



The proof is from an old high school level math book of the Open University in Israel.






share|cite|improve this answer











$endgroup$













  • $begingroup$
    Comments are not for extended discussion; this conversation has been moved to chat.
    $endgroup$
    – quid
    Jan 6 at 13:44



















151












$begingroup$

Assuming:




  1. infinite decimals are series where the terms are the digits divided by the proper power of the base

  2. the infinite geometric series $a + a cdot r + a cdot r^2 + a cdot r^3 + cdots$ has sum $dfrac{a}{1 - r}$ as long as $|r|<1$




$$0.99999ldots = frac{9}{10} + frac{9}{10^2} + frac{9}{10^3} + cdots$$



This is the infinite geometric series with first term $a = frac{9}{10}$ and common ratio $r = frac{1}{10}$, so it has sum $$frac{frac{9}{10}}{1 - frac{1}{10}} = frac{frac{9}{10}}{frac{9}{10}} = 1.$$






share|cite|improve this answer











$endgroup$









  • 9




    $begingroup$
    Your method is a simple way of converting the decimal representation of a rational number into a fraction, e.g. $0.150150150...=sum_{ngeq 1}frac{150}{10^{3n}}=frac{0.150}{1-10^{-3}}=frac{50}{333}$
    $endgroup$
    – Américo Tavares
    Aug 16 '10 at 22:02












  • $begingroup$
    The question is where are you getting this common ratio of 1/10 and what law permits you to use it in this case?
    $endgroup$
    – Serj Sagan
    Aug 2 '16 at 19:11










  • $begingroup$
    @SerjSagan Generally, when not otherwise indicated, numbers are written in base 10. That makes the ratio of the values of successive "places" 10 or $frac{1}{10}$, depending on direction. This is the "... proper power of the base" piece of my point 1.
    $endgroup$
    – Isaac
    Aug 2 '16 at 19:17





















71












$begingroup$

$$x=0.999...$$
$$10x=9.999...$$
$$10x-x=9.999...-0.999...$$
$$9x=9$$
$$x=1$$



thus, $0.999...=1$






share|cite|improve this answer











$endgroup$









  • 1




    $begingroup$
    This is the most intuitive argument, although some might say "But 10x-x isn't 9, because there's going to be a mismatch all the way to the right" - Noah's more complex deals with that.
    $endgroup$
    – Charles Stewart
    Jul 21 '10 at 10:55










  • $begingroup$
    10x - x != 9. 10x - x would be 8.9999...1. However infinite the extent of 9s is in x, if we multiply it by 10, the nines are shifted left by one position and a zero inserted at the "last" place, and then when you subtract the other number there is a nine subtracted from a zero at the far right. Otherwise we'd have to give 0.999.. some unusual properties like automatically increasing the number of nines when it is multiplied. It would not be just an ordinary number. Maybe that's the problem. 0.999... might just not be an ordinary type number as some people are using it.
    $endgroup$
    – Doug Treadwell
    Jul 23 '10 at 2:19






  • 8




    $begingroup$
    @Doug It's incorrect to talk about the "number of nines" because infinity minus a number = infinity. Infinity is transcendent. It means "uncountable". If you take infinity and slide it left a little bit, it's still infinity long.
    $endgroup$
    – ErikE
    Sep 25 '10 at 7:00










  • $begingroup$
    The problem is the assumption that 9x = 9 when in fact 9x = 8.999... your proof is flawed.
    $endgroup$
    – Serj Sagan
    Aug 2 '16 at 19:16






  • 9




    $begingroup$
    @SerjSagan that line isn't an assumption, it's a deduction. If you believe that $9.999dots - 0.999dots = 9$, then you must therefore believe that $9x = 9$.
    $endgroup$
    – Omnomnomnom
    Dec 15 '16 at 17:03



















52












$begingroup$

Okay I burned a lot of reputation points (at least for me) on MathOverflow to gain clarity on how to give some intuition into this problem, so hopefully this answer will be at least be somewhat illuminating.



To gain a deeper understanding of what is going on, first we need to answer the question, "What is a number?"



There are a lot of ways to define numbers, but in general numbers are thought of as symbols that represent sets.



This is easy for things like the natural numbers. So 10 would correspond to the set with ten things -- like a bag of ten stones. Pretty straight forward.



The tricky part is that when we consider ten a subset of the real numbers, we actually redefine it. This is not emphasized even in higher mathematics classes, like real analysis; it just happens when we define the real numbers.



So what is 10 when constructed in the real numbers? Well, at least with the Dedekind cut version of the real numbers, all real numbers correspond to a set with an infinite amount of elements. This makes 10 under the hood look drastically different, although in practice it operates exactly the same.



So let's return to the question: Why is 10 the same as 9.99999? Because the real numbers have this completely surprising quality, where there is no next real number. So when you have two real numbers that are as close together as possible, they are the same. I can't think of any physical object that has this quality, but it's how the real numbers work (makes "real" seem ironic).



With integers (bag of stones version) this is not the same. When you have two integers as close to each other as possible they are still different, and they are distance one apart.



Put another way, 10 bag of stones are not the same as 9.9999999 but 10 the natural number, where natural numbers are a subset of the real numbers is.



The bottom line is that the real numbers have these tricky edge cases that are hard to understand intuitively. Don't worry, your intuition is not really failing you. :)



I didn't feel confident answering until I got this Terence Tao link:



(Wayback Machine) https://web.archive.org/web/20100725014132/http://www.google.com:80/buzz/114134834346472219368/RarPutThCJv/In-the-foundations-of-mathematics-the-standard



(PDF, page 12) https://terrytao.files.wordpress.com/2011/06/blog-book.pdf






share|cite|improve this answer











$endgroup$









  • 4




    $begingroup$
    The link doesn't work.
    $endgroup$
    – user 170039
    Jan 5 '16 at 15:08










  • $begingroup$
    I'm not sure that most people think of real numbers as representing sets. Indeed, probably few of those who didn't study mathematics or attend mathematics lectures have even heard of the Dedekind construction. The common visualization of real numbers is as points on a line.
    $endgroup$
    – celtschk
    Aug 14 '16 at 9:03






  • 2




    $begingroup$
    the link is dead
    $endgroup$
    – baxx
    Sep 10 '16 at 20:43










  • $begingroup$
    As a smart person, I actually like this answer better than balpha and Elezar Laibovich's answer. It's good enough for me because I can figure out from it a fuller answer that explains how people were looking for a definition of a real number in ZF such that the set of all of them is a complete ordered field so they found one and proved it's a complete ordered field. math.stackexchange.com/questions/2437893/… actually says what a real natural number is. Those answers aren't bad either because I can prove that all real numbers have a decimal representation.
    $endgroup$
    – Timothy
    May 17 '18 at 1:34



















44












$begingroup$

There are genuine conceptual difficulties implicit in this question. The transition from the rational numbers to the real numbers is a difficult one, and it took a long time and a lot of thought to make it truly rigorous. It has been pointed out in other answers that the notation $0.999999ldots$ is just a shorthand notation for the infinite geometric series $sumlimits_{n=1}^{infty} 9left( frac{1}{10} right)^{n},$ which has sum $1.$ This is factually correct, but still sweeps some of the conceptual questions under the carpet. There are questions to be addressed about what we mean when we write down (or pretend to) an infinite decimal, or an infinite series. Either of those devices is just a shorthand notation which mathematicians agree will represent some numbers, given a set of ground rules. Let me try to present an argument to suggest that if the notation $0.99999ldots$ is to meaningfully represent any real number, then that number could be nothing other than the real number $1$, if we can agree that some truths are "self-evident".



Surely we can agree that the real number it represents can't be strictly greater than $1$, if it does indeed represent a real number. Let's now convince ourselves that it can't be a real number strictly less than $1,$
if it makes any sense at all. Well, if it was a real number $r < 1,$ that real number would be greater than or equal to $sumlimits_{n=1}^{k} 9left( frac{1}{10} right)^{n}$ for any finite integer $k.$ This last number is the decimal $0.99 ldots 9 $ which terminates after $k$ occurrences of $9,$ and differs from $1$ by $frac{1}{10^{k}}.$ Since $0 < r <1,$ there is a value of $k$ such that $frac{1}{10^{k}} < 1-r,$ so $1 - frac{1}{10^{k}} >r.$ Hence $sumlimits_{n=1}^{k} 9left( frac{1}{10} right)^{n} > r.$ But this can't be, because we agreed that $r$ should be greater than or equal to each of those truncated sums.



Have I proved that the recurring decimal is equal to $1$? Not really- what I have proved is that if we allow that recurring decimal to meaningfully represent any real number, that real number has to be $1,$ since it can't be strictly less than $1$ and can't be strictly greater than $1$. At this point, it becomes a matter of convention to agree that the real number $1$ can be represented in that form, and that convention will be consistent with our usual operations with real numbers and ordering of the real numbers, and equating the expression with any other real number would not maintain that consistency.






share|cite|improve this answer











$endgroup$









  • 6




    $begingroup$
    This is the best answer here; it's a shame it was posted four years later so it's gotten so little attention.
    $endgroup$
    – Eric Wofsey
    Oct 4 '15 at 0:44






  • 1




    $begingroup$
    If I am not mistaken then this answer also answers the question of this post.
    $endgroup$
    – user 170039
    Jan 6 '16 at 14:32










  • $begingroup$
    By the way, I was thinking that maybe you could write an answer for the linked post as well (or else, can you just tell me how can I give a link to this answer in a comment below my post?).
    $endgroup$
    – user 170039
    Jan 6 '16 at 14:40






  • 1




    $begingroup$
    I'm not sure how to set up links on here ( maybe the "cite") button. I am basically agreeing with what you say in your post.
    $endgroup$
    – Geoff Robinson
    Jan 6 '16 at 15:03










  • $begingroup$
    I think that the "evident" issue of a number being an abstract concept existing in isolation from any positional representation should also be explicitly stated. One then proceeds to give rules as to how the positional representation works (as you did), and from these rules it follows that all real numbers have an infinitely long representation in any base, and rational numbers have a finitely long representation in some bases. The integers are a special case of rational numbers that have a finite representation in all bases.
    $endgroup$
    – Kuba Ober
    Jul 28 '16 at 19:55





















39












$begingroup$

One argument against this is that 0.99999999... is "somewhat" less than 1. How much exactly?



      1 - 0.999999... = ε              (0)


If the above is true, the following also must be true:



9 × (1 - 0.999999...) = ε × 9


Let's calculate:



0.999... ×
9 =
───────────
8.1
81
81
.
.
.

───────────
8.999...


Thus:



     9 - 8.999999... = 9ε              (1)


But:



         8.999999... = 8 + 0.99999...  (2)


Indeed:



8.00000000... +
0.99999999... =
────────────────
8.99999999...


Now let's see what we can deduce from (0), (1) and (2).



9 - 8.999999... = 9ε                      because of (2)
9 - 8.999999... = 9 - (8 + 0.99999...) = because of (1)
= 9 - 8 - (1 - ε) because of (0)
= 1 - 1 + ε
= ε.


Thus:



9ε = ε

8ε = 0

ε = 0

1 - 0.999999... = ε = 0


Quod erat demonstrandum. Pardon my unicode.






share|cite|improve this answer











$endgroup$













  • $begingroup$
    I did my best to avoid 0.00000..., but this made the calculations not as strikingly simple as I'd have liked to.
    $endgroup$
    – badp
    Jul 20 '10 at 21:10










  • $begingroup$
    Why was this voted down? It seems reasonable to this amateur math enjoyer.
    $endgroup$
    – ErikE
    Sep 25 '10 at 7:07










  • $begingroup$
    @Emtucifor I guess this sounds like "nonsense" to people that disagree on the basic premise of 0.999... = 1 :)
    $endgroup$
    – badp
    Sep 25 '10 at 7:21










  • $begingroup$
    8ε = 0 instead of 10ε = 0
    $endgroup$
    – user59671
    May 9 '13 at 9:37






  • 2




    $begingroup$
    @SerjSagan nope, the RHS of (1) follows from (0). The fact that $9ε = ε$ is precisely the point I'm making.
    $endgroup$
    – badp
    Aug 2 '16 at 20:00



















23












$begingroup$

.999... = 1 because .999... is a concise symbolic representation of "the limit of some variable as it approaches one." Therefore, .999... = 1 for the same reason the limit of x as x approaches 1 equals 1.






share|cite|improve this answer











$endgroup$





















    18












    $begingroup$

    If you take two real numbers x and y then there per definition of the real number z for which x < z < y or x > z > y is true.



    For x = 0.99999... and y = 1 you can't find a z and therefore 0.99999... = 1.






    share|cite|improve this answer











    $endgroup$





















      15












      $begingroup$

      You can visualise it by thinking about it in infinitesimals. The more $9's$ you have on the end of $0.999$, the closer you get to $1$. When you add an infinite number of $9's$ to the decimal expansion, you are infinitely close to $1$ (or an infinitesimal distance away).



      And this isn't a rigorous proof, just an aid to visualisation of the result.






      share|cite|improve this answer











      $endgroup$













      • $begingroup$
        I think this answer is not needed. It's similar to another answer but has wrong information. Infinitesimal numbers exist only in the hyperreal number system and in the hyperreal system, numbers infinitesimally close are not equal. The real numbers are a subset of the hyperreal numbers so the community decided theat the notation represents 1 and not a number infinitesimally close because then it wouldn't represent a real number. Maybe the real numbers can be defined as equivalence classes infinitesimally close hyperreal numbers and from that, redefine the notation to represent the number 1.
        $endgroup$
        – Timothy
        May 22 '18 at 19:19



















      13












      $begingroup$

      Indeed this is true. The underlying reason is that decimal numbers are not unique representations of the reals. (Technically, there does not exist a bijection between the set of all decimal numbers and the reals.)



      Here's a very simple proof:



      $$begin{align}
      frac13&=0.333ldots&hbox{(by long division)}\
      implies0.333ldotstimes3&=0.999ldots&hbox{(multiplying each digit by $3$)}
      end{align}$$



      Then we already know $0.333ldotstimes3=1$ therefore $0.999ldots=1$.






      share|cite|improve this answer











      $endgroup$









      • 22




        $begingroup$
        -1. This is not a proof at all! Why is 1/3=0.333...? Seriously folks, for the private beta, let's try to maintain a little correctiness.
        $endgroup$
        – Scott Morrison
        Jul 20 '10 at 20:03






      • 5




        $begingroup$
        @Scott: Sure it is. You can prove it easily by long division. This is about algorithms for mathematical methods really.
        $endgroup$
        – Noldorin
        Jul 20 '10 at 20:12






      • 6




        $begingroup$
        @Scott: Might help to stop whining and post what you think is the 'correct' answer then.
        $endgroup$
        – Noldorin
        Jul 20 '10 at 20:17






      • 11




        $begingroup$
        Just to nitpick, there is a bijection between the set of decimal expansions and reals because they are sets with the same cardinality. It's just that the natural map taking expansions to real numbers isn't injective.
        $endgroup$
        – Simon Nickerson
        Jul 20 '10 at 21:15






      • 3




        $begingroup$
        @Scott, I see it would not be obvious that 1/3=0.333..., but as by Noldorins comment regarding long division, what would be wrong with this as a proof, if the first line is annotated with 'by long division' ?
        $endgroup$
        – Sami
        Jul 21 '10 at 4:58



















      12












      $begingroup$

      Given (by long division):

      $frac{1}{3} = 0.bar{3}$



      Multiply by 3:

      $3times left( frac{1}{3} right) = left( 0.bar{3} right) times 3$



      Therefore:

      $frac{3}{3} = 0.bar{9}$



      QED.






      share|cite|improve this answer











      $endgroup$













      • $begingroup$
        I think the long division precisely involves the proof of a limit like the sum mentioned above...
        $endgroup$
        – C-Star-Puppy
        Aug 27 '14 at 21:07






      • 2




        $begingroup$
        @FreeziiS, for this particular question the fact of a rigorous proof happens to be less important than the fact of a convincing proof. Plenty of people trust the long division algorithm because they learned it in school; they haven't seen a rigorous proof for its workability because they don't care; it obviously works. It is these people (who don't care about or trust rigorous proofs) who argue the most about $0.999999...$
        $endgroup$
        – Wildcard
        Dec 20 '16 at 0:16



















      9












      $begingroup$

      Often times people who ask this question are not very convinced by a proof. Since they may not be particularly math inclined, they may feel that a proof is a sort of sleight-of-hand trick, and I find the following intuitive argument (read "don't down-vote me for lack of rigor, lack of rigor is the point") a bit more convincing:



      STEP 1) If $.99...neq1$, everyone agrees that it must be less than $1$. Let $alpha$ denote $.99...$, this mysterious number less than $1$.



      STEP 2) Using a number line, you can convince them that since $alpha<1$, there must be another number $beta$ such that $alpha<beta<1$.



      STEP 3) Since $alpha<beta$, one of the digits of $beta$ must be bigger than the corresponding digit of $alpha$.



      STEP 4) However it is usually intitively clear that you cannot make any digit of $.99...$ bigger without making the resulting number (ie $beta$) bigger than $1$.



      STEP 5) Thus no such $beta$ can exist, and thus $.99...$ cannot be less than $1$.






      share|cite|improve this answer











      $endgroup$





















        7












        $begingroup$

        The real number system is defined as an extension of the rationals with the property that any sequence with an upper bound has a LEAST upper bound. The expression " 0.9-repeated" is defined to be the least real-number upper bound of the sequence 0.9. 0.99, 0.999,..... , which is 1. The rationals (and the reals) can also be extended to an arithmetic system (an ordered field) in which there are positive values which are less than every positive rational. In such systems the expression ".9-repeated" has no meaning.






        share|cite|improve this answer











        $endgroup$





















          7












          $begingroup$

          The problem isn't proving that $0.9999... = 1$. There are many proofs and they all are easy.



          The problem is being convinced that every argument you are making actually is valid and makes sense, and not having a sinking feeling you aren't just falling for some parlor trick.



          $0.99...9;$ (with $n$ 9s) is $sum_{i= 1}^n frac 9 {10^i}$ so "obviously" $0.999....$ (with an infinite number of 9s) is $sum_{i = 1}^{infty} frac 9{10^i}$.



          The obvious objection is: does it even make sense to talk about adding an infinite number of terms? How can we talk about taking and adding an infinite number of terms?



          And it's a legitimate objection.



          So when we learn math in elementary school we are told: Every real number can be written as a decimal expansion (maybe infinite) and every possible decimal expansion is a real number. And this is true. But we are not told why and we are expected to take it on faith, and we usually do.



          IF we take this on faith then a proof is very easy:



          $0.9999.... = sum_{i = 1}^{infty} frac 9{10^i}$



          $10*(0.9999....) = 10*sum_{i = 1}^{infty} frac 9{10^i}= sum_{i = 1}^{infty} frac {90}{10^i}=$



          $sum_{i = 1}^{infty} frac 9{10^{i-1}} = 9/10^0 + sum_{i = 2}^{infty} frac 9{10^{i-1}}= 9 + sum_{i = 1}^{infty} frac 9{10^i}$ (Look at the indexes!)



          So...



          $10*(0.999...) - (0.9999...) = (10 - 1)*0.9999.... = 9*0.99999.... = $



          $9 + sum_{i = 1}^{infty} frac 9{10^i} - sum_{i = 1}^{infty} frac 9{10^i} = 9$.



          So...



          $0.9999.... = 9/9 = 1$.



          Easy! !!!!!!!IF!!!!!!! we take it on faith that: Every real number can be written as a decimal expansion (maybe infinite) and every possible decimal expansion is a real number.



          So why can we take that on faith? That's the issue: why is that true and what does it mean?



          So....



          We've got the Integers. We use them to count discrete measurements. We can use an integer to divide a unit 1 into $m$ sub-units to measure measurements of $1/m$. As the $m$ can be as large as we want the $1/m$ can be as precise as we want and the system of all possible $n/m; m ne 0$ can measure any possible quantity with arbitrary and infinite precision.



          We hope. We call these $n/m$ numbers the Rationals and everything is fine until we discover that we can't actually measure measurements such as the square root of two or pi.



          But the Rationals still have infinite precision. We can get within 1/10 away from pi. We can get within 1/100 away from pi. Within $1/10^n$ for any possible power of 10.



          At this point, we hope we can say "we can't measure it with any finite power of 10 but we can always go one more significant measure, so if we go through infinite powers of 10 we will measure it to precision" and we hope that explanation will be convincing.



          But it isn't really. We have these "missing numbers" and we can get infinitely close the them, but what are they really?



          Well, we decide to become math majors and in our senior year of college we take a Real Analysis course and we find out.



          We can view numbers as sets of rational numbers. We can split the rational numbers at any point into two sets. We can split the rational numbers so that all the rational numbers less than 1/2 are in set A and all the rational numbers greater than or equal to 1/2 are in set B (which we ignore; we are only interested in set A.)



          These "cuts" can occur at any point but they must follow the following rules:



          --the set A of all the smaller rational numbers is not empty. Nor does it contain every rational number. Some rational number is not in it.



          --if any rational number (call it q) is in A, then every rational number smaller than q is also in A. (This means that if r is a rational not in A, then every rational bigger than r is also not in A.)



          -- A does not have a single largest element. (So it can be all the elements less than 1/2 but it can't be all the elements less than or equal to 1/2).



          And we let $overline R$ be the collection of all possible ways to "cut" the rational numbers in half that way.



          Notice sometimes the cut will occur at a rational number (all the rationals less than 1/2), but sometimes it will occur at points "between" the rational numbers. (All the rationals whose squares are less than 2). So the collection $overline R$ is a larger set than the set of Rational numbers.



          It turns out we can define the Real numbers as the points of $overline R$ where we can cut the rationals in two.



          We need to do a bit or work to show that this is actually a number system. We say $x, y in overline R; x < y$ if the "Set A made by cutting at x" $subset$ "Set A made by cutting at y". And we say $x + y = $ the point where we need to cut so that the set A created contains all the sums of the two other sets created by cutting at x and y. And we have to prove math works on $overline R$. But we can do it. And we do.



          But as a consequence we see that every real number is the least upper bound limit of a sequence of rational numbers. That's pretty much the definition of what a "cut point" is; the point that separate all the rationals less than it from all the other rationals.



          I like to say (somewhat trivially) that: the real number $x$ is the least upper bound of all the rational numbers that are smaller than $x$. And it's true!



          In the real numbers, every real number is the limit of some sequence of rational numbers. And every bounded sequence of rational numbers will have a real number least upper bound limit.



          ...



          Let that sink in for a minute.



          =====



          Okay, so given a sequence {3, 3.1, 3.14, 3.141,....} = {finite decimals that are less than pi} is a bounded sequence of rational numbers so $pi = $ the limit of the sequence which is also the limit of the infinite sequence 3.1415926....



          It now makes sense to talk of $0.9999.... = sum_{i=1}^{infty}9/10^i = lim{sum_{i=1}^n9/10^i}$ = a precise and real number.



          And from there we can say with confidence that that number is $1$. (By any of these proofs.)






          share|cite|improve this answer











          $endgroup$





















            6












            $begingroup$

            Another approach is the following:



            $$begin{align}
            0.overline9 &=lim_{n to infty} 0.underbrace{99dots 9}_{ntext{ times}} \
            &= lim_{n to infty} sumlimits_{k=1}^n frac{9}{10^k} \
            &=lim_{n to infty} 1-frac{1}{10^n} \
            &=1-lim_{n to infty} frac{1}{10^n} \
            &=1.
            end{align}$$






            share|cite|improve this answer











            $endgroup$









            • 7




              $begingroup$
              This approach appears in Isaac's answer from 4 years earlier.
              $endgroup$
              – Jonas Meyer
              Aug 26 '14 at 2:15



















            5












            $begingroup$

            There are some situations in which something like $0.99999ldots < 1$ indeed holds. Here is one coming from social choice theory.



            Let $w_1>w_2>ldots$ be an infinite sequence of positive numbers, and let $T$ be a number in the range $(0,sum_i w_i)$. Pick an index $i$. Choose a random permutation $pi$ of the positive integers, and consider the running totals
            $$
            w_{pi(1)}, w_{pi(1)} + w_{pi(2)}, w_{pi(1)} + w_{pi(2)} + w_{pi(3)}, cdots
            $$
            The Shapley value $varphi_i(T)$ is the probability that the first time that the running total exceeds $T$ is when $w_i$ is added.



            We will particularly be interested in the case in which the sequence $w_i$ is super-increasing: for each $i$, $w_i geq sum_{j=i+1}^infty w_j$. The simplest case is $w_i = 2^{-i}$. Every number $T in (0,1)$ can be written in the form
            $$ T = 2^{-a_0} + 2^{-a_1} + cdots, qquad a_0 < a_1 < cdots. $$
            In this case we can give an explicit formula for $varphi_i(T)$:
            $$
            varphi_i(T) = begin{cases}
            sum_{tcolon a_t>i} frac{1}{a_t binom{a_t-1}{t}} & text{if } i notin {a_0,a_1,ldots}, \
            frac{1}{a_s binom{a_s-1}{s}} - sum_{tcolon a_t>i} frac{1}{a_t binom{a_t-1}{t-1}} & text{if } i = a_s.
            end{cases}
            $$



            The first two functions are plotted here:
            enter image description here



            What happens for different sets of weights? The same formula applies, for
            $$
            T = w_{a_0} + w_{a_1} + cdots, qquad a_0 < a_1 < cdots.
            $$
            In general not all $T$ will be of this form; for $T$ not of this form, we take the lowest upper bound which is of this form. What we get for $w_i = 3^{-i}$ is:
            enter image description here



            Notice all the horizontal parts, for example the blue line at $y=1$ at $x in (1/6,1/3)$. Where does this stem from? Note that $1/3 = 3^{-1} = w_1$, whereas $1/6 = sum_{i=2}^infty 3^{-i} = sum_{i=2}^infty w_i$. If we substitute $w_i = 2^{-i}$, then $1/3$ corresponds to $0.1$ (in binary), whereas $1/6$ corresponds to $0.011111ldots$. So in this case there is a (visible) gap between $0.011111ldots$ and $0.1$!



            For more, take a look at this question and this manuscript.






            share|cite|improve this answer











            $endgroup$





















              4












              $begingroup$

              One cool way I learned to prove this is that, assuming by $0.99999...$ you mean $0.bar{9}$. Well we can say that
              $$0.bar{9}=sum_{n=1}^{infty}9cdot 10^{-n}=9sum_{n=1}^{infty}frac{1}{10^n}$$
              Which we know converges by fact that this is a geometric series with the ratio between terms being less than $1$. So we know that
              $$9sum_{n=1}^{infty}frac{1}{10^n}=9left(frac{1}{1-frac{1}{10}}-1right)=10-9=1$$
              Note that we subtract off the $1$ in the parentheses because we started indexing at $1$ rather than at $0$, so we have to subtract of the value of the sequence at $n=0$ which is $1$.






              share|cite|improve this answer











              $endgroup$









              • 2




                $begingroup$
                "Which we know converges by fact that this is a geometric series with the ratio between terms being less than 1." I don't think anyone who does know this would be questioning whether .9999.... = 1. The only people who are confused on the concept most assuredly do not know this.
                $endgroup$
                – fleablood
                Jan 15 '16 at 20:03










              • $begingroup$
                Just because you have studied series doesn't mean you have seen this proof. Hence why I have added it as a wiki answer on such a highly up voted answer, which people visiting it do not necessary have the same mathematical knowledge as the OP.
                $endgroup$
                – Will Fisher
                Jan 15 '16 at 21:55



















              3












              $begingroup$

              Here's my favorite reason why $.999ldots$ should equal $1$:$^{*}$
              begin{align*}
              .999ldots + .999ldots
              &= (.9 + .09 + .009 + cdots) + (.9 + .09 + .009 + cdots) \
              &= (.9 + .9) + (.09 + .09) + (.009 + .009) + cdots \
              &= 1.8 + .18 + .018 + .0018 + cdots \
              &= (1 + .8) + (.1 + .08) + (.01 + .008) + (.001 + .0008) + cdots \
              &= 1 + (.8 + .1) + (.08 + .01) + (.008 + .001) + cdots \
              &= 1 + .9 + .09 + .009 + cdots \
              &= 1 + .999ldots
              end{align*}
              It follows subtracting $.999ldots$ from both sides that $.999ldots = 1$.



              The reason I like this explanation best is that addition of (positive) infinite decimal expansions (defined in a particular way) is both commutative and associative even if you insist that $.999ldots$ and $1$ are different numbers. That is, it forms a commutative monoid.
              But the cancellation property fails: if $a + b = a + c$, then we can't necessarily conclude $b = c$. The example of this is above, and the most fundamental reason why $.999ldots = 1$ is arguably so that the cancellation property can hold.





              $^{*}$The calculation given here (using rearrangmenet and regrouping of terms) is informal, and not intended to be a proof, but rather to give some idea of how you can add infinite decimal expansions in the monoid where $.999ldots ne 1$. It does end up being true that $.999ldots + .999ldots = 1 + .999ldots$ in this monoid.






              share|cite|improve this answer











              $endgroup$









              • 2




                $begingroup$
                It doesn't feel convincing to me to assume that blithely that you can rearrange the infinite sums the way you do here, without changing their value. It is true that it is safe in this particular case, but I don't see how one can show that except by developing enough of a theory of limits that we might as well just sum $sum_{n=1}^infty 9/10^n$ directly.
                $endgroup$
                – Henning Makholm
                Jul 17 '16 at 2:46












              • $begingroup$
                @HenningMakholm Right--it's not a convincing proof that $.999ldots = 1$ unless you accept that infinite sums of real numbers can be rearranged and regrouped in any way (which is true of course but much more nontrivial than the basic result that the limit of $.999ldots$ is $1$).
                $endgroup$
                – 6005
                Jul 17 '16 at 4:19












              • $begingroup$
                My actual point is that you can define the addition of infinite expansions even while you insist that $.999ldots ne 1$, and if you do so (although I don't give the definition) you get $.999ldots + .999ldots = 1 + .999ldots$. The aligned equation is just informal rearrangements but is intended to sort of show how the definition of addition of infinite decimal expansions would work, in this case at least.
                $endgroup$
                – 6005
                Jul 17 '16 at 4:20








              • 1




                $begingroup$
                It is not true in general that "infinite sums of real numbers can be rearranged and regrouped in any way". It works in this particular case because all of the terms being rearranged are all positive -- but in order to argue that this is safer than the general case (rearranging a conditionally convergent series such as $sum frac{(-1)^n}{n}$ fails spectacularly) it seems that you need to develop so much material that you cannot really keep open an option for $0.999...$ to differ from $1$.
                $endgroup$
                – Henning Makholm
                Jul 17 '16 at 9:12












              • $begingroup$
                @HenningMakholm Oh I'm sorry, that was a typo. Of course I'm aware. I meant positive real numbers.
                $endgroup$
                – 6005
                Jul 17 '16 at 14:43



















              3












              $begingroup$

              Use the Squeeze Theorem:



              $$0<1-0.999...=0.1+0.9-0.999...=0.1-0.0999...<0.1=0.1^1$$
              $$0<0.1-0.0999...=0.01+0.09-0.0999...=0.01-0.00999...<0.01=0.1^2$$
              $$...$$
              $$0<1-0.999...<0.1^n$$
              $$0le 1-0.999... le limlimits_{ntoinfty}0.1^n=0.$$






              share|cite|improve this answer











              $endgroup$





















                1












                $begingroup$

                If we take a version of decimal notation with full complement it is indeed so.



                In this system, instead of allowing $0.2$ and $0.1999...$ we restrict decimal notation to use only infinite version $0.19999...$ shortly written as $0.1overline{9}$



                Simply, we do not allow an infinite trail of zeros.



                In this system, there is no $0$ written as $0.000...$ instead it is $...999.999...$ or with our succinct notation $overline{9}.overline{9}$



                Negative numbers are written in complement notation. For example, $...998.999...=overline{9}8.overline{9}=-1$



                All rules of multiplication addition subtraction are totally valid.



                In this system, it is indeed $0.99999...=0.overline{9}=1$ because we cannot represent $1$ as $1.0000...$.






                share|cite|improve this answer











                $endgroup$





















                  0












                  $begingroup$

                  If you allow a "decimal representation" of a number to end finally with period $9$ as in $0.bar9$ or $1.123bar9$ this "decimal representation "of a number would not be unique.



                  We know by definition that $0.bar9=sum_{n=1}^{infty}(9/10)^n=1$, but for the sake of the uniquenes of the decimal representation $0.bar9$ is not a decimal representation of any number.






                  share|cite|improve this answer











                  $endgroup$





















                    0












                    $begingroup$

                    Edit: I totally rewrote this answer in a completely different way than before. I decided to write it as another answer instead of undeleting my existing answer because it was posted as Community Wiki and Community Wiki answers are supposed to be pretty infrequent.



                    Edit: I see that it ended up being Community Wiki anyway without me purposely making it that way. Maybe the previous answer was marked as Community Wiki to limit the number of questions I answer not as Community Wiki because some of my answers weren't that great so it would have been better for me not to write another answer even if I could not undelete my other answer because it was Community Wiki.



                    Some people claim that $(mathbb{R}, 1, 0, +, times, leq)$ is a complete ordered field from which we can deduce that 0.999... = 1. Now how to we know that it's a complete ordered field? Because we can construct a complete ordered field in ZF and show that it's a complete ordered field which is unique up to isomorphism. First, we construct the natural numbers as finite ordinal numbers and define +, $times$, and $leq$ on them. 0 is not the successor of any number so we invent the number -1. -1 is still not the successor of any number so we invent the number -2 and keep going to get the integers and then redefine +, $times$, and $leq$ on them in the intuitive way. Now each odd number $x$ is not a solution to $2 times y = x$ so we invent a number $y$ to be the solution. Let's call those newly invented numbers the half integers. Now each half integer $y$ is still not a solution to $2 times z = y$ so we invent solutions to them and keep going forever to get the dyadic rationals and again redefine +, $times$, and $leq$ on them. Take any subset of the set of dyadic rationals such that that set and its complement are nonempty and for any dyadic rational in the subset, all smaller dyadic rationals are in the subset. Some of them have the additional property that they have no maximal element nor does their complement have a minimal element. For each one where that's the case, we can invent a number that's larger than all dyadic rationals in the subset and smaller than all dyadic rationals in its complement. We can call the set of all real numbers the set of all numbers constructed in this way and then redefine +, $times$, and $leq$ on that set in the intuitive way and show that $(mathbb{R}, text{{{}}}, text{{}}, +, times, leq)$ is a complete ordered field.



                    Some people might not want to use the fact that there exists a complete ordered field which is unique up to isomprphism. Some might prefer to use the fact that there exists a set with operations satisfying a different property which is unique up to isomorphism as I will describe. Take any totally ordered field with the property that if you take any subset of it such that it's nonempty and its complement is nonempty and for any number in the subset, all smaller numbers in the set and you can take a number in the set and a number not in the set arbitrarily close, then either the set has a maximal element or its complement has a minimal element. The real number system satisfies that property but so does the hyperreal number system. Some people might instead want to use the fact that if you add the criterion that there are no infinite numbers in the system, there is a set with operations satisfying those criteria which is unique up to isomporphism. It turns out that given the criteria for a hyperreal system I described, the system is complete if and only if it has no infinite numbers. Any set with those operations satisfying one set of criteria satisfies the other set of criteria so we don't need to disagree on the structure of the real numbers.



                    I've shown only how to construct the real numbers themselves. Inventing a decimal notation for all of them is another thing. It can be show that in that system all Cauchy sequences approach a real number. A decimal notation can be defined as a limit and since the limit exists for all notations, all notations represent a number. It can also be shown that all real numbers have a decimal notation according to that definition and some of them have 2 notations according to that definition. I read that some authors forbid a notation with a string of trailing 9's. You can also construct a set with +, $times$, and $leq$ a similar way to the way I just did except that you replace the number 2 with 10 and show that it's a complete ordered field and therefore isomorphic to the one I constructed. That also gives an intuitive way to define a decimal notation for each real number but it defines only one notation for each number and forbids a string of trailing 9's. It can also be shown that this notation for each real number is also one of the notations using the other definition of a decimal notation.






                    share|cite|improve this answer











                    $endgroup$













                    • $begingroup$
                      I didn't click the Community Wiki box. Why is this answer Community Wiki? Is it because there was no box this time because it's better for me to post it as Community Wiki because I'm not that great at answering questions?
                      $endgroup$
                      – Timothy
                      Dec 23 '18 at 5:49






                    • 1




                      $begingroup$
                      I think it might be because the question is now set to Community Wiki—I've just checked through and every answer is set that way (including mine which I've been thinking about deleting).
                      $endgroup$
                      – timtfj
                      Dec 25 '18 at 1:07





















                    -1












                    $begingroup$

                    The direct proof:




                    $$0.9999999999...=lim_{ntoinfty} left( 1-frac {1}{10^n}right)=1-0=1$$




                    Q.E.D.






                    share|cite|improve this answer











                    $endgroup$





















                      -2












                      $begingroup$

                      [Note: this is my original answer, but completely rewritten to clarify its purpose.]



                      This answer takes up Trevor Richards' point that people asking this question often aren't convinced by rigorous mathematical proofs and instead feel tricked by them. In this situation one thing that might help is a convincing visible demonstration that $0.999999 . . . =1$ has some chance of being true.



                      The usual demonstration consists of getting someone to agree that $frac13=0.33333 . . . $ and then multiply it by $3$ to get $0.99999 . . . $. At this point they might be convinced, but might equally feel puzzled or duped.



                      This, I think, is where more examples come in. We have to see that $frac13$ isn't some sort of special case that can be used to trick us.



                      When I first encountered $0.999999. . .$, I found looking at multiples of $frac19$ helpful. Once you've convinced yourself that this can be represented by an infinite string of $1$'s, it's easy to see that repeatedly adding it gives $0.222222. . .$, $0.333333. . .$, $0.444444. . .$ all the way up to $0.999999. . .$



                      There's a complete inevitability about this process, especially if you write it out on paper. But . . . maybe it's still just a trick with a repeating digit?



                      OK then: let's try multiples of $frac17=0.142857 . . . $. This one's fun because of the way the cycle of digits behaves:



                      $frac17=0.142857 . . .$
                      $frac27=0.285714 . . .$
                      $frac37=0.428571 . . .$



                      and the pattern continues nicely, and soon it's "obvious" that the digits will just keep rotating round. But then, all of a sudden, they don't:



                      $frac67=0.857142 . . .$
                      $frac77=0. 999999 . . .$



                      There it is again!



                      We can try with other fractions too, like $frac{1}{13}$ and $frac{1}{37}$, that recur after a manageable number of digits. Always we end up at $0.999999 . . .$.



                      At this stage, it should seem clear (but not formally proved) that accepting the idea of infinitely recurring decimals entails accepting that $0.999999. . . =1$.



                      The remaining issue, of course, is the acceptance of infinitely recurring decimals. That's addressed in other answers.






                      share|cite|improve this answer











                      $endgroup$













                      • $begingroup$
                        I think this answer could be improved to explain more clearly and be like Noldorin's answer but Noldorin's answer already exists to this answer doesn't add anything to it.
                        $endgroup$
                        – Timothy
                        Dec 24 '18 at 0:15










                      • $begingroup$
                        @Timothy My point was the psychology of it: by the time someone has worked their way up all the way from $frac19$ to $frac89$ it'll be obvious to them what's going to happen. I believe the block is more psychological than mathematical.
                        $endgroup$
                        – timtfj
                        Dec 24 '18 at 0:28





















                      26 Answers
                      26






                      active

                      oldest

                      votes








                      26 Answers
                      26






                      active

                      oldest

                      votes









                      active

                      oldest

                      votes






                      active

                      oldest

                      votes









                      347












                      $begingroup$

                      What does it mean when you refer to $.99999ldots$? Symbols don't mean anything in particular until you've defined what you mean by them.



                      In this case the definition is that you are taking the limit of $.9$, $.99$, $.999$, $.9999$, etc. What does it mean to say that limit is $1$? Well, it means that no matter how small a number $x$ you pick, I can show you a point in that sequence such that all further numbers in the sequence are within distance $x$ of $1$. But certainly whatever number you choose your number is bigger than $10^{-k}$ for some $k$. So I can just pick my point to be the $k$th spot in the sequence.



                      A more intuitive way of explaining the above argument is that the reason $.99999ldots = 1$ is that their difference is zero. So let's subtract $1.0000ldots -.99999ldots = .00000ldots = 0$. That is,



                      $1.0 -.9 = .1$



                      $1.00-.99 = .01$



                      $1.000-.999=.001$,



                      $ldots$



                      $1.000ldots -.99999ldots = .000ldots = 0$






                      share|cite|improve this answer











                      $endgroup$













                      • $begingroup$
                        Comments are not for extended discussion; this conversation has been moved to chat.
                        $endgroup$
                        – quid
                        Jan 6 at 13:41
















                      347












                      $begingroup$

                      What does it mean when you refer to $.99999ldots$? Symbols don't mean anything in particular until you've defined what you mean by them.



                      In this case the definition is that you are taking the limit of $.9$, $.99$, $.999$, $.9999$, etc. What does it mean to say that limit is $1$? Well, it means that no matter how small a number $x$ you pick, I can show you a point in that sequence such that all further numbers in the sequence are within distance $x$ of $1$. But certainly whatever number you choose your number is bigger than $10^{-k}$ for some $k$. So I can just pick my point to be the $k$th spot in the sequence.



                      A more intuitive way of explaining the above argument is that the reason $.99999ldots = 1$ is that their difference is zero. So let's subtract $1.0000ldots -.99999ldots = .00000ldots = 0$. That is,



                      $1.0 -.9 = .1$



                      $1.00-.99 = .01$



                      $1.000-.999=.001$,



                      $ldots$



                      $1.000ldots -.99999ldots = .000ldots = 0$






                      share|cite|improve this answer











                      $endgroup$













                      • $begingroup$
                        Comments are not for extended discussion; this conversation has been moved to chat.
                        $endgroup$
                        – quid
                        Jan 6 at 13:41














                      347












                      347








                      347





                      $begingroup$

                      What does it mean when you refer to $.99999ldots$? Symbols don't mean anything in particular until you've defined what you mean by them.



                      In this case the definition is that you are taking the limit of $.9$, $.99$, $.999$, $.9999$, etc. What does it mean to say that limit is $1$? Well, it means that no matter how small a number $x$ you pick, I can show you a point in that sequence such that all further numbers in the sequence are within distance $x$ of $1$. But certainly whatever number you choose your number is bigger than $10^{-k}$ for some $k$. So I can just pick my point to be the $k$th spot in the sequence.



                      A more intuitive way of explaining the above argument is that the reason $.99999ldots = 1$ is that their difference is zero. So let's subtract $1.0000ldots -.99999ldots = .00000ldots = 0$. That is,



                      $1.0 -.9 = .1$



                      $1.00-.99 = .01$



                      $1.000-.999=.001$,



                      $ldots$



                      $1.000ldots -.99999ldots = .000ldots = 0$






                      share|cite|improve this answer











                      $endgroup$



                      What does it mean when you refer to $.99999ldots$? Symbols don't mean anything in particular until you've defined what you mean by them.



                      In this case the definition is that you are taking the limit of $.9$, $.99$, $.999$, $.9999$, etc. What does it mean to say that limit is $1$? Well, it means that no matter how small a number $x$ you pick, I can show you a point in that sequence such that all further numbers in the sequence are within distance $x$ of $1$. But certainly whatever number you choose your number is bigger than $10^{-k}$ for some $k$. So I can just pick my point to be the $k$th spot in the sequence.



                      A more intuitive way of explaining the above argument is that the reason $.99999ldots = 1$ is that their difference is zero. So let's subtract $1.0000ldots -.99999ldots = .00000ldots = 0$. That is,



                      $1.0 -.9 = .1$



                      $1.00-.99 = .01$



                      $1.000-.999=.001$,



                      $ldots$



                      $1.000ldots -.99999ldots = .000ldots = 0$







                      share|cite|improve this answer














                      share|cite|improve this answer



                      share|cite|improve this answer








                      edited May 8 '18 at 20:22


























                      community wiki





                      6 revs, 6 users 62%
                      Noah Snyder













                      • $begingroup$
                        Comments are not for extended discussion; this conversation has been moved to chat.
                        $endgroup$
                        – quid
                        Jan 6 at 13:41


















                      • $begingroup$
                        Comments are not for extended discussion; this conversation has been moved to chat.
                        $endgroup$
                        – quid
                        Jan 6 at 13:41
















                      $begingroup$
                      Comments are not for extended discussion; this conversation has been moved to chat.
                      $endgroup$
                      – quid
                      Jan 6 at 13:41




                      $begingroup$
                      Comments are not for extended discussion; this conversation has been moved to chat.
                      $endgroup$
                      – quid
                      Jan 6 at 13:41











                      255












                      $begingroup$

                      Suppose this was not the case, i.e. $0.9999... neq 1$. Then $0.9999... < 1$ (I hope we agree on that). But between two distinct real numbers, there's always another one (say $x$) in between, hence $0.9999... < x < 1$.



                      The decimal representation of $x$ must have a digit somewhere that is not $9$ (otherwise $x = 0.9999...$). But that means it's actually smaller, $x < 0.9999...$, contradicting the definition of $x$.



                      Thus, the assumption that there's a number between $0.9999...$ and $1$ is false, hence they're equal.






                      share|cite|improve this answer











                      $endgroup$













                      • $begingroup$
                        Comments are not for extended discussion; this conversation has been moved to chat.
                        $endgroup$
                        – quid
                        Jan 6 at 13:43
















                      255












                      $begingroup$

                      Suppose this was not the case, i.e. $0.9999... neq 1$. Then $0.9999... < 1$ (I hope we agree on that). But between two distinct real numbers, there's always another one (say $x$) in between, hence $0.9999... < x < 1$.



                      The decimal representation of $x$ must have a digit somewhere that is not $9$ (otherwise $x = 0.9999...$). But that means it's actually smaller, $x < 0.9999...$, contradicting the definition of $x$.



                      Thus, the assumption that there's a number between $0.9999...$ and $1$ is false, hence they're equal.






                      share|cite|improve this answer











                      $endgroup$













                      • $begingroup$
                        Comments are not for extended discussion; this conversation has been moved to chat.
                        $endgroup$
                        – quid
                        Jan 6 at 13:43














                      255












                      255








                      255





                      $begingroup$

                      Suppose this was not the case, i.e. $0.9999... neq 1$. Then $0.9999... < 1$ (I hope we agree on that). But between two distinct real numbers, there's always another one (say $x$) in between, hence $0.9999... < x < 1$.



                      The decimal representation of $x$ must have a digit somewhere that is not $9$ (otherwise $x = 0.9999...$). But that means it's actually smaller, $x < 0.9999...$, contradicting the definition of $x$.



                      Thus, the assumption that there's a number between $0.9999...$ and $1$ is false, hence they're equal.






                      share|cite|improve this answer











                      $endgroup$



                      Suppose this was not the case, i.e. $0.9999... neq 1$. Then $0.9999... < 1$ (I hope we agree on that). But between two distinct real numbers, there's always another one (say $x$) in between, hence $0.9999... < x < 1$.



                      The decimal representation of $x$ must have a digit somewhere that is not $9$ (otherwise $x = 0.9999...$). But that means it's actually smaller, $x < 0.9999...$, contradicting the definition of $x$.



                      Thus, the assumption that there's a number between $0.9999...$ and $1$ is false, hence they're equal.







                      share|cite|improve this answer














                      share|cite|improve this answer



                      share|cite|improve this answer








                      edited Dec 15 '16 at 20:28


























                      community wiki





                      4 revs, 4 users 57%
                      balpha













                      • $begingroup$
                        Comments are not for extended discussion; this conversation has been moved to chat.
                        $endgroup$
                        – quid
                        Jan 6 at 13:43


















                      • $begingroup$
                        Comments are not for extended discussion; this conversation has been moved to chat.
                        $endgroup$
                        – quid
                        Jan 6 at 13:43
















                      $begingroup$
                      Comments are not for extended discussion; this conversation has been moved to chat.
                      $endgroup$
                      – quid
                      Jan 6 at 13:43




                      $begingroup$
                      Comments are not for extended discussion; this conversation has been moved to chat.
                      $endgroup$
                      – quid
                      Jan 6 at 13:43











                      164












                      $begingroup$

                      What I really don't like about all the above answers, is the underlying assumption that $1/3=0.3333ldots$ How do you know that? It seems to me like assuming the something which is already known.



                      A proof I really like is:



                      $$begin{align}
                      0.9999ldots × 10 &= 9.9999ldots\
                      0.9999ldots × (9+1) &= 9.9999ldots\
                      text{by distribution rule: }Space{15ex}{0ex}{0ex} \
                      0.9999ldots × 9 + 0.9999ldots × 1 &= 9.9999ldots\
                      0.9999ldots × 9 &= 9.9999dots-0.9999ldots\
                      0.9999ldots × 9 &= 9\
                      0.9999ldots &= 1
                      end{align}$$



                      The only things I need to assume is, that $9.999ldots - 0.999ldots = 9$ and that $0.999ldots × 10 = 9.999ldots$ These seems to me intuitive enough to take for granted.



                      The proof is from an old high school level math book of the Open University in Israel.






                      share|cite|improve this answer











                      $endgroup$













                      • $begingroup$
                        Comments are not for extended discussion; this conversation has been moved to chat.
                        $endgroup$
                        – quid
                        Jan 6 at 13:44
















                      164












                      $begingroup$

                      What I really don't like about all the above answers, is the underlying assumption that $1/3=0.3333ldots$ How do you know that? It seems to me like assuming the something which is already known.



                      A proof I really like is:



                      $$begin{align}
                      0.9999ldots × 10 &= 9.9999ldots\
                      0.9999ldots × (9+1) &= 9.9999ldots\
                      text{by distribution rule: }Space{15ex}{0ex}{0ex} \
                      0.9999ldots × 9 + 0.9999ldots × 1 &= 9.9999ldots\
                      0.9999ldots × 9 &= 9.9999dots-0.9999ldots\
                      0.9999ldots × 9 &= 9\
                      0.9999ldots &= 1
                      end{align}$$



                      The only things I need to assume is, that $9.999ldots - 0.999ldots = 9$ and that $0.999ldots × 10 = 9.999ldots$ These seems to me intuitive enough to take for granted.



                      The proof is from an old high school level math book of the Open University in Israel.






                      share|cite|improve this answer











                      $endgroup$













                      • $begingroup$
                        Comments are not for extended discussion; this conversation has been moved to chat.
                        $endgroup$
                        – quid
                        Jan 6 at 13:44














                      164












                      164








                      164





                      $begingroup$

                      What I really don't like about all the above answers, is the underlying assumption that $1/3=0.3333ldots$ How do you know that? It seems to me like assuming the something which is already known.



                      A proof I really like is:



                      $$begin{align}
                      0.9999ldots × 10 &= 9.9999ldots\
                      0.9999ldots × (9+1) &= 9.9999ldots\
                      text{by distribution rule: }Space{15ex}{0ex}{0ex} \
                      0.9999ldots × 9 + 0.9999ldots × 1 &= 9.9999ldots\
                      0.9999ldots × 9 &= 9.9999dots-0.9999ldots\
                      0.9999ldots × 9 &= 9\
                      0.9999ldots &= 1
                      end{align}$$



                      The only things I need to assume is, that $9.999ldots - 0.999ldots = 9$ and that $0.999ldots × 10 = 9.999ldots$ These seems to me intuitive enough to take for granted.



                      The proof is from an old high school level math book of the Open University in Israel.






                      share|cite|improve this answer











                      $endgroup$



                      What I really don't like about all the above answers, is the underlying assumption that $1/3=0.3333ldots$ How do you know that? It seems to me like assuming the something which is already known.



                      A proof I really like is:



                      $$begin{align}
                      0.9999ldots × 10 &= 9.9999ldots\
                      0.9999ldots × (9+1) &= 9.9999ldots\
                      text{by distribution rule: }Space{15ex}{0ex}{0ex} \
                      0.9999ldots × 9 + 0.9999ldots × 1 &= 9.9999ldots\
                      0.9999ldots × 9 &= 9.9999dots-0.9999ldots\
                      0.9999ldots × 9 &= 9\
                      0.9999ldots &= 1
                      end{align}$$



                      The only things I need to assume is, that $9.999ldots - 0.999ldots = 9$ and that $0.999ldots × 10 = 9.999ldots$ These seems to me intuitive enough to take for granted.



                      The proof is from an old high school level math book of the Open University in Israel.







                      share|cite|improve this answer














                      share|cite|improve this answer



                      share|cite|improve this answer








                      edited Oct 18 '14 at 13:45


























                      community wiki





                      5 revs, 3 users 72%
                      Elazar Leibovich













                      • $begingroup$
                        Comments are not for extended discussion; this conversation has been moved to chat.
                        $endgroup$
                        – quid
                        Jan 6 at 13:44


















                      • $begingroup$
                        Comments are not for extended discussion; this conversation has been moved to chat.
                        $endgroup$
                        – quid
                        Jan 6 at 13:44
















                      $begingroup$
                      Comments are not for extended discussion; this conversation has been moved to chat.
                      $endgroup$
                      – quid
                      Jan 6 at 13:44




                      $begingroup$
                      Comments are not for extended discussion; this conversation has been moved to chat.
                      $endgroup$
                      – quid
                      Jan 6 at 13:44











                      151












                      $begingroup$

                      Assuming:




                      1. infinite decimals are series where the terms are the digits divided by the proper power of the base

                      2. the infinite geometric series $a + a cdot r + a cdot r^2 + a cdot r^3 + cdots$ has sum $dfrac{a}{1 - r}$ as long as $|r|<1$




                      $$0.99999ldots = frac{9}{10} + frac{9}{10^2} + frac{9}{10^3} + cdots$$



                      This is the infinite geometric series with first term $a = frac{9}{10}$ and common ratio $r = frac{1}{10}$, so it has sum $$frac{frac{9}{10}}{1 - frac{1}{10}} = frac{frac{9}{10}}{frac{9}{10}} = 1.$$






                      share|cite|improve this answer











                      $endgroup$









                      • 9




                        $begingroup$
                        Your method is a simple way of converting the decimal representation of a rational number into a fraction, e.g. $0.150150150...=sum_{ngeq 1}frac{150}{10^{3n}}=frac{0.150}{1-10^{-3}}=frac{50}{333}$
                        $endgroup$
                        – Américo Tavares
                        Aug 16 '10 at 22:02












                      • $begingroup$
                        The question is where are you getting this common ratio of 1/10 and what law permits you to use it in this case?
                        $endgroup$
                        – Serj Sagan
                        Aug 2 '16 at 19:11










                      • $begingroup$
                        @SerjSagan Generally, when not otherwise indicated, numbers are written in base 10. That makes the ratio of the values of successive "places" 10 or $frac{1}{10}$, depending on direction. This is the "... proper power of the base" piece of my point 1.
                        $endgroup$
                        – Isaac
                        Aug 2 '16 at 19:17


















                      151












                      $begingroup$

                      Assuming:




                      1. infinite decimals are series where the terms are the digits divided by the proper power of the base

                      2. the infinite geometric series $a + a cdot r + a cdot r^2 + a cdot r^3 + cdots$ has sum $dfrac{a}{1 - r}$ as long as $|r|<1$




                      $$0.99999ldots = frac{9}{10} + frac{9}{10^2} + frac{9}{10^3} + cdots$$



                      This is the infinite geometric series with first term $a = frac{9}{10}$ and common ratio $r = frac{1}{10}$, so it has sum $$frac{frac{9}{10}}{1 - frac{1}{10}} = frac{frac{9}{10}}{frac{9}{10}} = 1.$$






                      share|cite|improve this answer











                      $endgroup$









                      • 9




                        $begingroup$
                        Your method is a simple way of converting the decimal representation of a rational number into a fraction, e.g. $0.150150150...=sum_{ngeq 1}frac{150}{10^{3n}}=frac{0.150}{1-10^{-3}}=frac{50}{333}$
                        $endgroup$
                        – Américo Tavares
                        Aug 16 '10 at 22:02












                      • $begingroup$
                        The question is where are you getting this common ratio of 1/10 and what law permits you to use it in this case?
                        $endgroup$
                        – Serj Sagan
                        Aug 2 '16 at 19:11










                      • $begingroup$
                        @SerjSagan Generally, when not otherwise indicated, numbers are written in base 10. That makes the ratio of the values of successive "places" 10 or $frac{1}{10}$, depending on direction. This is the "... proper power of the base" piece of my point 1.
                        $endgroup$
                        – Isaac
                        Aug 2 '16 at 19:17
















                      151












                      151








                      151





                      $begingroup$

                      Assuming:




                      1. infinite decimals are series where the terms are the digits divided by the proper power of the base

                      2. the infinite geometric series $a + a cdot r + a cdot r^2 + a cdot r^3 + cdots$ has sum $dfrac{a}{1 - r}$ as long as $|r|<1$




                      $$0.99999ldots = frac{9}{10} + frac{9}{10^2} + frac{9}{10^3} + cdots$$



                      This is the infinite geometric series with first term $a = frac{9}{10}$ and common ratio $r = frac{1}{10}$, so it has sum $$frac{frac{9}{10}}{1 - frac{1}{10}} = frac{frac{9}{10}}{frac{9}{10}} = 1.$$






                      share|cite|improve this answer











                      $endgroup$



                      Assuming:




                      1. infinite decimals are series where the terms are the digits divided by the proper power of the base

                      2. the infinite geometric series $a + a cdot r + a cdot r^2 + a cdot r^3 + cdots$ has sum $dfrac{a}{1 - r}$ as long as $|r|<1$




                      $$0.99999ldots = frac{9}{10} + frac{9}{10^2} + frac{9}{10^3} + cdots$$



                      This is the infinite geometric series with first term $a = frac{9}{10}$ and common ratio $r = frac{1}{10}$, so it has sum $$frac{frac{9}{10}}{1 - frac{1}{10}} = frac{frac{9}{10}}{frac{9}{10}} = 1.$$







                      share|cite|improve this answer














                      share|cite|improve this answer



                      share|cite|improve this answer








                      edited Nov 16 '13 at 16:42


























                      community wiki





                      2 revs
                      Isaac









                      • 9




                        $begingroup$
                        Your method is a simple way of converting the decimal representation of a rational number into a fraction, e.g. $0.150150150...=sum_{ngeq 1}frac{150}{10^{3n}}=frac{0.150}{1-10^{-3}}=frac{50}{333}$
                        $endgroup$
                        – Américo Tavares
                        Aug 16 '10 at 22:02












                      • $begingroup$
                        The question is where are you getting this common ratio of 1/10 and what law permits you to use it in this case?
                        $endgroup$
                        – Serj Sagan
                        Aug 2 '16 at 19:11










                      • $begingroup$
                        @SerjSagan Generally, when not otherwise indicated, numbers are written in base 10. That makes the ratio of the values of successive "places" 10 or $frac{1}{10}$, depending on direction. This is the "... proper power of the base" piece of my point 1.
                        $endgroup$
                        – Isaac
                        Aug 2 '16 at 19:17
















                      • 9




                        $begingroup$
                        Your method is a simple way of converting the decimal representation of a rational number into a fraction, e.g. $0.150150150...=sum_{ngeq 1}frac{150}{10^{3n}}=frac{0.150}{1-10^{-3}}=frac{50}{333}$
                        $endgroup$
                        – Américo Tavares
                        Aug 16 '10 at 22:02












                      • $begingroup$
                        The question is where are you getting this common ratio of 1/10 and what law permits you to use it in this case?
                        $endgroup$
                        – Serj Sagan
                        Aug 2 '16 at 19:11










                      • $begingroup$
                        @SerjSagan Generally, when not otherwise indicated, numbers are written in base 10. That makes the ratio of the values of successive "places" 10 or $frac{1}{10}$, depending on direction. This is the "... proper power of the base" piece of my point 1.
                        $endgroup$
                        – Isaac
                        Aug 2 '16 at 19:17










                      9




                      9




                      $begingroup$
                      Your method is a simple way of converting the decimal representation of a rational number into a fraction, e.g. $0.150150150...=sum_{ngeq 1}frac{150}{10^{3n}}=frac{0.150}{1-10^{-3}}=frac{50}{333}$
                      $endgroup$
                      – Américo Tavares
                      Aug 16 '10 at 22:02






                      $begingroup$
                      Your method is a simple way of converting the decimal representation of a rational number into a fraction, e.g. $0.150150150...=sum_{ngeq 1}frac{150}{10^{3n}}=frac{0.150}{1-10^{-3}}=frac{50}{333}$
                      $endgroup$
                      – Américo Tavares
                      Aug 16 '10 at 22:02














                      $begingroup$
                      The question is where are you getting this common ratio of 1/10 and what law permits you to use it in this case?
                      $endgroup$
                      – Serj Sagan
                      Aug 2 '16 at 19:11




                      $begingroup$
                      The question is where are you getting this common ratio of 1/10 and what law permits you to use it in this case?
                      $endgroup$
                      – Serj Sagan
                      Aug 2 '16 at 19:11












                      $begingroup$
                      @SerjSagan Generally, when not otherwise indicated, numbers are written in base 10. That makes the ratio of the values of successive "places" 10 or $frac{1}{10}$, depending on direction. This is the "... proper power of the base" piece of my point 1.
                      $endgroup$
                      – Isaac
                      Aug 2 '16 at 19:17






                      $begingroup$
                      @SerjSagan Generally, when not otherwise indicated, numbers are written in base 10. That makes the ratio of the values of successive "places" 10 or $frac{1}{10}$, depending on direction. This is the "... proper power of the base" piece of my point 1.
                      $endgroup$
                      – Isaac
                      Aug 2 '16 at 19:17













                      71












                      $begingroup$

                      $$x=0.999...$$
                      $$10x=9.999...$$
                      $$10x-x=9.999...-0.999...$$
                      $$9x=9$$
                      $$x=1$$



                      thus, $0.999...=1$






                      share|cite|improve this answer











                      $endgroup$









                      • 1




                        $begingroup$
                        This is the most intuitive argument, although some might say "But 10x-x isn't 9, because there's going to be a mismatch all the way to the right" - Noah's more complex deals with that.
                        $endgroup$
                        – Charles Stewart
                        Jul 21 '10 at 10:55










                      • $begingroup$
                        10x - x != 9. 10x - x would be 8.9999...1. However infinite the extent of 9s is in x, if we multiply it by 10, the nines are shifted left by one position and a zero inserted at the "last" place, and then when you subtract the other number there is a nine subtracted from a zero at the far right. Otherwise we'd have to give 0.999.. some unusual properties like automatically increasing the number of nines when it is multiplied. It would not be just an ordinary number. Maybe that's the problem. 0.999... might just not be an ordinary type number as some people are using it.
                        $endgroup$
                        – Doug Treadwell
                        Jul 23 '10 at 2:19






                      • 8




                        $begingroup$
                        @Doug It's incorrect to talk about the "number of nines" because infinity minus a number = infinity. Infinity is transcendent. It means "uncountable". If you take infinity and slide it left a little bit, it's still infinity long.
                        $endgroup$
                        – ErikE
                        Sep 25 '10 at 7:00










                      • $begingroup$
                        The problem is the assumption that 9x = 9 when in fact 9x = 8.999... your proof is flawed.
                        $endgroup$
                        – Serj Sagan
                        Aug 2 '16 at 19:16






                      • 9




                        $begingroup$
                        @SerjSagan that line isn't an assumption, it's a deduction. If you believe that $9.999dots - 0.999dots = 9$, then you must therefore believe that $9x = 9$.
                        $endgroup$
                        – Omnomnomnom
                        Dec 15 '16 at 17:03
















                      71












                      $begingroup$

                      $$x=0.999...$$
                      $$10x=9.999...$$
                      $$10x-x=9.999...-0.999...$$
                      $$9x=9$$
                      $$x=1$$



                      thus, $0.999...=1$






                      share|cite|improve this answer











                      $endgroup$









                      • 1




                        $begingroup$
                        This is the most intuitive argument, although some might say "But 10x-x isn't 9, because there's going to be a mismatch all the way to the right" - Noah's more complex deals with that.
                        $endgroup$
                        – Charles Stewart
                        Jul 21 '10 at 10:55










                      • $begingroup$
                        10x - x != 9. 10x - x would be 8.9999...1. However infinite the extent of 9s is in x, if we multiply it by 10, the nines are shifted left by one position and a zero inserted at the "last" place, and then when you subtract the other number there is a nine subtracted from a zero at the far right. Otherwise we'd have to give 0.999.. some unusual properties like automatically increasing the number of nines when it is multiplied. It would not be just an ordinary number. Maybe that's the problem. 0.999... might just not be an ordinary type number as some people are using it.
                        $endgroup$
                        – Doug Treadwell
                        Jul 23 '10 at 2:19






                      • 8




                        $begingroup$
                        @Doug It's incorrect to talk about the "number of nines" because infinity minus a number = infinity. Infinity is transcendent. It means "uncountable". If you take infinity and slide it left a little bit, it's still infinity long.
                        $endgroup$
                        – ErikE
                        Sep 25 '10 at 7:00










                      • $begingroup$
                        The problem is the assumption that 9x = 9 when in fact 9x = 8.999... your proof is flawed.
                        $endgroup$
                        – Serj Sagan
                        Aug 2 '16 at 19:16






                      • 9




                        $begingroup$
                        @SerjSagan that line isn't an assumption, it's a deduction. If you believe that $9.999dots - 0.999dots = 9$, then you must therefore believe that $9x = 9$.
                        $endgroup$
                        – Omnomnomnom
                        Dec 15 '16 at 17:03














                      71












                      71








                      71





                      $begingroup$

                      $$x=0.999...$$
                      $$10x=9.999...$$
                      $$10x-x=9.999...-0.999...$$
                      $$9x=9$$
                      $$x=1$$



                      thus, $0.999...=1$






                      share|cite|improve this answer











                      $endgroup$



                      $$x=0.999...$$
                      $$10x=9.999...$$
                      $$10x-x=9.999...-0.999...$$
                      $$9x=9$$
                      $$x=1$$



                      thus, $0.999...=1$







                      share|cite|improve this answer














                      share|cite|improve this answer



                      share|cite|improve this answer








                      edited Dec 18 '16 at 16:54


























                      community wiki





                      2 revs, 2 users 53%
                      Pieces









                      • 1




                        $begingroup$
                        This is the most intuitive argument, although some might say "But 10x-x isn't 9, because there's going to be a mismatch all the way to the right" - Noah's more complex deals with that.
                        $endgroup$
                        – Charles Stewart
                        Jul 21 '10 at 10:55










                      • $begingroup$
                        10x - x != 9. 10x - x would be 8.9999...1. However infinite the extent of 9s is in x, if we multiply it by 10, the nines are shifted left by one position and a zero inserted at the "last" place, and then when you subtract the other number there is a nine subtracted from a zero at the far right. Otherwise we'd have to give 0.999.. some unusual properties like automatically increasing the number of nines when it is multiplied. It would not be just an ordinary number. Maybe that's the problem. 0.999... might just not be an ordinary type number as some people are using it.
                        $endgroup$
                        – Doug Treadwell
                        Jul 23 '10 at 2:19






                      • 8




                        $begingroup$
                        @Doug It's incorrect to talk about the "number of nines" because infinity minus a number = infinity. Infinity is transcendent. It means "uncountable". If you take infinity and slide it left a little bit, it's still infinity long.
                        $endgroup$
                        – ErikE
                        Sep 25 '10 at 7:00










                      • $begingroup$
                        The problem is the assumption that 9x = 9 when in fact 9x = 8.999... your proof is flawed.
                        $endgroup$
                        – Serj Sagan
                        Aug 2 '16 at 19:16






                      • 9




                        $begingroup$
                        @SerjSagan that line isn't an assumption, it's a deduction. If you believe that $9.999dots - 0.999dots = 9$, then you must therefore believe that $9x = 9$.
                        $endgroup$
                        – Omnomnomnom
                        Dec 15 '16 at 17:03














                      • 1




                        $begingroup$
                        This is the most intuitive argument, although some might say "But 10x-x isn't 9, because there's going to be a mismatch all the way to the right" - Noah's more complex deals with that.
                        $endgroup$
                        – Charles Stewart
                        Jul 21 '10 at 10:55










                      • $begingroup$
                        10x - x != 9. 10x - x would be 8.9999...1. However infinite the extent of 9s is in x, if we multiply it by 10, the nines are shifted left by one position and a zero inserted at the "last" place, and then when you subtract the other number there is a nine subtracted from a zero at the far right. Otherwise we'd have to give 0.999.. some unusual properties like automatically increasing the number of nines when it is multiplied. It would not be just an ordinary number. Maybe that's the problem. 0.999... might just not be an ordinary type number as some people are using it.
                        $endgroup$
                        – Doug Treadwell
                        Jul 23 '10 at 2:19






                      • 8




                        $begingroup$
                        @Doug It's incorrect to talk about the "number of nines" because infinity minus a number = infinity. Infinity is transcendent. It means "uncountable". If you take infinity and slide it left a little bit, it's still infinity long.
                        $endgroup$
                        – ErikE
                        Sep 25 '10 at 7:00










                      • $begingroup$
                        The problem is the assumption that 9x = 9 when in fact 9x = 8.999... your proof is flawed.
                        $endgroup$
                        – Serj Sagan
                        Aug 2 '16 at 19:16






                      • 9




                        $begingroup$
                        @SerjSagan that line isn't an assumption, it's a deduction. If you believe that $9.999dots - 0.999dots = 9$, then you must therefore believe that $9x = 9$.
                        $endgroup$
                        – Omnomnomnom
                        Dec 15 '16 at 17:03








                      1




                      1




                      $begingroup$
                      This is the most intuitive argument, although some might say "But 10x-x isn't 9, because there's going to be a mismatch all the way to the right" - Noah's more complex deals with that.
                      $endgroup$
                      – Charles Stewart
                      Jul 21 '10 at 10:55




                      $begingroup$
                      This is the most intuitive argument, although some might say "But 10x-x isn't 9, because there's going to be a mismatch all the way to the right" - Noah's more complex deals with that.
                      $endgroup$
                      – Charles Stewart
                      Jul 21 '10 at 10:55












                      $begingroup$
                      10x - x != 9. 10x - x would be 8.9999...1. However infinite the extent of 9s is in x, if we multiply it by 10, the nines are shifted left by one position and a zero inserted at the "last" place, and then when you subtract the other number there is a nine subtracted from a zero at the far right. Otherwise we'd have to give 0.999.. some unusual properties like automatically increasing the number of nines when it is multiplied. It would not be just an ordinary number. Maybe that's the problem. 0.999... might just not be an ordinary type number as some people are using it.
                      $endgroup$
                      – Doug Treadwell
                      Jul 23 '10 at 2:19




                      $begingroup$
                      10x - x != 9. 10x - x would be 8.9999...1. However infinite the extent of 9s is in x, if we multiply it by 10, the nines are shifted left by one position and a zero inserted at the "last" place, and then when you subtract the other number there is a nine subtracted from a zero at the far right. Otherwise we'd have to give 0.999.. some unusual properties like automatically increasing the number of nines when it is multiplied. It would not be just an ordinary number. Maybe that's the problem. 0.999... might just not be an ordinary type number as some people are using it.
                      $endgroup$
                      – Doug Treadwell
                      Jul 23 '10 at 2:19




                      8




                      8




                      $begingroup$
                      @Doug It's incorrect to talk about the "number of nines" because infinity minus a number = infinity. Infinity is transcendent. It means "uncountable". If you take infinity and slide it left a little bit, it's still infinity long.
                      $endgroup$
                      – ErikE
                      Sep 25 '10 at 7:00




                      $begingroup$
                      @Doug It's incorrect to talk about the "number of nines" because infinity minus a number = infinity. Infinity is transcendent. It means "uncountable". If you take infinity and slide it left a little bit, it's still infinity long.
                      $endgroup$
                      – ErikE
                      Sep 25 '10 at 7:00












                      $begingroup$
                      The problem is the assumption that 9x = 9 when in fact 9x = 8.999... your proof is flawed.
                      $endgroup$
                      – Serj Sagan
                      Aug 2 '16 at 19:16




                      $begingroup$
                      The problem is the assumption that 9x = 9 when in fact 9x = 8.999... your proof is flawed.
                      $endgroup$
                      – Serj Sagan
                      Aug 2 '16 at 19:16




                      9




                      9




                      $begingroup$
                      @SerjSagan that line isn't an assumption, it's a deduction. If you believe that $9.999dots - 0.999dots = 9$, then you must therefore believe that $9x = 9$.
                      $endgroup$
                      – Omnomnomnom
                      Dec 15 '16 at 17:03




                      $begingroup$
                      @SerjSagan that line isn't an assumption, it's a deduction. If you believe that $9.999dots - 0.999dots = 9$, then you must therefore believe that $9x = 9$.
                      $endgroup$
                      – Omnomnomnom
                      Dec 15 '16 at 17:03











                      52












                      $begingroup$

                      Okay I burned a lot of reputation points (at least for me) on MathOverflow to gain clarity on how to give some intuition into this problem, so hopefully this answer will be at least be somewhat illuminating.



                      To gain a deeper understanding of what is going on, first we need to answer the question, "What is a number?"



                      There are a lot of ways to define numbers, but in general numbers are thought of as symbols that represent sets.



                      This is easy for things like the natural numbers. So 10 would correspond to the set with ten things -- like a bag of ten stones. Pretty straight forward.



                      The tricky part is that when we consider ten a subset of the real numbers, we actually redefine it. This is not emphasized even in higher mathematics classes, like real analysis; it just happens when we define the real numbers.



                      So what is 10 when constructed in the real numbers? Well, at least with the Dedekind cut version of the real numbers, all real numbers correspond to a set with an infinite amount of elements. This makes 10 under the hood look drastically different, although in practice it operates exactly the same.



                      So let's return to the question: Why is 10 the same as 9.99999? Because the real numbers have this completely surprising quality, where there is no next real number. So when you have two real numbers that are as close together as possible, they are the same. I can't think of any physical object that has this quality, but it's how the real numbers work (makes "real" seem ironic).



                      With integers (bag of stones version) this is not the same. When you have two integers as close to each other as possible they are still different, and they are distance one apart.



                      Put another way, 10 bag of stones are not the same as 9.9999999 but 10 the natural number, where natural numbers are a subset of the real numbers is.



                      The bottom line is that the real numbers have these tricky edge cases that are hard to understand intuitively. Don't worry, your intuition is not really failing you. :)



                      I didn't feel confident answering until I got this Terence Tao link:



                      (Wayback Machine) https://web.archive.org/web/20100725014132/http://www.google.com:80/buzz/114134834346472219368/RarPutThCJv/In-the-foundations-of-mathematics-the-standard



                      (PDF, page 12) https://terrytao.files.wordpress.com/2011/06/blog-book.pdf






                      share|cite|improve this answer











                      $endgroup$









                      • 4




                        $begingroup$
                        The link doesn't work.
                        $endgroup$
                        – user 170039
                        Jan 5 '16 at 15:08










                      • $begingroup$
                        I'm not sure that most people think of real numbers as representing sets. Indeed, probably few of those who didn't study mathematics or attend mathematics lectures have even heard of the Dedekind construction. The common visualization of real numbers is as points on a line.
                        $endgroup$
                        – celtschk
                        Aug 14 '16 at 9:03






                      • 2




                        $begingroup$
                        the link is dead
                        $endgroup$
                        – baxx
                        Sep 10 '16 at 20:43










                      • $begingroup$
                        As a smart person, I actually like this answer better than balpha and Elezar Laibovich's answer. It's good enough for me because I can figure out from it a fuller answer that explains how people were looking for a definition of a real number in ZF such that the set of all of them is a complete ordered field so they found one and proved it's a complete ordered field. math.stackexchange.com/questions/2437893/… actually says what a real natural number is. Those answers aren't bad either because I can prove that all real numbers have a decimal representation.
                        $endgroup$
                        – Timothy
                        May 17 '18 at 1:34
















                      52












                      $begingroup$

                      Okay I burned a lot of reputation points (at least for me) on MathOverflow to gain clarity on how to give some intuition into this problem, so hopefully this answer will be at least be somewhat illuminating.



                      To gain a deeper understanding of what is going on, first we need to answer the question, "What is a number?"



                      There are a lot of ways to define numbers, but in general numbers are thought of as symbols that represent sets.



                      This is easy for things like the natural numbers. So 10 would correspond to the set with ten things -- like a bag of ten stones. Pretty straight forward.



                      The tricky part is that when we consider ten a subset of the real numbers, we actually redefine it. This is not emphasized even in higher mathematics classes, like real analysis; it just happens when we define the real numbers.



                      So what is 10 when constructed in the real numbers? Well, at least with the Dedekind cut version of the real numbers, all real numbers correspond to a set with an infinite amount of elements. This makes 10 under the hood look drastically different, although in practice it operates exactly the same.



                      So let's return to the question: Why is 10 the same as 9.99999? Because the real numbers have this completely surprising quality, where there is no next real number. So when you have two real numbers that are as close together as possible, they are the same. I can't think of any physical object that has this quality, but it's how the real numbers work (makes "real" seem ironic).



                      With integers (bag of stones version) this is not the same. When you have two integers as close to each other as possible they are still different, and they are distance one apart.



                      Put another way, 10 bag of stones are not the same as 9.9999999 but 10 the natural number, where natural numbers are a subset of the real numbers is.



                      The bottom line is that the real numbers have these tricky edge cases that are hard to understand intuitively. Don't worry, your intuition is not really failing you. :)



                      I didn't feel confident answering until I got this Terence Tao link:



                      (Wayback Machine) https://web.archive.org/web/20100725014132/http://www.google.com:80/buzz/114134834346472219368/RarPutThCJv/In-the-foundations-of-mathematics-the-standard



                      (PDF, page 12) https://terrytao.files.wordpress.com/2011/06/blog-book.pdf






                      share|cite|improve this answer











                      $endgroup$









                      • 4




                        $begingroup$
                        The link doesn't work.
                        $endgroup$
                        – user 170039
                        Jan 5 '16 at 15:08










                      • $begingroup$
                        I'm not sure that most people think of real numbers as representing sets. Indeed, probably few of those who didn't study mathematics or attend mathematics lectures have even heard of the Dedekind construction. The common visualization of real numbers is as points on a line.
                        $endgroup$
                        – celtschk
                        Aug 14 '16 at 9:03






                      • 2




                        $begingroup$
                        the link is dead
                        $endgroup$
                        – baxx
                        Sep 10 '16 at 20:43










                      • $begingroup$
                        As a smart person, I actually like this answer better than balpha and Elezar Laibovich's answer. It's good enough for me because I can figure out from it a fuller answer that explains how people were looking for a definition of a real number in ZF such that the set of all of them is a complete ordered field so they found one and proved it's a complete ordered field. math.stackexchange.com/questions/2437893/… actually says what a real natural number is. Those answers aren't bad either because I can prove that all real numbers have a decimal representation.
                        $endgroup$
                        – Timothy
                        May 17 '18 at 1:34














                      52












                      52








                      52





                      $begingroup$

                      Okay I burned a lot of reputation points (at least for me) on MathOverflow to gain clarity on how to give some intuition into this problem, so hopefully this answer will be at least be somewhat illuminating.



                      To gain a deeper understanding of what is going on, first we need to answer the question, "What is a number?"



                      There are a lot of ways to define numbers, but in general numbers are thought of as symbols that represent sets.



                      This is easy for things like the natural numbers. So 10 would correspond to the set with ten things -- like a bag of ten stones. Pretty straight forward.



                      The tricky part is that when we consider ten a subset of the real numbers, we actually redefine it. This is not emphasized even in higher mathematics classes, like real analysis; it just happens when we define the real numbers.



                      So what is 10 when constructed in the real numbers? Well, at least with the Dedekind cut version of the real numbers, all real numbers correspond to a set with an infinite amount of elements. This makes 10 under the hood look drastically different, although in practice it operates exactly the same.



                      So let's return to the question: Why is 10 the same as 9.99999? Because the real numbers have this completely surprising quality, where there is no next real number. So when you have two real numbers that are as close together as possible, they are the same. I can't think of any physical object that has this quality, but it's how the real numbers work (makes "real" seem ironic).



                      With integers (bag of stones version) this is not the same. When you have two integers as close to each other as possible they are still different, and they are distance one apart.



                      Put another way, 10 bag of stones are not the same as 9.9999999 but 10 the natural number, where natural numbers are a subset of the real numbers is.



                      The bottom line is that the real numbers have these tricky edge cases that are hard to understand intuitively. Don't worry, your intuition is not really failing you. :)



                      I didn't feel confident answering until I got this Terence Tao link:



                      (Wayback Machine) https://web.archive.org/web/20100725014132/http://www.google.com:80/buzz/114134834346472219368/RarPutThCJv/In-the-foundations-of-mathematics-the-standard



                      (PDF, page 12) https://terrytao.files.wordpress.com/2011/06/blog-book.pdf






                      share|cite|improve this answer











                      $endgroup$



                      Okay I burned a lot of reputation points (at least for me) on MathOverflow to gain clarity on how to give some intuition into this problem, so hopefully this answer will be at least be somewhat illuminating.



                      To gain a deeper understanding of what is going on, first we need to answer the question, "What is a number?"



                      There are a lot of ways to define numbers, but in general numbers are thought of as symbols that represent sets.



                      This is easy for things like the natural numbers. So 10 would correspond to the set with ten things -- like a bag of ten stones. Pretty straight forward.



                      The tricky part is that when we consider ten a subset of the real numbers, we actually redefine it. This is not emphasized even in higher mathematics classes, like real analysis; it just happens when we define the real numbers.



                      So what is 10 when constructed in the real numbers? Well, at least with the Dedekind cut version of the real numbers, all real numbers correspond to a set with an infinite amount of elements. This makes 10 under the hood look drastically different, although in practice it operates exactly the same.



                      So let's return to the question: Why is 10 the same as 9.99999? Because the real numbers have this completely surprising quality, where there is no next real number. So when you have two real numbers that are as close together as possible, they are the same. I can't think of any physical object that has this quality, but it's how the real numbers work (makes "real" seem ironic).



                      With integers (bag of stones version) this is not the same. When you have two integers as close to each other as possible they are still different, and they are distance one apart.



                      Put another way, 10 bag of stones are not the same as 9.9999999 but 10 the natural number, where natural numbers are a subset of the real numbers is.



                      The bottom line is that the real numbers have these tricky edge cases that are hard to understand intuitively. Don't worry, your intuition is not really failing you. :)



                      I didn't feel confident answering until I got this Terence Tao link:



                      (Wayback Machine) https://web.archive.org/web/20100725014132/http://www.google.com:80/buzz/114134834346472219368/RarPutThCJv/In-the-foundations-of-mathematics-the-standard



                      (PDF, page 12) https://terrytao.files.wordpress.com/2011/06/blog-book.pdf







                      share|cite|improve this answer














                      share|cite|improve this answer



                      share|cite|improve this answer








                      edited Dec 30 '17 at 20:20


























                      community wiki





                      4 revs, 2 users 91%
                      Jonathan Fischoff









                      • 4




                        $begingroup$
                        The link doesn't work.
                        $endgroup$
                        – user 170039
                        Jan 5 '16 at 15:08










                      • $begingroup$
                        I'm not sure that most people think of real numbers as representing sets. Indeed, probably few of those who didn't study mathematics or attend mathematics lectures have even heard of the Dedekind construction. The common visualization of real numbers is as points on a line.
                        $endgroup$
                        – celtschk
                        Aug 14 '16 at 9:03






                      • 2




                        $begingroup$
                        the link is dead
                        $endgroup$
                        – baxx
                        Sep 10 '16 at 20:43










                      • $begingroup$
                        As a smart person, I actually like this answer better than balpha and Elezar Laibovich's answer. It's good enough for me because I can figure out from it a fuller answer that explains how people were looking for a definition of a real number in ZF such that the set of all of them is a complete ordered field so they found one and proved it's a complete ordered field. math.stackexchange.com/questions/2437893/… actually says what a real natural number is. Those answers aren't bad either because I can prove that all real numbers have a decimal representation.
                        $endgroup$
                        – Timothy
                        May 17 '18 at 1:34














                      • 4




                        $begingroup$
                        The link doesn't work.
                        $endgroup$
                        – user 170039
                        Jan 5 '16 at 15:08










                      • $begingroup$
                        I'm not sure that most people think of real numbers as representing sets. Indeed, probably few of those who didn't study mathematics or attend mathematics lectures have even heard of the Dedekind construction. The common visualization of real numbers is as points on a line.
                        $endgroup$
                        – celtschk
                        Aug 14 '16 at 9:03






                      • 2




                        $begingroup$
                        the link is dead
                        $endgroup$
                        – baxx
                        Sep 10 '16 at 20:43










                      • $begingroup$
                        As a smart person, I actually like this answer better than balpha and Elezar Laibovich's answer. It's good enough for me because I can figure out from it a fuller answer that explains how people were looking for a definition of a real number in ZF such that the set of all of them is a complete ordered field so they found one and proved it's a complete ordered field. math.stackexchange.com/questions/2437893/… actually says what a real natural number is. Those answers aren't bad either because I can prove that all real numbers have a decimal representation.
                        $endgroup$
                        – Timothy
                        May 17 '18 at 1:34








                      4




                      4




                      $begingroup$
                      The link doesn't work.
                      $endgroup$
                      – user 170039
                      Jan 5 '16 at 15:08




                      $begingroup$
                      The link doesn't work.
                      $endgroup$
                      – user 170039
                      Jan 5 '16 at 15:08












                      $begingroup$
                      I'm not sure that most people think of real numbers as representing sets. Indeed, probably few of those who didn't study mathematics or attend mathematics lectures have even heard of the Dedekind construction. The common visualization of real numbers is as points on a line.
                      $endgroup$
                      – celtschk
                      Aug 14 '16 at 9:03




                      $begingroup$
                      I'm not sure that most people think of real numbers as representing sets. Indeed, probably few of those who didn't study mathematics or attend mathematics lectures have even heard of the Dedekind construction. The common visualization of real numbers is as points on a line.
                      $endgroup$
                      – celtschk
                      Aug 14 '16 at 9:03




                      2




                      2




                      $begingroup$
                      the link is dead
                      $endgroup$
                      – baxx
                      Sep 10 '16 at 20:43




                      $begingroup$
                      the link is dead
                      $endgroup$
                      – baxx
                      Sep 10 '16 at 20:43












                      $begingroup$
                      As a smart person, I actually like this answer better than balpha and Elezar Laibovich's answer. It's good enough for me because I can figure out from it a fuller answer that explains how people were looking for a definition of a real number in ZF such that the set of all of them is a complete ordered field so they found one and proved it's a complete ordered field. math.stackexchange.com/questions/2437893/… actually says what a real natural number is. Those answers aren't bad either because I can prove that all real numbers have a decimal representation.
                      $endgroup$
                      – Timothy
                      May 17 '18 at 1:34




                      $begingroup$
                      As a smart person, I actually like this answer better than balpha and Elezar Laibovich's answer. It's good enough for me because I can figure out from it a fuller answer that explains how people were looking for a definition of a real number in ZF such that the set of all of them is a complete ordered field so they found one and proved it's a complete ordered field. math.stackexchange.com/questions/2437893/… actually says what a real natural number is. Those answers aren't bad either because I can prove that all real numbers have a decimal representation.
                      $endgroup$
                      – Timothy
                      May 17 '18 at 1:34











                      44












                      $begingroup$

                      There are genuine conceptual difficulties implicit in this question. The transition from the rational numbers to the real numbers is a difficult one, and it took a long time and a lot of thought to make it truly rigorous. It has been pointed out in other answers that the notation $0.999999ldots$ is just a shorthand notation for the infinite geometric series $sumlimits_{n=1}^{infty} 9left( frac{1}{10} right)^{n},$ which has sum $1.$ This is factually correct, but still sweeps some of the conceptual questions under the carpet. There are questions to be addressed about what we mean when we write down (or pretend to) an infinite decimal, or an infinite series. Either of those devices is just a shorthand notation which mathematicians agree will represent some numbers, given a set of ground rules. Let me try to present an argument to suggest that if the notation $0.99999ldots$ is to meaningfully represent any real number, then that number could be nothing other than the real number $1$, if we can agree that some truths are "self-evident".



                      Surely we can agree that the real number it represents can't be strictly greater than $1$, if it does indeed represent a real number. Let's now convince ourselves that it can't be a real number strictly less than $1,$
                      if it makes any sense at all. Well, if it was a real number $r < 1,$ that real number would be greater than or equal to $sumlimits_{n=1}^{k} 9left( frac{1}{10} right)^{n}$ for any finite integer $k.$ This last number is the decimal $0.99 ldots 9 $ which terminates after $k$ occurrences of $9,$ and differs from $1$ by $frac{1}{10^{k}}.$ Since $0 < r <1,$ there is a value of $k$ such that $frac{1}{10^{k}} < 1-r,$ so $1 - frac{1}{10^{k}} >r.$ Hence $sumlimits_{n=1}^{k} 9left( frac{1}{10} right)^{n} > r.$ But this can't be, because we agreed that $r$ should be greater than or equal to each of those truncated sums.



                      Have I proved that the recurring decimal is equal to $1$? Not really- what I have proved is that if we allow that recurring decimal to meaningfully represent any real number, that real number has to be $1,$ since it can't be strictly less than $1$ and can't be strictly greater than $1$. At this point, it becomes a matter of convention to agree that the real number $1$ can be represented in that form, and that convention will be consistent with our usual operations with real numbers and ordering of the real numbers, and equating the expression with any other real number would not maintain that consistency.






                      share|cite|improve this answer











                      $endgroup$









                      • 6




                        $begingroup$
                        This is the best answer here; it's a shame it was posted four years later so it's gotten so little attention.
                        $endgroup$
                        – Eric Wofsey
                        Oct 4 '15 at 0:44






                      • 1




                        $begingroup$
                        If I am not mistaken then this answer also answers the question of this post.
                        $endgroup$
                        – user 170039
                        Jan 6 '16 at 14:32










                      • $begingroup$
                        By the way, I was thinking that maybe you could write an answer for the linked post as well (or else, can you just tell me how can I give a link to this answer in a comment below my post?).
                        $endgroup$
                        – user 170039
                        Jan 6 '16 at 14:40






                      • 1




                        $begingroup$
                        I'm not sure how to set up links on here ( maybe the "cite") button. I am basically agreeing with what you say in your post.
                        $endgroup$
                        – Geoff Robinson
                        Jan 6 '16 at 15:03










                      • $begingroup$
                        I think that the "evident" issue of a number being an abstract concept existing in isolation from any positional representation should also be explicitly stated. One then proceeds to give rules as to how the positional representation works (as you did), and from these rules it follows that all real numbers have an infinitely long representation in any base, and rational numbers have a finitely long representation in some bases. The integers are a special case of rational numbers that have a finite representation in all bases.
                        $endgroup$
                        – Kuba Ober
                        Jul 28 '16 at 19:55


















                      44












                      $begingroup$

                      There are genuine conceptual difficulties implicit in this question. The transition from the rational numbers to the real numbers is a difficult one, and it took a long time and a lot of thought to make it truly rigorous. It has been pointed out in other answers that the notation $0.999999ldots$ is just a shorthand notation for the infinite geometric series $sumlimits_{n=1}^{infty} 9left( frac{1}{10} right)^{n},$ which has sum $1.$ This is factually correct, but still sweeps some of the conceptual questions under the carpet. There are questions to be addressed about what we mean when we write down (or pretend to) an infinite decimal, or an infinite series. Either of those devices is just a shorthand notation which mathematicians agree will represent some numbers, given a set of ground rules. Let me try to present an argument to suggest that if the notation $0.99999ldots$ is to meaningfully represent any real number, then that number could be nothing other than the real number $1$, if we can agree that some truths are "self-evident".



                      Surely we can agree that the real number it represents can't be strictly greater than $1$, if it does indeed represent a real number. Let's now convince ourselves that it can't be a real number strictly less than $1,$
                      if it makes any sense at all. Well, if it was a real number $r < 1,$ that real number would be greater than or equal to $sumlimits_{n=1}^{k} 9left( frac{1}{10} right)^{n}$ for any finite integer $k.$ This last number is the decimal $0.99 ldots 9 $ which terminates after $k$ occurrences of $9,$ and differs from $1$ by $frac{1}{10^{k}}.$ Since $0 < r <1,$ there is a value of $k$ such that $frac{1}{10^{k}} < 1-r,$ so $1 - frac{1}{10^{k}} >r.$ Hence $sumlimits_{n=1}^{k} 9left( frac{1}{10} right)^{n} > r.$ But this can't be, because we agreed that $r$ should be greater than or equal to each of those truncated sums.



                      Have I proved that the recurring decimal is equal to $1$? Not really- what I have proved is that if we allow that recurring decimal to meaningfully represent any real number, that real number has to be $1,$ since it can't be strictly less than $1$ and can't be strictly greater than $1$. At this point, it becomes a matter of convention to agree that the real number $1$ can be represented in that form, and that convention will be consistent with our usual operations with real numbers and ordering of the real numbers, and equating the expression with any other real number would not maintain that consistency.






                      share|cite|improve this answer











                      $endgroup$









                      • 6




                        $begingroup$
                        This is the best answer here; it's a shame it was posted four years later so it's gotten so little attention.
                        $endgroup$
                        – Eric Wofsey
                        Oct 4 '15 at 0:44






                      • 1




                        $begingroup$
                        If I am not mistaken then this answer also answers the question of this post.
                        $endgroup$
                        – user 170039
                        Jan 6 '16 at 14:32










                      • $begingroup$
                        By the way, I was thinking that maybe you could write an answer for the linked post as well (or else, can you just tell me how can I give a link to this answer in a comment below my post?).
                        $endgroup$
                        – user 170039
                        Jan 6 '16 at 14:40






                      • 1




                        $begingroup$
                        I'm not sure how to set up links on here ( maybe the "cite") button. I am basically agreeing with what you say in your post.
                        $endgroup$
                        – Geoff Robinson
                        Jan 6 '16 at 15:03










                      • $begingroup$
                        I think that the "evident" issue of a number being an abstract concept existing in isolation from any positional representation should also be explicitly stated. One then proceeds to give rules as to how the positional representation works (as you did), and from these rules it follows that all real numbers have an infinitely long representation in any base, and rational numbers have a finitely long representation in some bases. The integers are a special case of rational numbers that have a finite representation in all bases.
                        $endgroup$
                        – Kuba Ober
                        Jul 28 '16 at 19:55
















                      44












                      44








                      44





                      $begingroup$

                      There are genuine conceptual difficulties implicit in this question. The transition from the rational numbers to the real numbers is a difficult one, and it took a long time and a lot of thought to make it truly rigorous. It has been pointed out in other answers that the notation $0.999999ldots$ is just a shorthand notation for the infinite geometric series $sumlimits_{n=1}^{infty} 9left( frac{1}{10} right)^{n},$ which has sum $1.$ This is factually correct, but still sweeps some of the conceptual questions under the carpet. There are questions to be addressed about what we mean when we write down (or pretend to) an infinite decimal, or an infinite series. Either of those devices is just a shorthand notation which mathematicians agree will represent some numbers, given a set of ground rules. Let me try to present an argument to suggest that if the notation $0.99999ldots$ is to meaningfully represent any real number, then that number could be nothing other than the real number $1$, if we can agree that some truths are "self-evident".



                      Surely we can agree that the real number it represents can't be strictly greater than $1$, if it does indeed represent a real number. Let's now convince ourselves that it can't be a real number strictly less than $1,$
                      if it makes any sense at all. Well, if it was a real number $r < 1,$ that real number would be greater than or equal to $sumlimits_{n=1}^{k} 9left( frac{1}{10} right)^{n}$ for any finite integer $k.$ This last number is the decimal $0.99 ldots 9 $ which terminates after $k$ occurrences of $9,$ and differs from $1$ by $frac{1}{10^{k}}.$ Since $0 < r <1,$ there is a value of $k$ such that $frac{1}{10^{k}} < 1-r,$ so $1 - frac{1}{10^{k}} >r.$ Hence $sumlimits_{n=1}^{k} 9left( frac{1}{10} right)^{n} > r.$ But this can't be, because we agreed that $r$ should be greater than or equal to each of those truncated sums.



                      Have I proved that the recurring decimal is equal to $1$? Not really- what I have proved is that if we allow that recurring decimal to meaningfully represent any real number, that real number has to be $1,$ since it can't be strictly less than $1$ and can't be strictly greater than $1$. At this point, it becomes a matter of convention to agree that the real number $1$ can be represented in that form, and that convention will be consistent with our usual operations with real numbers and ordering of the real numbers, and equating the expression with any other real number would not maintain that consistency.






                      share|cite|improve this answer











                      $endgroup$



                      There are genuine conceptual difficulties implicit in this question. The transition from the rational numbers to the real numbers is a difficult one, and it took a long time and a lot of thought to make it truly rigorous. It has been pointed out in other answers that the notation $0.999999ldots$ is just a shorthand notation for the infinite geometric series $sumlimits_{n=1}^{infty} 9left( frac{1}{10} right)^{n},$ which has sum $1.$ This is factually correct, but still sweeps some of the conceptual questions under the carpet. There are questions to be addressed about what we mean when we write down (or pretend to) an infinite decimal, or an infinite series. Either of those devices is just a shorthand notation which mathematicians agree will represent some numbers, given a set of ground rules. Let me try to present an argument to suggest that if the notation $0.99999ldots$ is to meaningfully represent any real number, then that number could be nothing other than the real number $1$, if we can agree that some truths are "self-evident".



                      Surely we can agree that the real number it represents can't be strictly greater than $1$, if it does indeed represent a real number. Let's now convince ourselves that it can't be a real number strictly less than $1,$
                      if it makes any sense at all. Well, if it was a real number $r < 1,$ that real number would be greater than or equal to $sumlimits_{n=1}^{k} 9left( frac{1}{10} right)^{n}$ for any finite integer $k.$ This last number is the decimal $0.99 ldots 9 $ which terminates after $k$ occurrences of $9,$ and differs from $1$ by $frac{1}{10^{k}}.$ Since $0 < r <1,$ there is a value of $k$ such that $frac{1}{10^{k}} < 1-r,$ so $1 - frac{1}{10^{k}} >r.$ Hence $sumlimits_{n=1}^{k} 9left( frac{1}{10} right)^{n} > r.$ But this can't be, because we agreed that $r$ should be greater than or equal to each of those truncated sums.



                      Have I proved that the recurring decimal is equal to $1$? Not really- what I have proved is that if we allow that recurring decimal to meaningfully represent any real number, that real number has to be $1,$ since it can't be strictly less than $1$ and can't be strictly greater than $1$. At this point, it becomes a matter of convention to agree that the real number $1$ can be represented in that form, and that convention will be consistent with our usual operations with real numbers and ordering of the real numbers, and equating the expression with any other real number would not maintain that consistency.







                      share|cite|improve this answer














                      share|cite|improve this answer



                      share|cite|improve this answer








                      edited Mar 11 '18 at 15:29


























                      community wiki





                      6 revs, 3 users 50%
                      Did









                      • 6




                        $begingroup$
                        This is the best answer here; it's a shame it was posted four years later so it's gotten so little attention.
                        $endgroup$
                        – Eric Wofsey
                        Oct 4 '15 at 0:44






                      • 1




                        $begingroup$
                        If I am not mistaken then this answer also answers the question of this post.
                        $endgroup$
                        – user 170039
                        Jan 6 '16 at 14:32










                      • $begingroup$
                        By the way, I was thinking that maybe you could write an answer for the linked post as well (or else, can you just tell me how can I give a link to this answer in a comment below my post?).
                        $endgroup$
                        – user 170039
                        Jan 6 '16 at 14:40






                      • 1




                        $begingroup$
                        I'm not sure how to set up links on here ( maybe the "cite") button. I am basically agreeing with what you say in your post.
                        $endgroup$
                        – Geoff Robinson
                        Jan 6 '16 at 15:03










                      • $begingroup$
                        I think that the "evident" issue of a number being an abstract concept existing in isolation from any positional representation should also be explicitly stated. One then proceeds to give rules as to how the positional representation works (as you did), and from these rules it follows that all real numbers have an infinitely long representation in any base, and rational numbers have a finitely long representation in some bases. The integers are a special case of rational numbers that have a finite representation in all bases.
                        $endgroup$
                        – Kuba Ober
                        Jul 28 '16 at 19:55
















                      • 6




                        $begingroup$
                        This is the best answer here; it's a shame it was posted four years later so it's gotten so little attention.
                        $endgroup$
                        – Eric Wofsey
                        Oct 4 '15 at 0:44






                      • 1




                        $begingroup$
                        If I am not mistaken then this answer also answers the question of this post.
                        $endgroup$
                        – user 170039
                        Jan 6 '16 at 14:32










                      • $begingroup$
                        By the way, I was thinking that maybe you could write an answer for the linked post as well (or else, can you just tell me how can I give a link to this answer in a comment below my post?).
                        $endgroup$
                        – user 170039
                        Jan 6 '16 at 14:40






                      • 1




                        $begingroup$
                        I'm not sure how to set up links on here ( maybe the "cite") button. I am basically agreeing with what you say in your post.
                        $endgroup$
                        – Geoff Robinson
                        Jan 6 '16 at 15:03










                      • $begingroup$
                        I think that the "evident" issue of a number being an abstract concept existing in isolation from any positional representation should also be explicitly stated. One then proceeds to give rules as to how the positional representation works (as you did), and from these rules it follows that all real numbers have an infinitely long representation in any base, and rational numbers have a finitely long representation in some bases. The integers are a special case of rational numbers that have a finite representation in all bases.
                        $endgroup$
                        – Kuba Ober
                        Jul 28 '16 at 19:55










                      6




                      6




                      $begingroup$
                      This is the best answer here; it's a shame it was posted four years later so it's gotten so little attention.
                      $endgroup$
                      – Eric Wofsey
                      Oct 4 '15 at 0:44




                      $begingroup$
                      This is the best answer here; it's a shame it was posted four years later so it's gotten so little attention.
                      $endgroup$
                      – Eric Wofsey
                      Oct 4 '15 at 0:44




                      1




                      1




                      $begingroup$
                      If I am not mistaken then this answer also answers the question of this post.
                      $endgroup$
                      – user 170039
                      Jan 6 '16 at 14:32




                      $begingroup$
                      If I am not mistaken then this answer also answers the question of this post.
                      $endgroup$
                      – user 170039
                      Jan 6 '16 at 14:32












                      $begingroup$
                      By the way, I was thinking that maybe you could write an answer for the linked post as well (or else, can you just tell me how can I give a link to this answer in a comment below my post?).
                      $endgroup$
                      – user 170039
                      Jan 6 '16 at 14:40




                      $begingroup$
                      By the way, I was thinking that maybe you could write an answer for the linked post as well (or else, can you just tell me how can I give a link to this answer in a comment below my post?).
                      $endgroup$
                      – user 170039
                      Jan 6 '16 at 14:40




                      1




                      1




                      $begingroup$
                      I'm not sure how to set up links on here ( maybe the "cite") button. I am basically agreeing with what you say in your post.
                      $endgroup$
                      – Geoff Robinson
                      Jan 6 '16 at 15:03




                      $begingroup$
                      I'm not sure how to set up links on here ( maybe the "cite") button. I am basically agreeing with what you say in your post.
                      $endgroup$
                      – Geoff Robinson
                      Jan 6 '16 at 15:03












                      $begingroup$
                      I think that the "evident" issue of a number being an abstract concept existing in isolation from any positional representation should also be explicitly stated. One then proceeds to give rules as to how the positional representation works (as you did), and from these rules it follows that all real numbers have an infinitely long representation in any base, and rational numbers have a finitely long representation in some bases. The integers are a special case of rational numbers that have a finite representation in all bases.
                      $endgroup$
                      – Kuba Ober
                      Jul 28 '16 at 19:55






                      $begingroup$
                      I think that the "evident" issue of a number being an abstract concept existing in isolation from any positional representation should also be explicitly stated. One then proceeds to give rules as to how the positional representation works (as you did), and from these rules it follows that all real numbers have an infinitely long representation in any base, and rational numbers have a finitely long representation in some bases. The integers are a special case of rational numbers that have a finite representation in all bases.
                      $endgroup$
                      – Kuba Ober
                      Jul 28 '16 at 19:55













                      39












                      $begingroup$

                      One argument against this is that 0.99999999... is "somewhat" less than 1. How much exactly?



                            1 - 0.999999... = ε              (0)


                      If the above is true, the following also must be true:



                      9 × (1 - 0.999999...) = ε × 9


                      Let's calculate:



                      0.999... ×
                      9 =
                      ───────────
                      8.1
                      81
                      81
                      .
                      .
                      .

                      ───────────
                      8.999...


                      Thus:



                           9 - 8.999999... = 9ε              (1)


                      But:



                               8.999999... = 8 + 0.99999...  (2)


                      Indeed:



                      8.00000000... +
                      0.99999999... =
                      ────────────────
                      8.99999999...


                      Now let's see what we can deduce from (0), (1) and (2).



                      9 - 8.999999... = 9ε                      because of (2)
                      9 - 8.999999... = 9 - (8 + 0.99999...) = because of (1)
                      = 9 - 8 - (1 - ε) because of (0)
                      = 1 - 1 + ε
                      = ε.


                      Thus:



                      9ε = ε

                      8ε = 0

                      ε = 0

                      1 - 0.999999... = ε = 0


                      Quod erat demonstrandum. Pardon my unicode.






                      share|cite|improve this answer











                      $endgroup$













                      • $begingroup$
                        I did my best to avoid 0.00000..., but this made the calculations not as strikingly simple as I'd have liked to.
                        $endgroup$
                        – badp
                        Jul 20 '10 at 21:10










                      • $begingroup$
                        Why was this voted down? It seems reasonable to this amateur math enjoyer.
                        $endgroup$
                        – ErikE
                        Sep 25 '10 at 7:07










                      • $begingroup$
                        @Emtucifor I guess this sounds like "nonsense" to people that disagree on the basic premise of 0.999... = 1 :)
                        $endgroup$
                        – badp
                        Sep 25 '10 at 7:21










                      • $begingroup$
                        8ε = 0 instead of 10ε = 0
                        $endgroup$
                        – user59671
                        May 9 '13 at 9:37






                      • 2




                        $begingroup$
                        @SerjSagan nope, the RHS of (1) follows from (0). The fact that $9ε = ε$ is precisely the point I'm making.
                        $endgroup$
                        – badp
                        Aug 2 '16 at 20:00
















                      39












                      $begingroup$

                      One argument against this is that 0.99999999... is "somewhat" less than 1. How much exactly?



                            1 - 0.999999... = ε              (0)


                      If the above is true, the following also must be true:



                      9 × (1 - 0.999999...) = ε × 9


                      Let's calculate:



                      0.999... ×
                      9 =
                      ───────────
                      8.1
                      81
                      81
                      .
                      .
                      .

                      ───────────
                      8.999...


                      Thus:



                           9 - 8.999999... = 9ε              (1)


                      But:



                               8.999999... = 8 + 0.99999...  (2)


                      Indeed:



                      8.00000000... +
                      0.99999999... =
                      ────────────────
                      8.99999999...


                      Now let's see what we can deduce from (0), (1) and (2).



                      9 - 8.999999... = 9ε                      because of (2)
                      9 - 8.999999... = 9 - (8 + 0.99999...) = because of (1)
                      = 9 - 8 - (1 - ε) because of (0)
                      = 1 - 1 + ε
                      = ε.


                      Thus:



                      9ε = ε

                      8ε = 0

                      ε = 0

                      1 - 0.999999... = ε = 0


                      Quod erat demonstrandum. Pardon my unicode.






                      share|cite|improve this answer











                      $endgroup$













                      • $begingroup$
                        I did my best to avoid 0.00000..., but this made the calculations not as strikingly simple as I'd have liked to.
                        $endgroup$
                        – badp
                        Jul 20 '10 at 21:10










                      • $begingroup$
                        Why was this voted down? It seems reasonable to this amateur math enjoyer.
                        $endgroup$
                        – ErikE
                        Sep 25 '10 at 7:07










                      • $begingroup$
                        @Emtucifor I guess this sounds like "nonsense" to people that disagree on the basic premise of 0.999... = 1 :)
                        $endgroup$
                        – badp
                        Sep 25 '10 at 7:21










                      • $begingroup$
                        8ε = 0 instead of 10ε = 0
                        $endgroup$
                        – user59671
                        May 9 '13 at 9:37






                      • 2




                        $begingroup$
                        @SerjSagan nope, the RHS of (1) follows from (0). The fact that $9ε = ε$ is precisely the point I'm making.
                        $endgroup$
                        – badp
                        Aug 2 '16 at 20:00














                      39












                      39








                      39





                      $begingroup$

                      One argument against this is that 0.99999999... is "somewhat" less than 1. How much exactly?



                            1 - 0.999999... = ε              (0)


                      If the above is true, the following also must be true:



                      9 × (1 - 0.999999...) = ε × 9


                      Let's calculate:



                      0.999... ×
                      9 =
                      ───────────
                      8.1
                      81
                      81
                      .
                      .
                      .

                      ───────────
                      8.999...


                      Thus:



                           9 - 8.999999... = 9ε              (1)


                      But:



                               8.999999... = 8 + 0.99999...  (2)


                      Indeed:



                      8.00000000... +
                      0.99999999... =
                      ────────────────
                      8.99999999...


                      Now let's see what we can deduce from (0), (1) and (2).



                      9 - 8.999999... = 9ε                      because of (2)
                      9 - 8.999999... = 9 - (8 + 0.99999...) = because of (1)
                      = 9 - 8 - (1 - ε) because of (0)
                      = 1 - 1 + ε
                      = ε.


                      Thus:



                      9ε = ε

                      8ε = 0

                      ε = 0

                      1 - 0.999999... = ε = 0


                      Quod erat demonstrandum. Pardon my unicode.






                      share|cite|improve this answer











                      $endgroup$



                      One argument against this is that 0.99999999... is "somewhat" less than 1. How much exactly?



                            1 - 0.999999... = ε              (0)


                      If the above is true, the following also must be true:



                      9 × (1 - 0.999999...) = ε × 9


                      Let's calculate:



                      0.999... ×
                      9 =
                      ───────────
                      8.1
                      81
                      81
                      .
                      .
                      .

                      ───────────
                      8.999...


                      Thus:



                           9 - 8.999999... = 9ε              (1)


                      But:



                               8.999999... = 8 + 0.99999...  (2)


                      Indeed:



                      8.00000000... +
                      0.99999999... =
                      ────────────────
                      8.99999999...


                      Now let's see what we can deduce from (0), (1) and (2).



                      9 - 8.999999... = 9ε                      because of (2)
                      9 - 8.999999... = 9 - (8 + 0.99999...) = because of (1)
                      = 9 - 8 - (1 - ε) because of (0)
                      = 1 - 1 + ε
                      = ε.


                      Thus:



                      9ε = ε

                      8ε = 0

                      ε = 0

                      1 - 0.999999... = ε = 0


                      Quod erat demonstrandum. Pardon my unicode.







                      share|cite|improve this answer














                      share|cite|improve this answer



                      share|cite|improve this answer








                      edited Apr 22 '14 at 9:38


























                      community wiki





                      7 revs, 2 users 99%
                      badp













                      • $begingroup$
                        I did my best to avoid 0.00000..., but this made the calculations not as strikingly simple as I'd have liked to.
                        $endgroup$
                        – badp
                        Jul 20 '10 at 21:10










                      • $begingroup$
                        Why was this voted down? It seems reasonable to this amateur math enjoyer.
                        $endgroup$
                        – ErikE
                        Sep 25 '10 at 7:07










                      • $begingroup$
                        @Emtucifor I guess this sounds like "nonsense" to people that disagree on the basic premise of 0.999... = 1 :)
                        $endgroup$
                        – badp
                        Sep 25 '10 at 7:21










                      • $begingroup$
                        8ε = 0 instead of 10ε = 0
                        $endgroup$
                        – user59671
                        May 9 '13 at 9:37






                      • 2




                        $begingroup$
                        @SerjSagan nope, the RHS of (1) follows from (0). The fact that $9ε = ε$ is precisely the point I'm making.
                        $endgroup$
                        – badp
                        Aug 2 '16 at 20:00


















                      • $begingroup$
                        I did my best to avoid 0.00000..., but this made the calculations not as strikingly simple as I'd have liked to.
                        $endgroup$
                        – badp
                        Jul 20 '10 at 21:10










                      • $begingroup$
                        Why was this voted down? It seems reasonable to this amateur math enjoyer.
                        $endgroup$
                        – ErikE
                        Sep 25 '10 at 7:07










                      • $begingroup$
                        @Emtucifor I guess this sounds like "nonsense" to people that disagree on the basic premise of 0.999... = 1 :)
                        $endgroup$
                        – badp
                        Sep 25 '10 at 7:21










                      • $begingroup$
                        8ε = 0 instead of 10ε = 0
                        $endgroup$
                        – user59671
                        May 9 '13 at 9:37






                      • 2




                        $begingroup$
                        @SerjSagan nope, the RHS of (1) follows from (0). The fact that $9ε = ε$ is precisely the point I'm making.
                        $endgroup$
                        – badp
                        Aug 2 '16 at 20:00
















                      $begingroup$
                      I did my best to avoid 0.00000..., but this made the calculations not as strikingly simple as I'd have liked to.
                      $endgroup$
                      – badp
                      Jul 20 '10 at 21:10




                      $begingroup$
                      I did my best to avoid 0.00000..., but this made the calculations not as strikingly simple as I'd have liked to.
                      $endgroup$
                      – badp
                      Jul 20 '10 at 21:10












                      $begingroup$
                      Why was this voted down? It seems reasonable to this amateur math enjoyer.
                      $endgroup$
                      – ErikE
                      Sep 25 '10 at 7:07




                      $begingroup$
                      Why was this voted down? It seems reasonable to this amateur math enjoyer.
                      $endgroup$
                      – ErikE
                      Sep 25 '10 at 7:07












                      $begingroup$
                      @Emtucifor I guess this sounds like "nonsense" to people that disagree on the basic premise of 0.999... = 1 :)
                      $endgroup$
                      – badp
                      Sep 25 '10 at 7:21




                      $begingroup$
                      @Emtucifor I guess this sounds like "nonsense" to people that disagree on the basic premise of 0.999... = 1 :)
                      $endgroup$
                      – badp
                      Sep 25 '10 at 7:21












                      $begingroup$
                      8ε = 0 instead of 10ε = 0
                      $endgroup$
                      – user59671
                      May 9 '13 at 9:37




                      $begingroup$
                      8ε = 0 instead of 10ε = 0
                      $endgroup$
                      – user59671
                      May 9 '13 at 9:37




                      2




                      2




                      $begingroup$
                      @SerjSagan nope, the RHS of (1) follows from (0). The fact that $9ε = ε$ is precisely the point I'm making.
                      $endgroup$
                      – badp
                      Aug 2 '16 at 20:00




                      $begingroup$
                      @SerjSagan nope, the RHS of (1) follows from (0). The fact that $9ε = ε$ is precisely the point I'm making.
                      $endgroup$
                      – badp
                      Aug 2 '16 at 20:00











                      23












                      $begingroup$

                      .999... = 1 because .999... is a concise symbolic representation of "the limit of some variable as it approaches one." Therefore, .999... = 1 for the same reason the limit of x as x approaches 1 equals 1.






                      share|cite|improve this answer











                      $endgroup$


















                        23












                        $begingroup$

                        .999... = 1 because .999... is a concise symbolic representation of "the limit of some variable as it approaches one." Therefore, .999... = 1 for the same reason the limit of x as x approaches 1 equals 1.






                        share|cite|improve this answer











                        $endgroup$
















                          23












                          23








                          23





                          $begingroup$

                          .999... = 1 because .999... is a concise symbolic representation of "the limit of some variable as it approaches one." Therefore, .999... = 1 for the same reason the limit of x as x approaches 1 equals 1.






                          share|cite|improve this answer











                          $endgroup$



                          .999... = 1 because .999... is a concise symbolic representation of "the limit of some variable as it approaches one." Therefore, .999... = 1 for the same reason the limit of x as x approaches 1 equals 1.







                          share|cite|improve this answer














                          share|cite|improve this answer



                          share|cite|improve this answer








                          edited Jul 23 '10 at 2:13


























                          community wiki





                          2 revs
                          Ami
























                              18












                              $begingroup$

                              If you take two real numbers x and y then there per definition of the real number z for which x < z < y or x > z > y is true.



                              For x = 0.99999... and y = 1 you can't find a z and therefore 0.99999... = 1.






                              share|cite|improve this answer











                              $endgroup$


















                                18












                                $begingroup$

                                If you take two real numbers x and y then there per definition of the real number z for which x < z < y or x > z > y is true.



                                For x = 0.99999... and y = 1 you can't find a z and therefore 0.99999... = 1.






                                share|cite|improve this answer











                                $endgroup$
















                                  18












                                  18








                                  18





                                  $begingroup$

                                  If you take two real numbers x and y then there per definition of the real number z for which x < z < y or x > z > y is true.



                                  For x = 0.99999... and y = 1 you can't find a z and therefore 0.99999... = 1.






                                  share|cite|improve this answer











                                  $endgroup$



                                  If you take two real numbers x and y then there per definition of the real number z for which x < z < y or x > z > y is true.



                                  For x = 0.99999... and y = 1 you can't find a z and therefore 0.99999... = 1.







                                  share|cite|improve this answer














                                  share|cite|improve this answer



                                  share|cite|improve this answer








                                  edited Jul 20 '10 at 23:50


























                                  community wiki





                                  2 revs
                                  Christian
























                                      15












                                      $begingroup$

                                      You can visualise it by thinking about it in infinitesimals. The more $9's$ you have on the end of $0.999$, the closer you get to $1$. When you add an infinite number of $9's$ to the decimal expansion, you are infinitely close to $1$ (or an infinitesimal distance away).



                                      And this isn't a rigorous proof, just an aid to visualisation of the result.






                                      share|cite|improve this answer











                                      $endgroup$













                                      • $begingroup$
                                        I think this answer is not needed. It's similar to another answer but has wrong information. Infinitesimal numbers exist only in the hyperreal number system and in the hyperreal system, numbers infinitesimally close are not equal. The real numbers are a subset of the hyperreal numbers so the community decided theat the notation represents 1 and not a number infinitesimally close because then it wouldn't represent a real number. Maybe the real numbers can be defined as equivalence classes infinitesimally close hyperreal numbers and from that, redefine the notation to represent the number 1.
                                        $endgroup$
                                        – Timothy
                                        May 22 '18 at 19:19
















                                      15












                                      $begingroup$

                                      You can visualise it by thinking about it in infinitesimals. The more $9's$ you have on the end of $0.999$, the closer you get to $1$. When you add an infinite number of $9's$ to the decimal expansion, you are infinitely close to $1$ (or an infinitesimal distance away).



                                      And this isn't a rigorous proof, just an aid to visualisation of the result.






                                      share|cite|improve this answer











                                      $endgroup$













                                      • $begingroup$
                                        I think this answer is not needed. It's similar to another answer but has wrong information. Infinitesimal numbers exist only in the hyperreal number system and in the hyperreal system, numbers infinitesimally close are not equal. The real numbers are a subset of the hyperreal numbers so the community decided theat the notation represents 1 and not a number infinitesimally close because then it wouldn't represent a real number. Maybe the real numbers can be defined as equivalence classes infinitesimally close hyperreal numbers and from that, redefine the notation to represent the number 1.
                                        $endgroup$
                                        – Timothy
                                        May 22 '18 at 19:19














                                      15












                                      15








                                      15





                                      $begingroup$

                                      You can visualise it by thinking about it in infinitesimals. The more $9's$ you have on the end of $0.999$, the closer you get to $1$. When you add an infinite number of $9's$ to the decimal expansion, you are infinitely close to $1$ (or an infinitesimal distance away).



                                      And this isn't a rigorous proof, just an aid to visualisation of the result.






                                      share|cite|improve this answer











                                      $endgroup$



                                      You can visualise it by thinking about it in infinitesimals. The more $9's$ you have on the end of $0.999$, the closer you get to $1$. When you add an infinite number of $9's$ to the decimal expansion, you are infinitely close to $1$ (or an infinitesimal distance away).



                                      And this isn't a rigorous proof, just an aid to visualisation of the result.







                                      share|cite|improve this answer














                                      share|cite|improve this answer



                                      share|cite|improve this answer








                                      edited May 6 '16 at 18:24


























                                      community wiki





                                      2 revs, 2 users 80%
                                      workmad3













                                      • $begingroup$
                                        I think this answer is not needed. It's similar to another answer but has wrong information. Infinitesimal numbers exist only in the hyperreal number system and in the hyperreal system, numbers infinitesimally close are not equal. The real numbers are a subset of the hyperreal numbers so the community decided theat the notation represents 1 and not a number infinitesimally close because then it wouldn't represent a real number. Maybe the real numbers can be defined as equivalence classes infinitesimally close hyperreal numbers and from that, redefine the notation to represent the number 1.
                                        $endgroup$
                                        – Timothy
                                        May 22 '18 at 19:19


















                                      • $begingroup$
                                        I think this answer is not needed. It's similar to another answer but has wrong information. Infinitesimal numbers exist only in the hyperreal number system and in the hyperreal system, numbers infinitesimally close are not equal. The real numbers are a subset of the hyperreal numbers so the community decided theat the notation represents 1 and not a number infinitesimally close because then it wouldn't represent a real number. Maybe the real numbers can be defined as equivalence classes infinitesimally close hyperreal numbers and from that, redefine the notation to represent the number 1.
                                        $endgroup$
                                        – Timothy
                                        May 22 '18 at 19:19
















                                      $begingroup$
                                      I think this answer is not needed. It's similar to another answer but has wrong information. Infinitesimal numbers exist only in the hyperreal number system and in the hyperreal system, numbers infinitesimally close are not equal. The real numbers are a subset of the hyperreal numbers so the community decided theat the notation represents 1 and not a number infinitesimally close because then it wouldn't represent a real number. Maybe the real numbers can be defined as equivalence classes infinitesimally close hyperreal numbers and from that, redefine the notation to represent the number 1.
                                      $endgroup$
                                      – Timothy
                                      May 22 '18 at 19:19




                                      $begingroup$
                                      I think this answer is not needed. It's similar to another answer but has wrong information. Infinitesimal numbers exist only in the hyperreal number system and in the hyperreal system, numbers infinitesimally close are not equal. The real numbers are a subset of the hyperreal numbers so the community decided theat the notation represents 1 and not a number infinitesimally close because then it wouldn't represent a real number. Maybe the real numbers can be defined as equivalence classes infinitesimally close hyperreal numbers and from that, redefine the notation to represent the number 1.
                                      $endgroup$
                                      – Timothy
                                      May 22 '18 at 19:19











                                      13












                                      $begingroup$

                                      Indeed this is true. The underlying reason is that decimal numbers are not unique representations of the reals. (Technically, there does not exist a bijection between the set of all decimal numbers and the reals.)



                                      Here's a very simple proof:



                                      $$begin{align}
                                      frac13&=0.333ldots&hbox{(by long division)}\
                                      implies0.333ldotstimes3&=0.999ldots&hbox{(multiplying each digit by $3$)}
                                      end{align}$$



                                      Then we already know $0.333ldotstimes3=1$ therefore $0.999ldots=1$.






                                      share|cite|improve this answer











                                      $endgroup$









                                      • 22




                                        $begingroup$
                                        -1. This is not a proof at all! Why is 1/3=0.333...? Seriously folks, for the private beta, let's try to maintain a little correctiness.
                                        $endgroup$
                                        – Scott Morrison
                                        Jul 20 '10 at 20:03






                                      • 5




                                        $begingroup$
                                        @Scott: Sure it is. You can prove it easily by long division. This is about algorithms for mathematical methods really.
                                        $endgroup$
                                        – Noldorin
                                        Jul 20 '10 at 20:12






                                      • 6




                                        $begingroup$
                                        @Scott: Might help to stop whining and post what you think is the 'correct' answer then.
                                        $endgroup$
                                        – Noldorin
                                        Jul 20 '10 at 20:17






                                      • 11




                                        $begingroup$
                                        Just to nitpick, there is a bijection between the set of decimal expansions and reals because they are sets with the same cardinality. It's just that the natural map taking expansions to real numbers isn't injective.
                                        $endgroup$
                                        – Simon Nickerson
                                        Jul 20 '10 at 21:15






                                      • 3




                                        $begingroup$
                                        @Scott, I see it would not be obvious that 1/3=0.333..., but as by Noldorins comment regarding long division, what would be wrong with this as a proof, if the first line is annotated with 'by long division' ?
                                        $endgroup$
                                        – Sami
                                        Jul 21 '10 at 4:58
















                                      13












                                      $begingroup$

                                      Indeed this is true. The underlying reason is that decimal numbers are not unique representations of the reals. (Technically, there does not exist a bijection between the set of all decimal numbers and the reals.)



                                      Here's a very simple proof:



                                      $$begin{align}
                                      frac13&=0.333ldots&hbox{(by long division)}\
                                      implies0.333ldotstimes3&=0.999ldots&hbox{(multiplying each digit by $3$)}
                                      end{align}$$



                                      Then we already know $0.333ldotstimes3=1$ therefore $0.999ldots=1$.






                                      share|cite|improve this answer











                                      $endgroup$









                                      • 22




                                        $begingroup$
                                        -1. This is not a proof at all! Why is 1/3=0.333...? Seriously folks, for the private beta, let's try to maintain a little correctiness.
                                        $endgroup$
                                        – Scott Morrison
                                        Jul 20 '10 at 20:03






                                      • 5




                                        $begingroup$
                                        @Scott: Sure it is. You can prove it easily by long division. This is about algorithms for mathematical methods really.
                                        $endgroup$
                                        – Noldorin
                                        Jul 20 '10 at 20:12






                                      • 6




                                        $begingroup$
                                        @Scott: Might help to stop whining and post what you think is the 'correct' answer then.
                                        $endgroup$
                                        – Noldorin
                                        Jul 20 '10 at 20:17






                                      • 11




                                        $begingroup$
                                        Just to nitpick, there is a bijection between the set of decimal expansions and reals because they are sets with the same cardinality. It's just that the natural map taking expansions to real numbers isn't injective.
                                        $endgroup$
                                        – Simon Nickerson
                                        Jul 20 '10 at 21:15






                                      • 3




                                        $begingroup$
                                        @Scott, I see it would not be obvious that 1/3=0.333..., but as by Noldorins comment regarding long division, what would be wrong with this as a proof, if the first line is annotated with 'by long division' ?
                                        $endgroup$
                                        – Sami
                                        Jul 21 '10 at 4:58














                                      13












                                      13








                                      13





                                      $begingroup$

                                      Indeed this is true. The underlying reason is that decimal numbers are not unique representations of the reals. (Technically, there does not exist a bijection between the set of all decimal numbers and the reals.)



                                      Here's a very simple proof:



                                      $$begin{align}
                                      frac13&=0.333ldots&hbox{(by long division)}\
                                      implies0.333ldotstimes3&=0.999ldots&hbox{(multiplying each digit by $3$)}
                                      end{align}$$



                                      Then we already know $0.333ldotstimes3=1$ therefore $0.999ldots=1$.






                                      share|cite|improve this answer











                                      $endgroup$



                                      Indeed this is true. The underlying reason is that decimal numbers are not unique representations of the reals. (Technically, there does not exist a bijection between the set of all decimal numbers and the reals.)



                                      Here's a very simple proof:



                                      $$begin{align}
                                      frac13&=0.333ldots&hbox{(by long division)}\
                                      implies0.333ldotstimes3&=0.999ldots&hbox{(multiplying each digit by $3$)}
                                      end{align}$$



                                      Then we already know $0.333ldotstimes3=1$ therefore $0.999ldots=1$.







                                      share|cite|improve this answer














                                      share|cite|improve this answer



                                      share|cite|improve this answer








                                      edited Feb 24 '17 at 20:51


























                                      community wiki





                                      3 revs, 2 users 77%
                                      Noldorin









                                      • 22




                                        $begingroup$
                                        -1. This is not a proof at all! Why is 1/3=0.333...? Seriously folks, for the private beta, let's try to maintain a little correctiness.
                                        $endgroup$
                                        – Scott Morrison
                                        Jul 20 '10 at 20:03






                                      • 5




                                        $begingroup$
                                        @Scott: Sure it is. You can prove it easily by long division. This is about algorithms for mathematical methods really.
                                        $endgroup$
                                        – Noldorin
                                        Jul 20 '10 at 20:12






                                      • 6




                                        $begingroup$
                                        @Scott: Might help to stop whining and post what you think is the 'correct' answer then.
                                        $endgroup$
                                        – Noldorin
                                        Jul 20 '10 at 20:17






                                      • 11




                                        $begingroup$
                                        Just to nitpick, there is a bijection between the set of decimal expansions and reals because they are sets with the same cardinality. It's just that the natural map taking expansions to real numbers isn't injective.
                                        $endgroup$
                                        – Simon Nickerson
                                        Jul 20 '10 at 21:15






                                      • 3




                                        $begingroup$
                                        @Scott, I see it would not be obvious that 1/3=0.333..., but as by Noldorins comment regarding long division, what would be wrong with this as a proof, if the first line is annotated with 'by long division' ?
                                        $endgroup$
                                        – Sami
                                        Jul 21 '10 at 4:58














                                      • 22




                                        $begingroup$
                                        -1. This is not a proof at all! Why is 1/3=0.333...? Seriously folks, for the private beta, let's try to maintain a little correctiness.
                                        $endgroup$
                                        – Scott Morrison
                                        Jul 20 '10 at 20:03






                                      • 5




                                        $begingroup$
                                        @Scott: Sure it is. You can prove it easily by long division. This is about algorithms for mathematical methods really.
                                        $endgroup$
                                        – Noldorin
                                        Jul 20 '10 at 20:12






                                      • 6




                                        $begingroup$
                                        @Scott: Might help to stop whining and post what you think is the 'correct' answer then.
                                        $endgroup$
                                        – Noldorin
                                        Jul 20 '10 at 20:17






                                      • 11




                                        $begingroup$
                                        Just to nitpick, there is a bijection between the set of decimal expansions and reals because they are sets with the same cardinality. It's just that the natural map taking expansions to real numbers isn't injective.
                                        $endgroup$
                                        – Simon Nickerson
                                        Jul 20 '10 at 21:15






                                      • 3




                                        $begingroup$
                                        @Scott, I see it would not be obvious that 1/3=0.333..., but as by Noldorins comment regarding long division, what would be wrong with this as a proof, if the first line is annotated with 'by long division' ?
                                        $endgroup$
                                        – Sami
                                        Jul 21 '10 at 4:58








                                      22




                                      22




                                      $begingroup$
                                      -1. This is not a proof at all! Why is 1/3=0.333...? Seriously folks, for the private beta, let's try to maintain a little correctiness.
                                      $endgroup$
                                      – Scott Morrison
                                      Jul 20 '10 at 20:03




                                      $begingroup$
                                      -1. This is not a proof at all! Why is 1/3=0.333...? Seriously folks, for the private beta, let's try to maintain a little correctiness.
                                      $endgroup$
                                      – Scott Morrison
                                      Jul 20 '10 at 20:03




                                      5




                                      5




                                      $begingroup$
                                      @Scott: Sure it is. You can prove it easily by long division. This is about algorithms for mathematical methods really.
                                      $endgroup$
                                      – Noldorin
                                      Jul 20 '10 at 20:12




                                      $begingroup$
                                      @Scott: Sure it is. You can prove it easily by long division. This is about algorithms for mathematical methods really.
                                      $endgroup$
                                      – Noldorin
                                      Jul 20 '10 at 20:12




                                      6




                                      6




                                      $begingroup$
                                      @Scott: Might help to stop whining and post what you think is the 'correct' answer then.
                                      $endgroup$
                                      – Noldorin
                                      Jul 20 '10 at 20:17




                                      $begingroup$
                                      @Scott: Might help to stop whining and post what you think is the 'correct' answer then.
                                      $endgroup$
                                      – Noldorin
                                      Jul 20 '10 at 20:17




                                      11




                                      11




                                      $begingroup$
                                      Just to nitpick, there is a bijection between the set of decimal expansions and reals because they are sets with the same cardinality. It's just that the natural map taking expansions to real numbers isn't injective.
                                      $endgroup$
                                      – Simon Nickerson
                                      Jul 20 '10 at 21:15




                                      $begingroup$
                                      Just to nitpick, there is a bijection between the set of decimal expansions and reals because they are sets with the same cardinality. It's just that the natural map taking expansions to real numbers isn't injective.
                                      $endgroup$
                                      – Simon Nickerson
                                      Jul 20 '10 at 21:15




                                      3




                                      3




                                      $begingroup$
                                      @Scott, I see it would not be obvious that 1/3=0.333..., but as by Noldorins comment regarding long division, what would be wrong with this as a proof, if the first line is annotated with 'by long division' ?
                                      $endgroup$
                                      – Sami
                                      Jul 21 '10 at 4:58




                                      $begingroup$
                                      @Scott, I see it would not be obvious that 1/3=0.333..., but as by Noldorins comment regarding long division, what would be wrong with this as a proof, if the first line is annotated with 'by long division' ?
                                      $endgroup$
                                      – Sami
                                      Jul 21 '10 at 4:58











                                      12












                                      $begingroup$

                                      Given (by long division):

                                      $frac{1}{3} = 0.bar{3}$



                                      Multiply by 3:

                                      $3times left( frac{1}{3} right) = left( 0.bar{3} right) times 3$



                                      Therefore:

                                      $frac{3}{3} = 0.bar{9}$



                                      QED.






                                      share|cite|improve this answer











                                      $endgroup$













                                      • $begingroup$
                                        I think the long division precisely involves the proof of a limit like the sum mentioned above...
                                        $endgroup$
                                        – C-Star-Puppy
                                        Aug 27 '14 at 21:07






                                      • 2




                                        $begingroup$
                                        @FreeziiS, for this particular question the fact of a rigorous proof happens to be less important than the fact of a convincing proof. Plenty of people trust the long division algorithm because they learned it in school; they haven't seen a rigorous proof for its workability because they don't care; it obviously works. It is these people (who don't care about or trust rigorous proofs) who argue the most about $0.999999...$
                                        $endgroup$
                                        – Wildcard
                                        Dec 20 '16 at 0:16
















                                      12












                                      $begingroup$

                                      Given (by long division):

                                      $frac{1}{3} = 0.bar{3}$



                                      Multiply by 3:

                                      $3times left( frac{1}{3} right) = left( 0.bar{3} right) times 3$



                                      Therefore:

                                      $frac{3}{3} = 0.bar{9}$



                                      QED.






                                      share|cite|improve this answer











                                      $endgroup$













                                      • $begingroup$
                                        I think the long division precisely involves the proof of a limit like the sum mentioned above...
                                        $endgroup$
                                        – C-Star-Puppy
                                        Aug 27 '14 at 21:07






                                      • 2




                                        $begingroup$
                                        @FreeziiS, for this particular question the fact of a rigorous proof happens to be less important than the fact of a convincing proof. Plenty of people trust the long division algorithm because they learned it in school; they haven't seen a rigorous proof for its workability because they don't care; it obviously works. It is these people (who don't care about or trust rigorous proofs) who argue the most about $0.999999...$
                                        $endgroup$
                                        – Wildcard
                                        Dec 20 '16 at 0:16














                                      12












                                      12








                                      12





                                      $begingroup$

                                      Given (by long division):

                                      $frac{1}{3} = 0.bar{3}$



                                      Multiply by 3:

                                      $3times left( frac{1}{3} right) = left( 0.bar{3} right) times 3$



                                      Therefore:

                                      $frac{3}{3} = 0.bar{9}$



                                      QED.






                                      share|cite|improve this answer











                                      $endgroup$



                                      Given (by long division):

                                      $frac{1}{3} = 0.bar{3}$



                                      Multiply by 3:

                                      $3times left( frac{1}{3} right) = left( 0.bar{3} right) times 3$



                                      Therefore:

                                      $frac{3}{3} = 0.bar{9}$



                                      QED.







                                      share|cite|improve this answer














                                      share|cite|improve this answer



                                      share|cite|improve this answer








                                      edited Feb 21 '11 at 2:52


























                                      community wiki





                                      3 revs
                                      John Gietzen













                                      • $begingroup$
                                        I think the long division precisely involves the proof of a limit like the sum mentioned above...
                                        $endgroup$
                                        – C-Star-Puppy
                                        Aug 27 '14 at 21:07






                                      • 2




                                        $begingroup$
                                        @FreeziiS, for this particular question the fact of a rigorous proof happens to be less important than the fact of a convincing proof. Plenty of people trust the long division algorithm because they learned it in school; they haven't seen a rigorous proof for its workability because they don't care; it obviously works. It is these people (who don't care about or trust rigorous proofs) who argue the most about $0.999999...$
                                        $endgroup$
                                        – Wildcard
                                        Dec 20 '16 at 0:16


















                                      • $begingroup$
                                        I think the long division precisely involves the proof of a limit like the sum mentioned above...
                                        $endgroup$
                                        – C-Star-Puppy
                                        Aug 27 '14 at 21:07






                                      • 2




                                        $begingroup$
                                        @FreeziiS, for this particular question the fact of a rigorous proof happens to be less important than the fact of a convincing proof. Plenty of people trust the long division algorithm because they learned it in school; they haven't seen a rigorous proof for its workability because they don't care; it obviously works. It is these people (who don't care about or trust rigorous proofs) who argue the most about $0.999999...$
                                        $endgroup$
                                        – Wildcard
                                        Dec 20 '16 at 0:16
















                                      $begingroup$
                                      I think the long division precisely involves the proof of a limit like the sum mentioned above...
                                      $endgroup$
                                      – C-Star-Puppy
                                      Aug 27 '14 at 21:07




                                      $begingroup$
                                      I think the long division precisely involves the proof of a limit like the sum mentioned above...
                                      $endgroup$
                                      – C-Star-Puppy
                                      Aug 27 '14 at 21:07




                                      2




                                      2




                                      $begingroup$
                                      @FreeziiS, for this particular question the fact of a rigorous proof happens to be less important than the fact of a convincing proof. Plenty of people trust the long division algorithm because they learned it in school; they haven't seen a rigorous proof for its workability because they don't care; it obviously works. It is these people (who don't care about or trust rigorous proofs) who argue the most about $0.999999...$
                                      $endgroup$
                                      – Wildcard
                                      Dec 20 '16 at 0:16




                                      $begingroup$
                                      @FreeziiS, for this particular question the fact of a rigorous proof happens to be less important than the fact of a convincing proof. Plenty of people trust the long division algorithm because they learned it in school; they haven't seen a rigorous proof for its workability because they don't care; it obviously works. It is these people (who don't care about or trust rigorous proofs) who argue the most about $0.999999...$
                                      $endgroup$
                                      – Wildcard
                                      Dec 20 '16 at 0:16











                                      9












                                      $begingroup$

                                      Often times people who ask this question are not very convinced by a proof. Since they may not be particularly math inclined, they may feel that a proof is a sort of sleight-of-hand trick, and I find the following intuitive argument (read "don't down-vote me for lack of rigor, lack of rigor is the point") a bit more convincing:



                                      STEP 1) If $.99...neq1$, everyone agrees that it must be less than $1$. Let $alpha$ denote $.99...$, this mysterious number less than $1$.



                                      STEP 2) Using a number line, you can convince them that since $alpha<1$, there must be another number $beta$ such that $alpha<beta<1$.



                                      STEP 3) Since $alpha<beta$, one of the digits of $beta$ must be bigger than the corresponding digit of $alpha$.



                                      STEP 4) However it is usually intitively clear that you cannot make any digit of $.99...$ bigger without making the resulting number (ie $beta$) bigger than $1$.



                                      STEP 5) Thus no such $beta$ can exist, and thus $.99...$ cannot be less than $1$.






                                      share|cite|improve this answer











                                      $endgroup$


















                                        9












                                        $begingroup$

                                        Often times people who ask this question are not very convinced by a proof. Since they may not be particularly math inclined, they may feel that a proof is a sort of sleight-of-hand trick, and I find the following intuitive argument (read "don't down-vote me for lack of rigor, lack of rigor is the point") a bit more convincing:



                                        STEP 1) If $.99...neq1$, everyone agrees that it must be less than $1$. Let $alpha$ denote $.99...$, this mysterious number less than $1$.



                                        STEP 2) Using a number line, you can convince them that since $alpha<1$, there must be another number $beta$ such that $alpha<beta<1$.



                                        STEP 3) Since $alpha<beta$, one of the digits of $beta$ must be bigger than the corresponding digit of $alpha$.



                                        STEP 4) However it is usually intitively clear that you cannot make any digit of $.99...$ bigger without making the resulting number (ie $beta$) bigger than $1$.



                                        STEP 5) Thus no such $beta$ can exist, and thus $.99...$ cannot be less than $1$.






                                        share|cite|improve this answer











                                        $endgroup$
















                                          9












                                          9








                                          9





                                          $begingroup$

                                          Often times people who ask this question are not very convinced by a proof. Since they may not be particularly math inclined, they may feel that a proof is a sort of sleight-of-hand trick, and I find the following intuitive argument (read "don't down-vote me for lack of rigor, lack of rigor is the point") a bit more convincing:



                                          STEP 1) If $.99...neq1$, everyone agrees that it must be less than $1$. Let $alpha$ denote $.99...$, this mysterious number less than $1$.



                                          STEP 2) Using a number line, you can convince them that since $alpha<1$, there must be another number $beta$ such that $alpha<beta<1$.



                                          STEP 3) Since $alpha<beta$, one of the digits of $beta$ must be bigger than the corresponding digit of $alpha$.



                                          STEP 4) However it is usually intitively clear that you cannot make any digit of $.99...$ bigger without making the resulting number (ie $beta$) bigger than $1$.



                                          STEP 5) Thus no such $beta$ can exist, and thus $.99...$ cannot be less than $1$.






                                          share|cite|improve this answer











                                          $endgroup$



                                          Often times people who ask this question are not very convinced by a proof. Since they may not be particularly math inclined, they may feel that a proof is a sort of sleight-of-hand trick, and I find the following intuitive argument (read "don't down-vote me for lack of rigor, lack of rigor is the point") a bit more convincing:



                                          STEP 1) If $.99...neq1$, everyone agrees that it must be less than $1$. Let $alpha$ denote $.99...$, this mysterious number less than $1$.



                                          STEP 2) Using a number line, you can convince them that since $alpha<1$, there must be another number $beta$ such that $alpha<beta<1$.



                                          STEP 3) Since $alpha<beta$, one of the digits of $beta$ must be bigger than the corresponding digit of $alpha$.



                                          STEP 4) However it is usually intitively clear that you cannot make any digit of $.99...$ bigger without making the resulting number (ie $beta$) bigger than $1$.



                                          STEP 5) Thus no such $beta$ can exist, and thus $.99...$ cannot be less than $1$.







                                          share|cite|improve this answer














                                          share|cite|improve this answer



                                          share|cite|improve this answer








                                          answered Sep 15 '14 at 14:39


























                                          community wiki





                                          Trevor Richards
























                                              7












                                              $begingroup$

                                              The real number system is defined as an extension of the rationals with the property that any sequence with an upper bound has a LEAST upper bound. The expression " 0.9-repeated" is defined to be the least real-number upper bound of the sequence 0.9. 0.99, 0.999,..... , which is 1. The rationals (and the reals) can also be extended to an arithmetic system (an ordered field) in which there are positive values which are less than every positive rational. In such systems the expression ".9-repeated" has no meaning.






                                              share|cite|improve this answer











                                              $endgroup$


















                                                7












                                                $begingroup$

                                                The real number system is defined as an extension of the rationals with the property that any sequence with an upper bound has a LEAST upper bound. The expression " 0.9-repeated" is defined to be the least real-number upper bound of the sequence 0.9. 0.99, 0.999,..... , which is 1. The rationals (and the reals) can also be extended to an arithmetic system (an ordered field) in which there are positive values which are less than every positive rational. In such systems the expression ".9-repeated" has no meaning.






                                                share|cite|improve this answer











                                                $endgroup$
















                                                  7












                                                  7








                                                  7





                                                  $begingroup$

                                                  The real number system is defined as an extension of the rationals with the property that any sequence with an upper bound has a LEAST upper bound. The expression " 0.9-repeated" is defined to be the least real-number upper bound of the sequence 0.9. 0.99, 0.999,..... , which is 1. The rationals (and the reals) can also be extended to an arithmetic system (an ordered field) in which there are positive values which are less than every positive rational. In such systems the expression ".9-repeated" has no meaning.






                                                  share|cite|improve this answer











                                                  $endgroup$



                                                  The real number system is defined as an extension of the rationals with the property that any sequence with an upper bound has a LEAST upper bound. The expression " 0.9-repeated" is defined to be the least real-number upper bound of the sequence 0.9. 0.99, 0.999,..... , which is 1. The rationals (and the reals) can also be extended to an arithmetic system (an ordered field) in which there are positive values which are less than every positive rational. In such systems the expression ".9-repeated" has no meaning.







                                                  share|cite|improve this answer














                                                  share|cite|improve this answer



                                                  share|cite|improve this answer








                                                  answered Jul 16 '15 at 5:37


























                                                  community wiki





                                                  DanielWainfleet
























                                                      7












                                                      $begingroup$

                                                      The problem isn't proving that $0.9999... = 1$. There are many proofs and they all are easy.



                                                      The problem is being convinced that every argument you are making actually is valid and makes sense, and not having a sinking feeling you aren't just falling for some parlor trick.



                                                      $0.99...9;$ (with $n$ 9s) is $sum_{i= 1}^n frac 9 {10^i}$ so "obviously" $0.999....$ (with an infinite number of 9s) is $sum_{i = 1}^{infty} frac 9{10^i}$.



                                                      The obvious objection is: does it even make sense to talk about adding an infinite number of terms? How can we talk about taking and adding an infinite number of terms?



                                                      And it's a legitimate objection.



                                                      So when we learn math in elementary school we are told: Every real number can be written as a decimal expansion (maybe infinite) and every possible decimal expansion is a real number. And this is true. But we are not told why and we are expected to take it on faith, and we usually do.



                                                      IF we take this on faith then a proof is very easy:



                                                      $0.9999.... = sum_{i = 1}^{infty} frac 9{10^i}$



                                                      $10*(0.9999....) = 10*sum_{i = 1}^{infty} frac 9{10^i}= sum_{i = 1}^{infty} frac {90}{10^i}=$



                                                      $sum_{i = 1}^{infty} frac 9{10^{i-1}} = 9/10^0 + sum_{i = 2}^{infty} frac 9{10^{i-1}}= 9 + sum_{i = 1}^{infty} frac 9{10^i}$ (Look at the indexes!)



                                                      So...



                                                      $10*(0.999...) - (0.9999...) = (10 - 1)*0.9999.... = 9*0.99999.... = $



                                                      $9 + sum_{i = 1}^{infty} frac 9{10^i} - sum_{i = 1}^{infty} frac 9{10^i} = 9$.



                                                      So...



                                                      $0.9999.... = 9/9 = 1$.



                                                      Easy! !!!!!!!IF!!!!!!! we take it on faith that: Every real number can be written as a decimal expansion (maybe infinite) and every possible decimal expansion is a real number.



                                                      So why can we take that on faith? That's the issue: why is that true and what does it mean?



                                                      So....



                                                      We've got the Integers. We use them to count discrete measurements. We can use an integer to divide a unit 1 into $m$ sub-units to measure measurements of $1/m$. As the $m$ can be as large as we want the $1/m$ can be as precise as we want and the system of all possible $n/m; m ne 0$ can measure any possible quantity with arbitrary and infinite precision.



                                                      We hope. We call these $n/m$ numbers the Rationals and everything is fine until we discover that we can't actually measure measurements such as the square root of two or pi.



                                                      But the Rationals still have infinite precision. We can get within 1/10 away from pi. We can get within 1/100 away from pi. Within $1/10^n$ for any possible power of 10.



                                                      At this point, we hope we can say "we can't measure it with any finite power of 10 but we can always go one more significant measure, so if we go through infinite powers of 10 we will measure it to precision" and we hope that explanation will be convincing.



                                                      But it isn't really. We have these "missing numbers" and we can get infinitely close the them, but what are they really?



                                                      Well, we decide to become math majors and in our senior year of college we take a Real Analysis course and we find out.



                                                      We can view numbers as sets of rational numbers. We can split the rational numbers at any point into two sets. We can split the rational numbers so that all the rational numbers less than 1/2 are in set A and all the rational numbers greater than or equal to 1/2 are in set B (which we ignore; we are only interested in set A.)



                                                      These "cuts" can occur at any point but they must follow the following rules:



                                                      --the set A of all the smaller rational numbers is not empty. Nor does it contain every rational number. Some rational number is not in it.



                                                      --if any rational number (call it q) is in A, then every rational number smaller than q is also in A. (This means that if r is a rational not in A, then every rational bigger than r is also not in A.)



                                                      -- A does not have a single largest element. (So it can be all the elements less than 1/2 but it can't be all the elements less than or equal to 1/2).



                                                      And we let $overline R$ be the collection of all possible ways to "cut" the rational numbers in half that way.



                                                      Notice sometimes the cut will occur at a rational number (all the rationals less than 1/2), but sometimes it will occur at points "between" the rational numbers. (All the rationals whose squares are less than 2). So the collection $overline R$ is a larger set than the set of Rational numbers.



                                                      It turns out we can define the Real numbers as the points of $overline R$ where we can cut the rationals in two.



                                                      We need to do a bit or work to show that this is actually a number system. We say $x, y in overline R; x < y$ if the "Set A made by cutting at x" $subset$ "Set A made by cutting at y". And we say $x + y = $ the point where we need to cut so that the set A created contains all the sums of the two other sets created by cutting at x and y. And we have to prove math works on $overline R$. But we can do it. And we do.



                                                      But as a consequence we see that every real number is the least upper bound limit of a sequence of rational numbers. That's pretty much the definition of what a "cut point" is; the point that separate all the rationals less than it from all the other rationals.



                                                      I like to say (somewhat trivially) that: the real number $x$ is the least upper bound of all the rational numbers that are smaller than $x$. And it's true!



                                                      In the real numbers, every real number is the limit of some sequence of rational numbers. And every bounded sequence of rational numbers will have a real number least upper bound limit.



                                                      ...



                                                      Let that sink in for a minute.



                                                      =====



                                                      Okay, so given a sequence {3, 3.1, 3.14, 3.141,....} = {finite decimals that are less than pi} is a bounded sequence of rational numbers so $pi = $ the limit of the sequence which is also the limit of the infinite sequence 3.1415926....



                                                      It now makes sense to talk of $0.9999.... = sum_{i=1}^{infty}9/10^i = lim{sum_{i=1}^n9/10^i}$ = a precise and real number.



                                                      And from there we can say with confidence that that number is $1$. (By any of these proofs.)






                                                      share|cite|improve this answer











                                                      $endgroup$


















                                                        7












                                                        $begingroup$

                                                        The problem isn't proving that $0.9999... = 1$. There are many proofs and they all are easy.



                                                        The problem is being convinced that every argument you are making actually is valid and makes sense, and not having a sinking feeling you aren't just falling for some parlor trick.



                                                        $0.99...9;$ (with $n$ 9s) is $sum_{i= 1}^n frac 9 {10^i}$ so "obviously" $0.999....$ (with an infinite number of 9s) is $sum_{i = 1}^{infty} frac 9{10^i}$.



                                                        The obvious objection is: does it even make sense to talk about adding an infinite number of terms? How can we talk about taking and adding an infinite number of terms?



                                                        And it's a legitimate objection.



                                                        So when we learn math in elementary school we are told: Every real number can be written as a decimal expansion (maybe infinite) and every possible decimal expansion is a real number. And this is true. But we are not told why and we are expected to take it on faith, and we usually do.



                                                        IF we take this on faith then a proof is very easy:



                                                        $0.9999.... = sum_{i = 1}^{infty} frac 9{10^i}$



                                                        $10*(0.9999....) = 10*sum_{i = 1}^{infty} frac 9{10^i}= sum_{i = 1}^{infty} frac {90}{10^i}=$



                                                        $sum_{i = 1}^{infty} frac 9{10^{i-1}} = 9/10^0 + sum_{i = 2}^{infty} frac 9{10^{i-1}}= 9 + sum_{i = 1}^{infty} frac 9{10^i}$ (Look at the indexes!)



                                                        So...



                                                        $10*(0.999...) - (0.9999...) = (10 - 1)*0.9999.... = 9*0.99999.... = $



                                                        $9 + sum_{i = 1}^{infty} frac 9{10^i} - sum_{i = 1}^{infty} frac 9{10^i} = 9$.



                                                        So...



                                                        $0.9999.... = 9/9 = 1$.



                                                        Easy! !!!!!!!IF!!!!!!! we take it on faith that: Every real number can be written as a decimal expansion (maybe infinite) and every possible decimal expansion is a real number.



                                                        So why can we take that on faith? That's the issue: why is that true and what does it mean?



                                                        So....



                                                        We've got the Integers. We use them to count discrete measurements. We can use an integer to divide a unit 1 into $m$ sub-units to measure measurements of $1/m$. As the $m$ can be as large as we want the $1/m$ can be as precise as we want and the system of all possible $n/m; m ne 0$ can measure any possible quantity with arbitrary and infinite precision.



                                                        We hope. We call these $n/m$ numbers the Rationals and everything is fine until we discover that we can't actually measure measurements such as the square root of two or pi.



                                                        But the Rationals still have infinite precision. We can get within 1/10 away from pi. We can get within 1/100 away from pi. Within $1/10^n$ for any possible power of 10.



                                                        At this point, we hope we can say "we can't measure it with any finite power of 10 but we can always go one more significant measure, so if we go through infinite powers of 10 we will measure it to precision" and we hope that explanation will be convincing.



                                                        But it isn't really. We have these "missing numbers" and we can get infinitely close the them, but what are they really?



                                                        Well, we decide to become math majors and in our senior year of college we take a Real Analysis course and we find out.



                                                        We can view numbers as sets of rational numbers. We can split the rational numbers at any point into two sets. We can split the rational numbers so that all the rational numbers less than 1/2 are in set A and all the rational numbers greater than or equal to 1/2 are in set B (which we ignore; we are only interested in set A.)



                                                        These "cuts" can occur at any point but they must follow the following rules:



                                                        --the set A of all the smaller rational numbers is not empty. Nor does it contain every rational number. Some rational number is not in it.



                                                        --if any rational number (call it q) is in A, then every rational number smaller than q is also in A. (This means that if r is a rational not in A, then every rational bigger than r is also not in A.)



                                                        -- A does not have a single largest element. (So it can be all the elements less than 1/2 but it can't be all the elements less than or equal to 1/2).



                                                        And we let $overline R$ be the collection of all possible ways to "cut" the rational numbers in half that way.



                                                        Notice sometimes the cut will occur at a rational number (all the rationals less than 1/2), but sometimes it will occur at points "between" the rational numbers. (All the rationals whose squares are less than 2). So the collection $overline R$ is a larger set than the set of Rational numbers.



                                                        It turns out we can define the Real numbers as the points of $overline R$ where we can cut the rationals in two.



                                                        We need to do a bit or work to show that this is actually a number system. We say $x, y in overline R; x < y$ if the "Set A made by cutting at x" $subset$ "Set A made by cutting at y". And we say $x + y = $ the point where we need to cut so that the set A created contains all the sums of the two other sets created by cutting at x and y. And we have to prove math works on $overline R$. But we can do it. And we do.



                                                        But as a consequence we see that every real number is the least upper bound limit of a sequence of rational numbers. That's pretty much the definition of what a "cut point" is; the point that separate all the rationals less than it from all the other rationals.



                                                        I like to say (somewhat trivially) that: the real number $x$ is the least upper bound of all the rational numbers that are smaller than $x$. And it's true!



                                                        In the real numbers, every real number is the limit of some sequence of rational numbers. And every bounded sequence of rational numbers will have a real number least upper bound limit.



                                                        ...



                                                        Let that sink in for a minute.



                                                        =====



                                                        Okay, so given a sequence {3, 3.1, 3.14, 3.141,....} = {finite decimals that are less than pi} is a bounded sequence of rational numbers so $pi = $ the limit of the sequence which is also the limit of the infinite sequence 3.1415926....



                                                        It now makes sense to talk of $0.9999.... = sum_{i=1}^{infty}9/10^i = lim{sum_{i=1}^n9/10^i}$ = a precise and real number.



                                                        And from there we can say with confidence that that number is $1$. (By any of these proofs.)






                                                        share|cite|improve this answer











                                                        $endgroup$
















                                                          7












                                                          7








                                                          7





                                                          $begingroup$

                                                          The problem isn't proving that $0.9999... = 1$. There are many proofs and they all are easy.



                                                          The problem is being convinced that every argument you are making actually is valid and makes sense, and not having a sinking feeling you aren't just falling for some parlor trick.



                                                          $0.99...9;$ (with $n$ 9s) is $sum_{i= 1}^n frac 9 {10^i}$ so "obviously" $0.999....$ (with an infinite number of 9s) is $sum_{i = 1}^{infty} frac 9{10^i}$.



                                                          The obvious objection is: does it even make sense to talk about adding an infinite number of terms? How can we talk about taking and adding an infinite number of terms?



                                                          And it's a legitimate objection.



                                                          So when we learn math in elementary school we are told: Every real number can be written as a decimal expansion (maybe infinite) and every possible decimal expansion is a real number. And this is true. But we are not told why and we are expected to take it on faith, and we usually do.



                                                          IF we take this on faith then a proof is very easy:



                                                          $0.9999.... = sum_{i = 1}^{infty} frac 9{10^i}$



                                                          $10*(0.9999....) = 10*sum_{i = 1}^{infty} frac 9{10^i}= sum_{i = 1}^{infty} frac {90}{10^i}=$



                                                          $sum_{i = 1}^{infty} frac 9{10^{i-1}} = 9/10^0 + sum_{i = 2}^{infty} frac 9{10^{i-1}}= 9 + sum_{i = 1}^{infty} frac 9{10^i}$ (Look at the indexes!)



                                                          So...



                                                          $10*(0.999...) - (0.9999...) = (10 - 1)*0.9999.... = 9*0.99999.... = $



                                                          $9 + sum_{i = 1}^{infty} frac 9{10^i} - sum_{i = 1}^{infty} frac 9{10^i} = 9$.



                                                          So...



                                                          $0.9999.... = 9/9 = 1$.



                                                          Easy! !!!!!!!IF!!!!!!! we take it on faith that: Every real number can be written as a decimal expansion (maybe infinite) and every possible decimal expansion is a real number.



                                                          So why can we take that on faith? That's the issue: why is that true and what does it mean?



                                                          So....



                                                          We've got the Integers. We use them to count discrete measurements. We can use an integer to divide a unit 1 into $m$ sub-units to measure measurements of $1/m$. As the $m$ can be as large as we want the $1/m$ can be as precise as we want and the system of all possible $n/m; m ne 0$ can measure any possible quantity with arbitrary and infinite precision.



                                                          We hope. We call these $n/m$ numbers the Rationals and everything is fine until we discover that we can't actually measure measurements such as the square root of two or pi.



                                                          But the Rationals still have infinite precision. We can get within 1/10 away from pi. We can get within 1/100 away from pi. Within $1/10^n$ for any possible power of 10.



                                                          At this point, we hope we can say "we can't measure it with any finite power of 10 but we can always go one more significant measure, so if we go through infinite powers of 10 we will measure it to precision" and we hope that explanation will be convincing.



                                                          But it isn't really. We have these "missing numbers" and we can get infinitely close the them, but what are they really?



                                                          Well, we decide to become math majors and in our senior year of college we take a Real Analysis course and we find out.



                                                          We can view numbers as sets of rational numbers. We can split the rational numbers at any point into two sets. We can split the rational numbers so that all the rational numbers less than 1/2 are in set A and all the rational numbers greater than or equal to 1/2 are in set B (which we ignore; we are only interested in set A.)



                                                          These "cuts" can occur at any point but they must follow the following rules:



                                                          --the set A of all the smaller rational numbers is not empty. Nor does it contain every rational number. Some rational number is not in it.



                                                          --if any rational number (call it q) is in A, then every rational number smaller than q is also in A. (This means that if r is a rational not in A, then every rational bigger than r is also not in A.)



                                                          -- A does not have a single largest element. (So it can be all the elements less than 1/2 but it can't be all the elements less than or equal to 1/2).



                                                          And we let $overline R$ be the collection of all possible ways to "cut" the rational numbers in half that way.



                                                          Notice sometimes the cut will occur at a rational number (all the rationals less than 1/2), but sometimes it will occur at points "between" the rational numbers. (All the rationals whose squares are less than 2). So the collection $overline R$ is a larger set than the set of Rational numbers.



                                                          It turns out we can define the Real numbers as the points of $overline R$ where we can cut the rationals in two.



                                                          We need to do a bit or work to show that this is actually a number system. We say $x, y in overline R; x < y$ if the "Set A made by cutting at x" $subset$ "Set A made by cutting at y". And we say $x + y = $ the point where we need to cut so that the set A created contains all the sums of the two other sets created by cutting at x and y. And we have to prove math works on $overline R$. But we can do it. And we do.



                                                          But as a consequence we see that every real number is the least upper bound limit of a sequence of rational numbers. That's pretty much the definition of what a "cut point" is; the point that separate all the rationals less than it from all the other rationals.



                                                          I like to say (somewhat trivially) that: the real number $x$ is the least upper bound of all the rational numbers that are smaller than $x$. And it's true!



                                                          In the real numbers, every real number is the limit of some sequence of rational numbers. And every bounded sequence of rational numbers will have a real number least upper bound limit.



                                                          ...



                                                          Let that sink in for a minute.



                                                          =====



                                                          Okay, so given a sequence {3, 3.1, 3.14, 3.141,....} = {finite decimals that are less than pi} is a bounded sequence of rational numbers so $pi = $ the limit of the sequence which is also the limit of the infinite sequence 3.1415926....



                                                          It now makes sense to talk of $0.9999.... = sum_{i=1}^{infty}9/10^i = lim{sum_{i=1}^n9/10^i}$ = a precise and real number.



                                                          And from there we can say with confidence that that number is $1$. (By any of these proofs.)






                                                          share|cite|improve this answer











                                                          $endgroup$



                                                          The problem isn't proving that $0.9999... = 1$. There are many proofs and they all are easy.



                                                          The problem is being convinced that every argument you are making actually is valid and makes sense, and not having a sinking feeling you aren't just falling for some parlor trick.



                                                          $0.99...9;$ (with $n$ 9s) is $sum_{i= 1}^n frac 9 {10^i}$ so "obviously" $0.999....$ (with an infinite number of 9s) is $sum_{i = 1}^{infty} frac 9{10^i}$.



                                                          The obvious objection is: does it even make sense to talk about adding an infinite number of terms? How can we talk about taking and adding an infinite number of terms?



                                                          And it's a legitimate objection.



                                                          So when we learn math in elementary school we are told: Every real number can be written as a decimal expansion (maybe infinite) and every possible decimal expansion is a real number. And this is true. But we are not told why and we are expected to take it on faith, and we usually do.



                                                          IF we take this on faith then a proof is very easy:



                                                          $0.9999.... = sum_{i = 1}^{infty} frac 9{10^i}$



                                                          $10*(0.9999....) = 10*sum_{i = 1}^{infty} frac 9{10^i}= sum_{i = 1}^{infty} frac {90}{10^i}=$



                                                          $sum_{i = 1}^{infty} frac 9{10^{i-1}} = 9/10^0 + sum_{i = 2}^{infty} frac 9{10^{i-1}}= 9 + sum_{i = 1}^{infty} frac 9{10^i}$ (Look at the indexes!)



                                                          So...



                                                          $10*(0.999...) - (0.9999...) = (10 - 1)*0.9999.... = 9*0.99999.... = $



                                                          $9 + sum_{i = 1}^{infty} frac 9{10^i} - sum_{i = 1}^{infty} frac 9{10^i} = 9$.



                                                          So...



                                                          $0.9999.... = 9/9 = 1$.



                                                          Easy! !!!!!!!IF!!!!!!! we take it on faith that: Every real number can be written as a decimal expansion (maybe infinite) and every possible decimal expansion is a real number.



                                                          So why can we take that on faith? That's the issue: why is that true and what does it mean?



                                                          So....



                                                          We've got the Integers. We use them to count discrete measurements. We can use an integer to divide a unit 1 into $m$ sub-units to measure measurements of $1/m$. As the $m$ can be as large as we want the $1/m$ can be as precise as we want and the system of all possible $n/m; m ne 0$ can measure any possible quantity with arbitrary and infinite precision.



                                                          We hope. We call these $n/m$ numbers the Rationals and everything is fine until we discover that we can't actually measure measurements such as the square root of two or pi.



                                                          But the Rationals still have infinite precision. We can get within 1/10 away from pi. We can get within 1/100 away from pi. Within $1/10^n$ for any possible power of 10.



                                                          At this point, we hope we can say "we can't measure it with any finite power of 10 but we can always go one more significant measure, so if we go through infinite powers of 10 we will measure it to precision" and we hope that explanation will be convincing.



                                                          But it isn't really. We have these "missing numbers" and we can get infinitely close the them, but what are they really?



                                                          Well, we decide to become math majors and in our senior year of college we take a Real Analysis course and we find out.



                                                          We can view numbers as sets of rational numbers. We can split the rational numbers at any point into two sets. We can split the rational numbers so that all the rational numbers less than 1/2 are in set A and all the rational numbers greater than or equal to 1/2 are in set B (which we ignore; we are only interested in set A.)



                                                          These "cuts" can occur at any point but they must follow the following rules:



                                                          --the set A of all the smaller rational numbers is not empty. Nor does it contain every rational number. Some rational number is not in it.



                                                          --if any rational number (call it q) is in A, then every rational number smaller than q is also in A. (This means that if r is a rational not in A, then every rational bigger than r is also not in A.)



                                                          -- A does not have a single largest element. (So it can be all the elements less than 1/2 but it can't be all the elements less than or equal to 1/2).



                                                          And we let $overline R$ be the collection of all possible ways to "cut" the rational numbers in half that way.



                                                          Notice sometimes the cut will occur at a rational number (all the rationals less than 1/2), but sometimes it will occur at points "between" the rational numbers. (All the rationals whose squares are less than 2). So the collection $overline R$ is a larger set than the set of Rational numbers.



                                                          It turns out we can define the Real numbers as the points of $overline R$ where we can cut the rationals in two.



                                                          We need to do a bit or work to show that this is actually a number system. We say $x, y in overline R; x < y$ if the "Set A made by cutting at x" $subset$ "Set A made by cutting at y". And we say $x + y = $ the point where we need to cut so that the set A created contains all the sums of the two other sets created by cutting at x and y. And we have to prove math works on $overline R$. But we can do it. And we do.



                                                          But as a consequence we see that every real number is the least upper bound limit of a sequence of rational numbers. That's pretty much the definition of what a "cut point" is; the point that separate all the rationals less than it from all the other rationals.



                                                          I like to say (somewhat trivially) that: the real number $x$ is the least upper bound of all the rational numbers that are smaller than $x$. And it's true!



                                                          In the real numbers, every real number is the limit of some sequence of rational numbers. And every bounded sequence of rational numbers will have a real number least upper bound limit.



                                                          ...



                                                          Let that sink in for a minute.



                                                          =====



                                                          Okay, so given a sequence {3, 3.1, 3.14, 3.141,....} = {finite decimals that are less than pi} is a bounded sequence of rational numbers so $pi = $ the limit of the sequence which is also the limit of the infinite sequence 3.1415926....



                                                          It now makes sense to talk of $0.9999.... = sum_{i=1}^{infty}9/10^i = lim{sum_{i=1}^n9/10^i}$ = a precise and real number.



                                                          And from there we can say with confidence that that number is $1$. (By any of these proofs.)







                                                          share|cite|improve this answer














                                                          share|cite|improve this answer



                                                          share|cite|improve this answer








                                                          edited Jan 29 '16 at 9:13


























                                                          community wiki





                                                          2 revs, 2 users 99%
                                                          fleablood
























                                                              6












                                                              $begingroup$

                                                              Another approach is the following:



                                                              $$begin{align}
                                                              0.overline9 &=lim_{n to infty} 0.underbrace{99dots 9}_{ntext{ times}} \
                                                              &= lim_{n to infty} sumlimits_{k=1}^n frac{9}{10^k} \
                                                              &=lim_{n to infty} 1-frac{1}{10^n} \
                                                              &=1-lim_{n to infty} frac{1}{10^n} \
                                                              &=1.
                                                              end{align}$$






                                                              share|cite|improve this answer











                                                              $endgroup$









                                                              • 7




                                                                $begingroup$
                                                                This approach appears in Isaac's answer from 4 years earlier.
                                                                $endgroup$
                                                                – Jonas Meyer
                                                                Aug 26 '14 at 2:15
















                                                              6












                                                              $begingroup$

                                                              Another approach is the following:



                                                              $$begin{align}
                                                              0.overline9 &=lim_{n to infty} 0.underbrace{99dots 9}_{ntext{ times}} \
                                                              &= lim_{n to infty} sumlimits_{k=1}^n frac{9}{10^k} \
                                                              &=lim_{n to infty} 1-frac{1}{10^n} \
                                                              &=1-lim_{n to infty} frac{1}{10^n} \
                                                              &=1.
                                                              end{align}$$






                                                              share|cite|improve this answer











                                                              $endgroup$









                                                              • 7




                                                                $begingroup$
                                                                This approach appears in Isaac's answer from 4 years earlier.
                                                                $endgroup$
                                                                – Jonas Meyer
                                                                Aug 26 '14 at 2:15














                                                              6












                                                              6








                                                              6





                                                              $begingroup$

                                                              Another approach is the following:



                                                              $$begin{align}
                                                              0.overline9 &=lim_{n to infty} 0.underbrace{99dots 9}_{ntext{ times}} \
                                                              &= lim_{n to infty} sumlimits_{k=1}^n frac{9}{10^k} \
                                                              &=lim_{n to infty} 1-frac{1}{10^n} \
                                                              &=1-lim_{n to infty} frac{1}{10^n} \
                                                              &=1.
                                                              end{align}$$






                                                              share|cite|improve this answer











                                                              $endgroup$



                                                              Another approach is the following:



                                                              $$begin{align}
                                                              0.overline9 &=lim_{n to infty} 0.underbrace{99dots 9}_{ntext{ times}} \
                                                              &= lim_{n to infty} sumlimits_{k=1}^n frac{9}{10^k} \
                                                              &=lim_{n to infty} 1-frac{1}{10^n} \
                                                              &=1-lim_{n to infty} frac{1}{10^n} \
                                                              &=1.
                                                              end{align}$$







                                                              share|cite|improve this answer














                                                              share|cite|improve this answer



                                                              share|cite|improve this answer








                                                              edited Dec 11 '18 at 11:30


























                                                              community wiki





                                                              2 revs, 2 users 54%
                                                              Shaun









                                                              • 7




                                                                $begingroup$
                                                                This approach appears in Isaac's answer from 4 years earlier.
                                                                $endgroup$
                                                                – Jonas Meyer
                                                                Aug 26 '14 at 2:15














                                                              • 7




                                                                $begingroup$
                                                                This approach appears in Isaac's answer from 4 years earlier.
                                                                $endgroup$
                                                                – Jonas Meyer
                                                                Aug 26 '14 at 2:15








                                                              7




                                                              7




                                                              $begingroup$
                                                              This approach appears in Isaac's answer from 4 years earlier.
                                                              $endgroup$
                                                              – Jonas Meyer
                                                              Aug 26 '14 at 2:15




                                                              $begingroup$
                                                              This approach appears in Isaac's answer from 4 years earlier.
                                                              $endgroup$
                                                              – Jonas Meyer
                                                              Aug 26 '14 at 2:15











                                                              5












                                                              $begingroup$

                                                              There are some situations in which something like $0.99999ldots < 1$ indeed holds. Here is one coming from social choice theory.



                                                              Let $w_1>w_2>ldots$ be an infinite sequence of positive numbers, and let $T$ be a number in the range $(0,sum_i w_i)$. Pick an index $i$. Choose a random permutation $pi$ of the positive integers, and consider the running totals
                                                              $$
                                                              w_{pi(1)}, w_{pi(1)} + w_{pi(2)}, w_{pi(1)} + w_{pi(2)} + w_{pi(3)}, cdots
                                                              $$
                                                              The Shapley value $varphi_i(T)$ is the probability that the first time that the running total exceeds $T$ is when $w_i$ is added.



                                                              We will particularly be interested in the case in which the sequence $w_i$ is super-increasing: for each $i$, $w_i geq sum_{j=i+1}^infty w_j$. The simplest case is $w_i = 2^{-i}$. Every number $T in (0,1)$ can be written in the form
                                                              $$ T = 2^{-a_0} + 2^{-a_1} + cdots, qquad a_0 < a_1 < cdots. $$
                                                              In this case we can give an explicit formula for $varphi_i(T)$:
                                                              $$
                                                              varphi_i(T) = begin{cases}
                                                              sum_{tcolon a_t>i} frac{1}{a_t binom{a_t-1}{t}} & text{if } i notin {a_0,a_1,ldots}, \
                                                              frac{1}{a_s binom{a_s-1}{s}} - sum_{tcolon a_t>i} frac{1}{a_t binom{a_t-1}{t-1}} & text{if } i = a_s.
                                                              end{cases}
                                                              $$



                                                              The first two functions are plotted here:
                                                              enter image description here



                                                              What happens for different sets of weights? The same formula applies, for
                                                              $$
                                                              T = w_{a_0} + w_{a_1} + cdots, qquad a_0 < a_1 < cdots.
                                                              $$
                                                              In general not all $T$ will be of this form; for $T$ not of this form, we take the lowest upper bound which is of this form. What we get for $w_i = 3^{-i}$ is:
                                                              enter image description here



                                                              Notice all the horizontal parts, for example the blue line at $y=1$ at $x in (1/6,1/3)$. Where does this stem from? Note that $1/3 = 3^{-1} = w_1$, whereas $1/6 = sum_{i=2}^infty 3^{-i} = sum_{i=2}^infty w_i$. If we substitute $w_i = 2^{-i}$, then $1/3$ corresponds to $0.1$ (in binary), whereas $1/6$ corresponds to $0.011111ldots$. So in this case there is a (visible) gap between $0.011111ldots$ and $0.1$!



                                                              For more, take a look at this question and this manuscript.






                                                              share|cite|improve this answer











                                                              $endgroup$


















                                                                5












                                                                $begingroup$

                                                                There are some situations in which something like $0.99999ldots < 1$ indeed holds. Here is one coming from social choice theory.



                                                                Let $w_1>w_2>ldots$ be an infinite sequence of positive numbers, and let $T$ be a number in the range $(0,sum_i w_i)$. Pick an index $i$. Choose a random permutation $pi$ of the positive integers, and consider the running totals
                                                                $$
                                                                w_{pi(1)}, w_{pi(1)} + w_{pi(2)}, w_{pi(1)} + w_{pi(2)} + w_{pi(3)}, cdots
                                                                $$
                                                                The Shapley value $varphi_i(T)$ is the probability that the first time that the running total exceeds $T$ is when $w_i$ is added.



                                                                We will particularly be interested in the case in which the sequence $w_i$ is super-increasing: for each $i$, $w_i geq sum_{j=i+1}^infty w_j$. The simplest case is $w_i = 2^{-i}$. Every number $T in (0,1)$ can be written in the form
                                                                $$ T = 2^{-a_0} + 2^{-a_1} + cdots, qquad a_0 < a_1 < cdots. $$
                                                                In this case we can give an explicit formula for $varphi_i(T)$:
                                                                $$
                                                                varphi_i(T) = begin{cases}
                                                                sum_{tcolon a_t>i} frac{1}{a_t binom{a_t-1}{t}} & text{if } i notin {a_0,a_1,ldots}, \
                                                                frac{1}{a_s binom{a_s-1}{s}} - sum_{tcolon a_t>i} frac{1}{a_t binom{a_t-1}{t-1}} & text{if } i = a_s.
                                                                end{cases}
                                                                $$



                                                                The first two functions are plotted here:
                                                                enter image description here



                                                                What happens for different sets of weights? The same formula applies, for
                                                                $$
                                                                T = w_{a_0} + w_{a_1} + cdots, qquad a_0 < a_1 < cdots.
                                                                $$
                                                                In general not all $T$ will be of this form; for $T$ not of this form, we take the lowest upper bound which is of this form. What we get for $w_i = 3^{-i}$ is:
                                                                enter image description here



                                                                Notice all the horizontal parts, for example the blue line at $y=1$ at $x in (1/6,1/3)$. Where does this stem from? Note that $1/3 = 3^{-1} = w_1$, whereas $1/6 = sum_{i=2}^infty 3^{-i} = sum_{i=2}^infty w_i$. If we substitute $w_i = 2^{-i}$, then $1/3$ corresponds to $0.1$ (in binary), whereas $1/6$ corresponds to $0.011111ldots$. So in this case there is a (visible) gap between $0.011111ldots$ and $0.1$!



                                                                For more, take a look at this question and this manuscript.






                                                                share|cite|improve this answer











                                                                $endgroup$
















                                                                  5












                                                                  5








                                                                  5





                                                                  $begingroup$

                                                                  There are some situations in which something like $0.99999ldots < 1$ indeed holds. Here is one coming from social choice theory.



                                                                  Let $w_1>w_2>ldots$ be an infinite sequence of positive numbers, and let $T$ be a number in the range $(0,sum_i w_i)$. Pick an index $i$. Choose a random permutation $pi$ of the positive integers, and consider the running totals
                                                                  $$
                                                                  w_{pi(1)}, w_{pi(1)} + w_{pi(2)}, w_{pi(1)} + w_{pi(2)} + w_{pi(3)}, cdots
                                                                  $$
                                                                  The Shapley value $varphi_i(T)$ is the probability that the first time that the running total exceeds $T$ is when $w_i$ is added.



                                                                  We will particularly be interested in the case in which the sequence $w_i$ is super-increasing: for each $i$, $w_i geq sum_{j=i+1}^infty w_j$. The simplest case is $w_i = 2^{-i}$. Every number $T in (0,1)$ can be written in the form
                                                                  $$ T = 2^{-a_0} + 2^{-a_1} + cdots, qquad a_0 < a_1 < cdots. $$
                                                                  In this case we can give an explicit formula for $varphi_i(T)$:
                                                                  $$
                                                                  varphi_i(T) = begin{cases}
                                                                  sum_{tcolon a_t>i} frac{1}{a_t binom{a_t-1}{t}} & text{if } i notin {a_0,a_1,ldots}, \
                                                                  frac{1}{a_s binom{a_s-1}{s}} - sum_{tcolon a_t>i} frac{1}{a_t binom{a_t-1}{t-1}} & text{if } i = a_s.
                                                                  end{cases}
                                                                  $$



                                                                  The first two functions are plotted here:
                                                                  enter image description here



                                                                  What happens for different sets of weights? The same formula applies, for
                                                                  $$
                                                                  T = w_{a_0} + w_{a_1} + cdots, qquad a_0 < a_1 < cdots.
                                                                  $$
                                                                  In general not all $T$ will be of this form; for $T$ not of this form, we take the lowest upper bound which is of this form. What we get for $w_i = 3^{-i}$ is:
                                                                  enter image description here



                                                                  Notice all the horizontal parts, for example the blue line at $y=1$ at $x in (1/6,1/3)$. Where does this stem from? Note that $1/3 = 3^{-1} = w_1$, whereas $1/6 = sum_{i=2}^infty 3^{-i} = sum_{i=2}^infty w_i$. If we substitute $w_i = 2^{-i}$, then $1/3$ corresponds to $0.1$ (in binary), whereas $1/6$ corresponds to $0.011111ldots$. So in this case there is a (visible) gap between $0.011111ldots$ and $0.1$!



                                                                  For more, take a look at this question and this manuscript.






                                                                  share|cite|improve this answer











                                                                  $endgroup$



                                                                  There are some situations in which something like $0.99999ldots < 1$ indeed holds. Here is one coming from social choice theory.



                                                                  Let $w_1>w_2>ldots$ be an infinite sequence of positive numbers, and let $T$ be a number in the range $(0,sum_i w_i)$. Pick an index $i$. Choose a random permutation $pi$ of the positive integers, and consider the running totals
                                                                  $$
                                                                  w_{pi(1)}, w_{pi(1)} + w_{pi(2)}, w_{pi(1)} + w_{pi(2)} + w_{pi(3)}, cdots
                                                                  $$
                                                                  The Shapley value $varphi_i(T)$ is the probability that the first time that the running total exceeds $T$ is when $w_i$ is added.



                                                                  We will particularly be interested in the case in which the sequence $w_i$ is super-increasing: for each $i$, $w_i geq sum_{j=i+1}^infty w_j$. The simplest case is $w_i = 2^{-i}$. Every number $T in (0,1)$ can be written in the form
                                                                  $$ T = 2^{-a_0} + 2^{-a_1} + cdots, qquad a_0 < a_1 < cdots. $$
                                                                  In this case we can give an explicit formula for $varphi_i(T)$:
                                                                  $$
                                                                  varphi_i(T) = begin{cases}
                                                                  sum_{tcolon a_t>i} frac{1}{a_t binom{a_t-1}{t}} & text{if } i notin {a_0,a_1,ldots}, \
                                                                  frac{1}{a_s binom{a_s-1}{s}} - sum_{tcolon a_t>i} frac{1}{a_t binom{a_t-1}{t-1}} & text{if } i = a_s.
                                                                  end{cases}
                                                                  $$



                                                                  The first two functions are plotted here:
                                                                  enter image description here



                                                                  What happens for different sets of weights? The same formula applies, for
                                                                  $$
                                                                  T = w_{a_0} + w_{a_1} + cdots, qquad a_0 < a_1 < cdots.
                                                                  $$
                                                                  In general not all $T$ will be of this form; for $T$ not of this form, we take the lowest upper bound which is of this form. What we get for $w_i = 3^{-i}$ is:
                                                                  enter image description here



                                                                  Notice all the horizontal parts, for example the blue line at $y=1$ at $x in (1/6,1/3)$. Where does this stem from? Note that $1/3 = 3^{-1} = w_1$, whereas $1/6 = sum_{i=2}^infty 3^{-i} = sum_{i=2}^infty w_i$. If we substitute $w_i = 2^{-i}$, then $1/3$ corresponds to $0.1$ (in binary), whereas $1/6$ corresponds to $0.011111ldots$. So in this case there is a (visible) gap between $0.011111ldots$ and $0.1$!



                                                                  For more, take a look at this question and this manuscript.







                                                                  share|cite|improve this answer














                                                                  share|cite|improve this answer



                                                                  share|cite|improve this answer








                                                                  edited Apr 13 '17 at 12:19


























                                                                  community wiki





                                                                  2 revs
                                                                  Yuval Filmus
























                                                                      4












                                                                      $begingroup$

                                                                      One cool way I learned to prove this is that, assuming by $0.99999...$ you mean $0.bar{9}$. Well we can say that
                                                                      $$0.bar{9}=sum_{n=1}^{infty}9cdot 10^{-n}=9sum_{n=1}^{infty}frac{1}{10^n}$$
                                                                      Which we know converges by fact that this is a geometric series with the ratio between terms being less than $1$. So we know that
                                                                      $$9sum_{n=1}^{infty}frac{1}{10^n}=9left(frac{1}{1-frac{1}{10}}-1right)=10-9=1$$
                                                                      Note that we subtract off the $1$ in the parentheses because we started indexing at $1$ rather than at $0$, so we have to subtract of the value of the sequence at $n=0$ which is $1$.






                                                                      share|cite|improve this answer











                                                                      $endgroup$









                                                                      • 2




                                                                        $begingroup$
                                                                        "Which we know converges by fact that this is a geometric series with the ratio between terms being less than 1." I don't think anyone who does know this would be questioning whether .9999.... = 1. The only people who are confused on the concept most assuredly do not know this.
                                                                        $endgroup$
                                                                        – fleablood
                                                                        Jan 15 '16 at 20:03










                                                                      • $begingroup$
                                                                        Just because you have studied series doesn't mean you have seen this proof. Hence why I have added it as a wiki answer on such a highly up voted answer, which people visiting it do not necessary have the same mathematical knowledge as the OP.
                                                                        $endgroup$
                                                                        – Will Fisher
                                                                        Jan 15 '16 at 21:55
















                                                                      4












                                                                      $begingroup$

                                                                      One cool way I learned to prove this is that, assuming by $0.99999...$ you mean $0.bar{9}$. Well we can say that
                                                                      $$0.bar{9}=sum_{n=1}^{infty}9cdot 10^{-n}=9sum_{n=1}^{infty}frac{1}{10^n}$$
                                                                      Which we know converges by fact that this is a geometric series with the ratio between terms being less than $1$. So we know that
                                                                      $$9sum_{n=1}^{infty}frac{1}{10^n}=9left(frac{1}{1-frac{1}{10}}-1right)=10-9=1$$
                                                                      Note that we subtract off the $1$ in the parentheses because we started indexing at $1$ rather than at $0$, so we have to subtract of the value of the sequence at $n=0$ which is $1$.






                                                                      share|cite|improve this answer











                                                                      $endgroup$









                                                                      • 2




                                                                        $begingroup$
                                                                        "Which we know converges by fact that this is a geometric series with the ratio between terms being less than 1." I don't think anyone who does know this would be questioning whether .9999.... = 1. The only people who are confused on the concept most assuredly do not know this.
                                                                        $endgroup$
                                                                        – fleablood
                                                                        Jan 15 '16 at 20:03










                                                                      • $begingroup$
                                                                        Just because you have studied series doesn't mean you have seen this proof. Hence why I have added it as a wiki answer on such a highly up voted answer, which people visiting it do not necessary have the same mathematical knowledge as the OP.
                                                                        $endgroup$
                                                                        – Will Fisher
                                                                        Jan 15 '16 at 21:55














                                                                      4












                                                                      4








                                                                      4





                                                                      $begingroup$

                                                                      One cool way I learned to prove this is that, assuming by $0.99999...$ you mean $0.bar{9}$. Well we can say that
                                                                      $$0.bar{9}=sum_{n=1}^{infty}9cdot 10^{-n}=9sum_{n=1}^{infty}frac{1}{10^n}$$
                                                                      Which we know converges by fact that this is a geometric series with the ratio between terms being less than $1$. So we know that
                                                                      $$9sum_{n=1}^{infty}frac{1}{10^n}=9left(frac{1}{1-frac{1}{10}}-1right)=10-9=1$$
                                                                      Note that we subtract off the $1$ in the parentheses because we started indexing at $1$ rather than at $0$, so we have to subtract of the value of the sequence at $n=0$ which is $1$.






                                                                      share|cite|improve this answer











                                                                      $endgroup$



                                                                      One cool way I learned to prove this is that, assuming by $0.99999...$ you mean $0.bar{9}$. Well we can say that
                                                                      $$0.bar{9}=sum_{n=1}^{infty}9cdot 10^{-n}=9sum_{n=1}^{infty}frac{1}{10^n}$$
                                                                      Which we know converges by fact that this is a geometric series with the ratio between terms being less than $1$. So we know that
                                                                      $$9sum_{n=1}^{infty}frac{1}{10^n}=9left(frac{1}{1-frac{1}{10}}-1right)=10-9=1$$
                                                                      Note that we subtract off the $1$ in the parentheses because we started indexing at $1$ rather than at $0$, so we have to subtract of the value of the sequence at $n=0$ which is $1$.







                                                                      share|cite|improve this answer














                                                                      share|cite|improve this answer



                                                                      share|cite|improve this answer








                                                                      answered Dec 28 '15 at 22:45


























                                                                      community wiki





                                                                      Will Fisher









                                                                      • 2




                                                                        $begingroup$
                                                                        "Which we know converges by fact that this is a geometric series with the ratio between terms being less than 1." I don't think anyone who does know this would be questioning whether .9999.... = 1. The only people who are confused on the concept most assuredly do not know this.
                                                                        $endgroup$
                                                                        – fleablood
                                                                        Jan 15 '16 at 20:03










                                                                      • $begingroup$
                                                                        Just because you have studied series doesn't mean you have seen this proof. Hence why I have added it as a wiki answer on such a highly up voted answer, which people visiting it do not necessary have the same mathematical knowledge as the OP.
                                                                        $endgroup$
                                                                        – Will Fisher
                                                                        Jan 15 '16 at 21:55














                                                                      • 2




                                                                        $begingroup$
                                                                        "Which we know converges by fact that this is a geometric series with the ratio between terms being less than 1." I don't think anyone who does know this would be questioning whether .9999.... = 1. The only people who are confused on the concept most assuredly do not know this.
                                                                        $endgroup$
                                                                        – fleablood
                                                                        Jan 15 '16 at 20:03










                                                                      • $begingroup$
                                                                        Just because you have studied series doesn't mean you have seen this proof. Hence why I have added it as a wiki answer on such a highly up voted answer, which people visiting it do not necessary have the same mathematical knowledge as the OP.
                                                                        $endgroup$
                                                                        – Will Fisher
                                                                        Jan 15 '16 at 21:55








                                                                      2




                                                                      2




                                                                      $begingroup$
                                                                      "Which we know converges by fact that this is a geometric series with the ratio between terms being less than 1." I don't think anyone who does know this would be questioning whether .9999.... = 1. The only people who are confused on the concept most assuredly do not know this.
                                                                      $endgroup$
                                                                      – fleablood
                                                                      Jan 15 '16 at 20:03




                                                                      $begingroup$
                                                                      "Which we know converges by fact that this is a geometric series with the ratio between terms being less than 1." I don't think anyone who does know this would be questioning whether .9999.... = 1. The only people who are confused on the concept most assuredly do not know this.
                                                                      $endgroup$
                                                                      – fleablood
                                                                      Jan 15 '16 at 20:03












                                                                      $begingroup$
                                                                      Just because you have studied series doesn't mean you have seen this proof. Hence why I have added it as a wiki answer on such a highly up voted answer, which people visiting it do not necessary have the same mathematical knowledge as the OP.
                                                                      $endgroup$
                                                                      – Will Fisher
                                                                      Jan 15 '16 at 21:55




                                                                      $begingroup$
                                                                      Just because you have studied series doesn't mean you have seen this proof. Hence why I have added it as a wiki answer on such a highly up voted answer, which people visiting it do not necessary have the same mathematical knowledge as the OP.
                                                                      $endgroup$
                                                                      – Will Fisher
                                                                      Jan 15 '16 at 21:55











                                                                      3












                                                                      $begingroup$

                                                                      Here's my favorite reason why $.999ldots$ should equal $1$:$^{*}$
                                                                      begin{align*}
                                                                      .999ldots + .999ldots
                                                                      &= (.9 + .09 + .009 + cdots) + (.9 + .09 + .009 + cdots) \
                                                                      &= (.9 + .9) + (.09 + .09) + (.009 + .009) + cdots \
                                                                      &= 1.8 + .18 + .018 + .0018 + cdots \
                                                                      &= (1 + .8) + (.1 + .08) + (.01 + .008) + (.001 + .0008) + cdots \
                                                                      &= 1 + (.8 + .1) + (.08 + .01) + (.008 + .001) + cdots \
                                                                      &= 1 + .9 + .09 + .009 + cdots \
                                                                      &= 1 + .999ldots
                                                                      end{align*}
                                                                      It follows subtracting $.999ldots$ from both sides that $.999ldots = 1$.



                                                                      The reason I like this explanation best is that addition of (positive) infinite decimal expansions (defined in a particular way) is both commutative and associative even if you insist that $.999ldots$ and $1$ are different numbers. That is, it forms a commutative monoid.
                                                                      But the cancellation property fails: if $a + b = a + c$, then we can't necessarily conclude $b = c$. The example of this is above, and the most fundamental reason why $.999ldots = 1$ is arguably so that the cancellation property can hold.





                                                                      $^{*}$The calculation given here (using rearrangmenet and regrouping of terms) is informal, and not intended to be a proof, but rather to give some idea of how you can add infinite decimal expansions in the monoid where $.999ldots ne 1$. It does end up being true that $.999ldots + .999ldots = 1 + .999ldots$ in this monoid.






                                                                      share|cite|improve this answer











                                                                      $endgroup$









                                                                      • 2




                                                                        $begingroup$
                                                                        It doesn't feel convincing to me to assume that blithely that you can rearrange the infinite sums the way you do here, without changing their value. It is true that it is safe in this particular case, but I don't see how one can show that except by developing enough of a theory of limits that we might as well just sum $sum_{n=1}^infty 9/10^n$ directly.
                                                                        $endgroup$
                                                                        – Henning Makholm
                                                                        Jul 17 '16 at 2:46












                                                                      • $begingroup$
                                                                        @HenningMakholm Right--it's not a convincing proof that $.999ldots = 1$ unless you accept that infinite sums of real numbers can be rearranged and regrouped in any way (which is true of course but much more nontrivial than the basic result that the limit of $.999ldots$ is $1$).
                                                                        $endgroup$
                                                                        – 6005
                                                                        Jul 17 '16 at 4:19












                                                                      • $begingroup$
                                                                        My actual point is that you can define the addition of infinite expansions even while you insist that $.999ldots ne 1$, and if you do so (although I don't give the definition) you get $.999ldots + .999ldots = 1 + .999ldots$. The aligned equation is just informal rearrangements but is intended to sort of show how the definition of addition of infinite decimal expansions would work, in this case at least.
                                                                        $endgroup$
                                                                        – 6005
                                                                        Jul 17 '16 at 4:20








                                                                      • 1




                                                                        $begingroup$
                                                                        It is not true in general that "infinite sums of real numbers can be rearranged and regrouped in any way". It works in this particular case because all of the terms being rearranged are all positive -- but in order to argue that this is safer than the general case (rearranging a conditionally convergent series such as $sum frac{(-1)^n}{n}$ fails spectacularly) it seems that you need to develop so much material that you cannot really keep open an option for $0.999...$ to differ from $1$.
                                                                        $endgroup$
                                                                        – Henning Makholm
                                                                        Jul 17 '16 at 9:12












                                                                      • $begingroup$
                                                                        @HenningMakholm Oh I'm sorry, that was a typo. Of course I'm aware. I meant positive real numbers.
                                                                        $endgroup$
                                                                        – 6005
                                                                        Jul 17 '16 at 14:43
















                                                                      3












                                                                      $begingroup$

                                                                      Here's my favorite reason why $.999ldots$ should equal $1$:$^{*}$
                                                                      begin{align*}
                                                                      .999ldots + .999ldots
                                                                      &= (.9 + .09 + .009 + cdots) + (.9 + .09 + .009 + cdots) \
                                                                      &= (.9 + .9) + (.09 + .09) + (.009 + .009) + cdots \
                                                                      &= 1.8 + .18 + .018 + .0018 + cdots \
                                                                      &= (1 + .8) + (.1 + .08) + (.01 + .008) + (.001 + .0008) + cdots \
                                                                      &= 1 + (.8 + .1) + (.08 + .01) + (.008 + .001) + cdots \
                                                                      &= 1 + .9 + .09 + .009 + cdots \
                                                                      &= 1 + .999ldots
                                                                      end{align*}
                                                                      It follows subtracting $.999ldots$ from both sides that $.999ldots = 1$.



                                                                      The reason I like this explanation best is that addition of (positive) infinite decimal expansions (defined in a particular way) is both commutative and associative even if you insist that $.999ldots$ and $1$ are different numbers. That is, it forms a commutative monoid.
                                                                      But the cancellation property fails: if $a + b = a + c$, then we can't necessarily conclude $b = c$. The example of this is above, and the most fundamental reason why $.999ldots = 1$ is arguably so that the cancellation property can hold.





                                                                      $^{*}$The calculation given here (using rearrangmenet and regrouping of terms) is informal, and not intended to be a proof, but rather to give some idea of how you can add infinite decimal expansions in the monoid where $.999ldots ne 1$. It does end up being true that $.999ldots + .999ldots = 1 + .999ldots$ in this monoid.






                                                                      share|cite|improve this answer











                                                                      $endgroup$









                                                                      • 2




                                                                        $begingroup$
                                                                        It doesn't feel convincing to me to assume that blithely that you can rearrange the infinite sums the way you do here, without changing their value. It is true that it is safe in this particular case, but I don't see how one can show that except by developing enough of a theory of limits that we might as well just sum $sum_{n=1}^infty 9/10^n$ directly.
                                                                        $endgroup$
                                                                        – Henning Makholm
                                                                        Jul 17 '16 at 2:46












                                                                      • $begingroup$
                                                                        @HenningMakholm Right--it's not a convincing proof that $.999ldots = 1$ unless you accept that infinite sums of real numbers can be rearranged and regrouped in any way (which is true of course but much more nontrivial than the basic result that the limit of $.999ldots$ is $1$).
                                                                        $endgroup$
                                                                        – 6005
                                                                        Jul 17 '16 at 4:19












                                                                      • $begingroup$
                                                                        My actual point is that you can define the addition of infinite expansions even while you insist that $.999ldots ne 1$, and if you do so (although I don't give the definition) you get $.999ldots + .999ldots = 1 + .999ldots$. The aligned equation is just informal rearrangements but is intended to sort of show how the definition of addition of infinite decimal expansions would work, in this case at least.
                                                                        $endgroup$
                                                                        – 6005
                                                                        Jul 17 '16 at 4:20








                                                                      • 1




                                                                        $begingroup$
                                                                        It is not true in general that "infinite sums of real numbers can be rearranged and regrouped in any way". It works in this particular case because all of the terms being rearranged are all positive -- but in order to argue that this is safer than the general case (rearranging a conditionally convergent series such as $sum frac{(-1)^n}{n}$ fails spectacularly) it seems that you need to develop so much material that you cannot really keep open an option for $0.999...$ to differ from $1$.
                                                                        $endgroup$
                                                                        – Henning Makholm
                                                                        Jul 17 '16 at 9:12












                                                                      • $begingroup$
                                                                        @HenningMakholm Oh I'm sorry, that was a typo. Of course I'm aware. I meant positive real numbers.
                                                                        $endgroup$
                                                                        – 6005
                                                                        Jul 17 '16 at 14:43














                                                                      3












                                                                      3








                                                                      3





                                                                      $begingroup$

                                                                      Here's my favorite reason why $.999ldots$ should equal $1$:$^{*}$
                                                                      begin{align*}
                                                                      .999ldots + .999ldots
                                                                      &= (.9 + .09 + .009 + cdots) + (.9 + .09 + .009 + cdots) \
                                                                      &= (.9 + .9) + (.09 + .09) + (.009 + .009) + cdots \
                                                                      &= 1.8 + .18 + .018 + .0018 + cdots \
                                                                      &= (1 + .8) + (.1 + .08) + (.01 + .008) + (.001 + .0008) + cdots \
                                                                      &= 1 + (.8 + .1) + (.08 + .01) + (.008 + .001) + cdots \
                                                                      &= 1 + .9 + .09 + .009 + cdots \
                                                                      &= 1 + .999ldots
                                                                      end{align*}
                                                                      It follows subtracting $.999ldots$ from both sides that $.999ldots = 1$.



                                                                      The reason I like this explanation best is that addition of (positive) infinite decimal expansions (defined in a particular way) is both commutative and associative even if you insist that $.999ldots$ and $1$ are different numbers. That is, it forms a commutative monoid.
                                                                      But the cancellation property fails: if $a + b = a + c$, then we can't necessarily conclude $b = c$. The example of this is above, and the most fundamental reason why $.999ldots = 1$ is arguably so that the cancellation property can hold.





                                                                      $^{*}$The calculation given here (using rearrangmenet and regrouping of terms) is informal, and not intended to be a proof, but rather to give some idea of how you can add infinite decimal expansions in the monoid where $.999ldots ne 1$. It does end up being true that $.999ldots + .999ldots = 1 + .999ldots$ in this monoid.






                                                                      share|cite|improve this answer











                                                                      $endgroup$



                                                                      Here's my favorite reason why $.999ldots$ should equal $1$:$^{*}$
                                                                      begin{align*}
                                                                      .999ldots + .999ldots
                                                                      &= (.9 + .09 + .009 + cdots) + (.9 + .09 + .009 + cdots) \
                                                                      &= (.9 + .9) + (.09 + .09) + (.009 + .009) + cdots \
                                                                      &= 1.8 + .18 + .018 + .0018 + cdots \
                                                                      &= (1 + .8) + (.1 + .08) + (.01 + .008) + (.001 + .0008) + cdots \
                                                                      &= 1 + (.8 + .1) + (.08 + .01) + (.008 + .001) + cdots \
                                                                      &= 1 + .9 + .09 + .009 + cdots \
                                                                      &= 1 + .999ldots
                                                                      end{align*}
                                                                      It follows subtracting $.999ldots$ from both sides that $.999ldots = 1$.



                                                                      The reason I like this explanation best is that addition of (positive) infinite decimal expansions (defined in a particular way) is both commutative and associative even if you insist that $.999ldots$ and $1$ are different numbers. That is, it forms a commutative monoid.
                                                                      But the cancellation property fails: if $a + b = a + c$, then we can't necessarily conclude $b = c$. The example of this is above, and the most fundamental reason why $.999ldots = 1$ is arguably so that the cancellation property can hold.





                                                                      $^{*}$The calculation given here (using rearrangmenet and regrouping of terms) is informal, and not intended to be a proof, but rather to give some idea of how you can add infinite decimal expansions in the monoid where $.999ldots ne 1$. It does end up being true that $.999ldots + .999ldots = 1 + .999ldots$ in this monoid.







                                                                      share|cite|improve this answer














                                                                      share|cite|improve this answer



                                                                      share|cite|improve this answer








                                                                      edited Aug 4 '16 at 13:18


























                                                                      community wiki





                                                                      3 revs
                                                                      6005









                                                                      • 2




                                                                        $begingroup$
                                                                        It doesn't feel convincing to me to assume that blithely that you can rearrange the infinite sums the way you do here, without changing their value. It is true that it is safe in this particular case, but I don't see how one can show that except by developing enough of a theory of limits that we might as well just sum $sum_{n=1}^infty 9/10^n$ directly.
                                                                        $endgroup$
                                                                        – Henning Makholm
                                                                        Jul 17 '16 at 2:46












                                                                      • $begingroup$
                                                                        @HenningMakholm Right--it's not a convincing proof that $.999ldots = 1$ unless you accept that infinite sums of real numbers can be rearranged and regrouped in any way (which is true of course but much more nontrivial than the basic result that the limit of $.999ldots$ is $1$).
                                                                        $endgroup$
                                                                        – 6005
                                                                        Jul 17 '16 at 4:19












                                                                      • $begingroup$
                                                                        My actual point is that you can define the addition of infinite expansions even while you insist that $.999ldots ne 1$, and if you do so (although I don't give the definition) you get $.999ldots + .999ldots = 1 + .999ldots$. The aligned equation is just informal rearrangements but is intended to sort of show how the definition of addition of infinite decimal expansions would work, in this case at least.
                                                                        $endgroup$
                                                                        – 6005
                                                                        Jul 17 '16 at 4:20








                                                                      • 1




                                                                        $begingroup$
                                                                        It is not true in general that "infinite sums of real numbers can be rearranged and regrouped in any way". It works in this particular case because all of the terms being rearranged are all positive -- but in order to argue that this is safer than the general case (rearranging a conditionally convergent series such as $sum frac{(-1)^n}{n}$ fails spectacularly) it seems that you need to develop so much material that you cannot really keep open an option for $0.999...$ to differ from $1$.
                                                                        $endgroup$
                                                                        – Henning Makholm
                                                                        Jul 17 '16 at 9:12












                                                                      • $begingroup$
                                                                        @HenningMakholm Oh I'm sorry, that was a typo. Of course I'm aware. I meant positive real numbers.
                                                                        $endgroup$
                                                                        – 6005
                                                                        Jul 17 '16 at 14:43














                                                                      • 2




                                                                        $begingroup$
                                                                        It doesn't feel convincing to me to assume that blithely that you can rearrange the infinite sums the way you do here, without changing their value. It is true that it is safe in this particular case, but I don't see how one can show that except by developing enough of a theory of limits that we might as well just sum $sum_{n=1}^infty 9/10^n$ directly.
                                                                        $endgroup$
                                                                        – Henning Makholm
                                                                        Jul 17 '16 at 2:46












                                                                      • $begingroup$
                                                                        @HenningMakholm Right--it's not a convincing proof that $.999ldots = 1$ unless you accept that infinite sums of real numbers can be rearranged and regrouped in any way (which is true of course but much more nontrivial than the basic result that the limit of $.999ldots$ is $1$).
                                                                        $endgroup$
                                                                        – 6005
                                                                        Jul 17 '16 at 4:19












                                                                      • $begingroup$
                                                                        My actual point is that you can define the addition of infinite expansions even while you insist that $.999ldots ne 1$, and if you do so (although I don't give the definition) you get $.999ldots + .999ldots = 1 + .999ldots$. The aligned equation is just informal rearrangements but is intended to sort of show how the definition of addition of infinite decimal expansions would work, in this case at least.
                                                                        $endgroup$
                                                                        – 6005
                                                                        Jul 17 '16 at 4:20








                                                                      • 1




                                                                        $begingroup$
                                                                        It is not true in general that "infinite sums of real numbers can be rearranged and regrouped in any way". It works in this particular case because all of the terms being rearranged are all positive -- but in order to argue that this is safer than the general case (rearranging a conditionally convergent series such as $sum frac{(-1)^n}{n}$ fails spectacularly) it seems that you need to develop so much material that you cannot really keep open an option for $0.999...$ to differ from $1$.
                                                                        $endgroup$
                                                                        – Henning Makholm
                                                                        Jul 17 '16 at 9:12












                                                                      • $begingroup$
                                                                        @HenningMakholm Oh I'm sorry, that was a typo. Of course I'm aware. I meant positive real numbers.
                                                                        $endgroup$
                                                                        – 6005
                                                                        Jul 17 '16 at 14:43








                                                                      2




                                                                      2




                                                                      $begingroup$
                                                                      It doesn't feel convincing to me to assume that blithely that you can rearrange the infinite sums the way you do here, without changing their value. It is true that it is safe in this particular case, but I don't see how one can show that except by developing enough of a theory of limits that we might as well just sum $sum_{n=1}^infty 9/10^n$ directly.
                                                                      $endgroup$
                                                                      – Henning Makholm
                                                                      Jul 17 '16 at 2:46






                                                                      $begingroup$
                                                                      It doesn't feel convincing to me to assume that blithely that you can rearrange the infinite sums the way you do here, without changing their value. It is true that it is safe in this particular case, but I don't see how one can show that except by developing enough of a theory of limits that we might as well just sum $sum_{n=1}^infty 9/10^n$ directly.
                                                                      $endgroup$
                                                                      – Henning Makholm
                                                                      Jul 17 '16 at 2:46














                                                                      $begingroup$
                                                                      @HenningMakholm Right--it's not a convincing proof that $.999ldots = 1$ unless you accept that infinite sums of real numbers can be rearranged and regrouped in any way (which is true of course but much more nontrivial than the basic result that the limit of $.999ldots$ is $1$).
                                                                      $endgroup$
                                                                      – 6005
                                                                      Jul 17 '16 at 4:19






                                                                      $begingroup$
                                                                      @HenningMakholm Right--it's not a convincing proof that $.999ldots = 1$ unless you accept that infinite sums of real numbers can be rearranged and regrouped in any way (which is true of course but much more nontrivial than the basic result that the limit of $.999ldots$ is $1$).
                                                                      $endgroup$
                                                                      – 6005
                                                                      Jul 17 '16 at 4:19














                                                                      $begingroup$
                                                                      My actual point is that you can define the addition of infinite expansions even while you insist that $.999ldots ne 1$, and if you do so (although I don't give the definition) you get $.999ldots + .999ldots = 1 + .999ldots$. The aligned equation is just informal rearrangements but is intended to sort of show how the definition of addition of infinite decimal expansions would work, in this case at least.
                                                                      $endgroup$
                                                                      – 6005
                                                                      Jul 17 '16 at 4:20






                                                                      $begingroup$
                                                                      My actual point is that you can define the addition of infinite expansions even while you insist that $.999ldots ne 1$, and if you do so (although I don't give the definition) you get $.999ldots + .999ldots = 1 + .999ldots$. The aligned equation is just informal rearrangements but is intended to sort of show how the definition of addition of infinite decimal expansions would work, in this case at least.
                                                                      $endgroup$
                                                                      – 6005
                                                                      Jul 17 '16 at 4:20






                                                                      1




                                                                      1




                                                                      $begingroup$
                                                                      It is not true in general that "infinite sums of real numbers can be rearranged and regrouped in any way". It works in this particular case because all of the terms being rearranged are all positive -- but in order to argue that this is safer than the general case (rearranging a conditionally convergent series such as $sum frac{(-1)^n}{n}$ fails spectacularly) it seems that you need to develop so much material that you cannot really keep open an option for $0.999...$ to differ from $1$.
                                                                      $endgroup$
                                                                      – Henning Makholm
                                                                      Jul 17 '16 at 9:12






                                                                      $begingroup$
                                                                      It is not true in general that "infinite sums of real numbers can be rearranged and regrouped in any way". It works in this particular case because all of the terms being rearranged are all positive -- but in order to argue that this is safer than the general case (rearranging a conditionally convergent series such as $sum frac{(-1)^n}{n}$ fails spectacularly) it seems that you need to develop so much material that you cannot really keep open an option for $0.999...$ to differ from $1$.
                                                                      $endgroup$
                                                                      – Henning Makholm
                                                                      Jul 17 '16 at 9:12














                                                                      $begingroup$
                                                                      @HenningMakholm Oh I'm sorry, that was a typo. Of course I'm aware. I meant positive real numbers.
                                                                      $endgroup$
                                                                      – 6005
                                                                      Jul 17 '16 at 14:43




                                                                      $begingroup$
                                                                      @HenningMakholm Oh I'm sorry, that was a typo. Of course I'm aware. I meant positive real numbers.
                                                                      $endgroup$
                                                                      – 6005
                                                                      Jul 17 '16 at 14:43











                                                                      3












                                                                      $begingroup$

                                                                      Use the Squeeze Theorem:



                                                                      $$0<1-0.999...=0.1+0.9-0.999...=0.1-0.0999...<0.1=0.1^1$$
                                                                      $$0<0.1-0.0999...=0.01+0.09-0.0999...=0.01-0.00999...<0.01=0.1^2$$
                                                                      $$...$$
                                                                      $$0<1-0.999...<0.1^n$$
                                                                      $$0le 1-0.999... le limlimits_{ntoinfty}0.1^n=0.$$






                                                                      share|cite|improve this answer











                                                                      $endgroup$


















                                                                        3












                                                                        $begingroup$

                                                                        Use the Squeeze Theorem:



                                                                        $$0<1-0.999...=0.1+0.9-0.999...=0.1-0.0999...<0.1=0.1^1$$
                                                                        $$0<0.1-0.0999...=0.01+0.09-0.0999...=0.01-0.00999...<0.01=0.1^2$$
                                                                        $$...$$
                                                                        $$0<1-0.999...<0.1^n$$
                                                                        $$0le 1-0.999... le limlimits_{ntoinfty}0.1^n=0.$$






                                                                        share|cite|improve this answer











                                                                        $endgroup$
















                                                                          3












                                                                          3








                                                                          3





                                                                          $begingroup$

                                                                          Use the Squeeze Theorem:



                                                                          $$0<1-0.999...=0.1+0.9-0.999...=0.1-0.0999...<0.1=0.1^1$$
                                                                          $$0<0.1-0.0999...=0.01+0.09-0.0999...=0.01-0.00999...<0.01=0.1^2$$
                                                                          $$...$$
                                                                          $$0<1-0.999...<0.1^n$$
                                                                          $$0le 1-0.999... le limlimits_{ntoinfty}0.1^n=0.$$






                                                                          share|cite|improve this answer











                                                                          $endgroup$



                                                                          Use the Squeeze Theorem:



                                                                          $$0<1-0.999...=0.1+0.9-0.999...=0.1-0.0999...<0.1=0.1^1$$
                                                                          $$0<0.1-0.0999...=0.01+0.09-0.0999...=0.01-0.00999...<0.01=0.1^2$$
                                                                          $$...$$
                                                                          $$0<1-0.999...<0.1^n$$
                                                                          $$0le 1-0.999... le limlimits_{ntoinfty}0.1^n=0.$$







                                                                          share|cite|improve this answer














                                                                          share|cite|improve this answer



                                                                          share|cite|improve this answer








                                                                          answered May 25 '17 at 12:28


























                                                                          community wiki





                                                                          farruhota
























                                                                              1












                                                                              $begingroup$

                                                                              If we take a version of decimal notation with full complement it is indeed so.



                                                                              In this system, instead of allowing $0.2$ and $0.1999...$ we restrict decimal notation to use only infinite version $0.19999...$ shortly written as $0.1overline{9}$



                                                                              Simply, we do not allow an infinite trail of zeros.



                                                                              In this system, there is no $0$ written as $0.000...$ instead it is $...999.999...$ or with our succinct notation $overline{9}.overline{9}$



                                                                              Negative numbers are written in complement notation. For example, $...998.999...=overline{9}8.overline{9}=-1$



                                                                              All rules of multiplication addition subtraction are totally valid.



                                                                              In this system, it is indeed $0.99999...=0.overline{9}=1$ because we cannot represent $1$ as $1.0000...$.






                                                                              share|cite|improve this answer











                                                                              $endgroup$


















                                                                                1












                                                                                $begingroup$

                                                                                If we take a version of decimal notation with full complement it is indeed so.



                                                                                In this system, instead of allowing $0.2$ and $0.1999...$ we restrict decimal notation to use only infinite version $0.19999...$ shortly written as $0.1overline{9}$



                                                                                Simply, we do not allow an infinite trail of zeros.



                                                                                In this system, there is no $0$ written as $0.000...$ instead it is $...999.999...$ or with our succinct notation $overline{9}.overline{9}$



                                                                                Negative numbers are written in complement notation. For example, $...998.999...=overline{9}8.overline{9}=-1$



                                                                                All rules of multiplication addition subtraction are totally valid.



                                                                                In this system, it is indeed $0.99999...=0.overline{9}=1$ because we cannot represent $1$ as $1.0000...$.






                                                                                share|cite|improve this answer











                                                                                $endgroup$
















                                                                                  1












                                                                                  1








                                                                                  1





                                                                                  $begingroup$

                                                                                  If we take a version of decimal notation with full complement it is indeed so.



                                                                                  In this system, instead of allowing $0.2$ and $0.1999...$ we restrict decimal notation to use only infinite version $0.19999...$ shortly written as $0.1overline{9}$



                                                                                  Simply, we do not allow an infinite trail of zeros.



                                                                                  In this system, there is no $0$ written as $0.000...$ instead it is $...999.999...$ or with our succinct notation $overline{9}.overline{9}$



                                                                                  Negative numbers are written in complement notation. For example, $...998.999...=overline{9}8.overline{9}=-1$



                                                                                  All rules of multiplication addition subtraction are totally valid.



                                                                                  In this system, it is indeed $0.99999...=0.overline{9}=1$ because we cannot represent $1$ as $1.0000...$.






                                                                                  share|cite|improve this answer











                                                                                  $endgroup$



                                                                                  If we take a version of decimal notation with full complement it is indeed so.



                                                                                  In this system, instead of allowing $0.2$ and $0.1999...$ we restrict decimal notation to use only infinite version $0.19999...$ shortly written as $0.1overline{9}$



                                                                                  Simply, we do not allow an infinite trail of zeros.



                                                                                  In this system, there is no $0$ written as $0.000...$ instead it is $...999.999...$ or with our succinct notation $overline{9}.overline{9}$



                                                                                  Negative numbers are written in complement notation. For example, $...998.999...=overline{9}8.overline{9}=-1$



                                                                                  All rules of multiplication addition subtraction are totally valid.



                                                                                  In this system, it is indeed $0.99999...=0.overline{9}=1$ because we cannot represent $1$ as $1.0000...$.







                                                                                  share|cite|improve this answer














                                                                                  share|cite|improve this answer



                                                                                  share|cite|improve this answer








                                                                                  edited Apr 10 '18 at 21:13


























                                                                                  community wiki





                                                                                  11 revs
                                                                                  user318107
























                                                                                      0












                                                                                      $begingroup$

                                                                                      If you allow a "decimal representation" of a number to end finally with period $9$ as in $0.bar9$ or $1.123bar9$ this "decimal representation "of a number would not be unique.



                                                                                      We know by definition that $0.bar9=sum_{n=1}^{infty}(9/10)^n=1$, but for the sake of the uniquenes of the decimal representation $0.bar9$ is not a decimal representation of any number.






                                                                                      share|cite|improve this answer











                                                                                      $endgroup$


















                                                                                        0












                                                                                        $begingroup$

                                                                                        If you allow a "decimal representation" of a number to end finally with period $9$ as in $0.bar9$ or $1.123bar9$ this "decimal representation "of a number would not be unique.



                                                                                        We know by definition that $0.bar9=sum_{n=1}^{infty}(9/10)^n=1$, but for the sake of the uniquenes of the decimal representation $0.bar9$ is not a decimal representation of any number.






                                                                                        share|cite|improve this answer











                                                                                        $endgroup$
















                                                                                          0












                                                                                          0








                                                                                          0





                                                                                          $begingroup$

                                                                                          If you allow a "decimal representation" of a number to end finally with period $9$ as in $0.bar9$ or $1.123bar9$ this "decimal representation "of a number would not be unique.



                                                                                          We know by definition that $0.bar9=sum_{n=1}^{infty}(9/10)^n=1$, but for the sake of the uniquenes of the decimal representation $0.bar9$ is not a decimal representation of any number.






                                                                                          share|cite|improve this answer











                                                                                          $endgroup$



                                                                                          If you allow a "decimal representation" of a number to end finally with period $9$ as in $0.bar9$ or $1.123bar9$ this "decimal representation "of a number would not be unique.



                                                                                          We know by definition that $0.bar9=sum_{n=1}^{infty}(9/10)^n=1$, but for the sake of the uniquenes of the decimal representation $0.bar9$ is not a decimal representation of any number.







                                                                                          share|cite|improve this answer














                                                                                          share|cite|improve this answer



                                                                                          share|cite|improve this answer








                                                                                          edited Nov 7 '18 at 17:16


























                                                                                          community wiki





                                                                                          2 revs
                                                                                          Michael Hoppe
























                                                                                              0












                                                                                              $begingroup$

                                                                                              Edit: I totally rewrote this answer in a completely different way than before. I decided to write it as another answer instead of undeleting my existing answer because it was posted as Community Wiki and Community Wiki answers are supposed to be pretty infrequent.



                                                                                              Edit: I see that it ended up being Community Wiki anyway without me purposely making it that way. Maybe the previous answer was marked as Community Wiki to limit the number of questions I answer not as Community Wiki because some of my answers weren't that great so it would have been better for me not to write another answer even if I could not undelete my other answer because it was Community Wiki.



                                                                                              Some people claim that $(mathbb{R}, 1, 0, +, times, leq)$ is a complete ordered field from which we can deduce that 0.999... = 1. Now how to we know that it's a complete ordered field? Because we can construct a complete ordered field in ZF and show that it's a complete ordered field which is unique up to isomorphism. First, we construct the natural numbers as finite ordinal numbers and define +, $times$, and $leq$ on them. 0 is not the successor of any number so we invent the number -1. -1 is still not the successor of any number so we invent the number -2 and keep going to get the integers and then redefine +, $times$, and $leq$ on them in the intuitive way. Now each odd number $x$ is not a solution to $2 times y = x$ so we invent a number $y$ to be the solution. Let's call those newly invented numbers the half integers. Now each half integer $y$ is still not a solution to $2 times z = y$ so we invent solutions to them and keep going forever to get the dyadic rationals and again redefine +, $times$, and $leq$ on them. Take any subset of the set of dyadic rationals such that that set and its complement are nonempty and for any dyadic rational in the subset, all smaller dyadic rationals are in the subset. Some of them have the additional property that they have no maximal element nor does their complement have a minimal element. For each one where that's the case, we can invent a number that's larger than all dyadic rationals in the subset and smaller than all dyadic rationals in its complement. We can call the set of all real numbers the set of all numbers constructed in this way and then redefine +, $times$, and $leq$ on that set in the intuitive way and show that $(mathbb{R}, text{{{}}}, text{{}}, +, times, leq)$ is a complete ordered field.



                                                                                              Some people might not want to use the fact that there exists a complete ordered field which is unique up to isomprphism. Some might prefer to use the fact that there exists a set with operations satisfying a different property which is unique up to isomorphism as I will describe. Take any totally ordered field with the property that if you take any subset of it such that it's nonempty and its complement is nonempty and for any number in the subset, all smaller numbers in the set and you can take a number in the set and a number not in the set arbitrarily close, then either the set has a maximal element or its complement has a minimal element. The real number system satisfies that property but so does the hyperreal number system. Some people might instead want to use the fact that if you add the criterion that there are no infinite numbers in the system, there is a set with operations satisfying those criteria which is unique up to isomporphism. It turns out that given the criteria for a hyperreal system I described, the system is complete if and only if it has no infinite numbers. Any set with those operations satisfying one set of criteria satisfies the other set of criteria so we don't need to disagree on the structure of the real numbers.



                                                                                              I've shown only how to construct the real numbers themselves. Inventing a decimal notation for all of them is another thing. It can be show that in that system all Cauchy sequences approach a real number. A decimal notation can be defined as a limit and since the limit exists for all notations, all notations represent a number. It can also be shown that all real numbers have a decimal notation according to that definition and some of them have 2 notations according to that definition. I read that some authors forbid a notation with a string of trailing 9's. You can also construct a set with +, $times$, and $leq$ a similar way to the way I just did except that you replace the number 2 with 10 and show that it's a complete ordered field and therefore isomorphic to the one I constructed. That also gives an intuitive way to define a decimal notation for each real number but it defines only one notation for each number and forbids a string of trailing 9's. It can also be shown that this notation for each real number is also one of the notations using the other definition of a decimal notation.






                                                                                              share|cite|improve this answer











                                                                                              $endgroup$













                                                                                              • $begingroup$
                                                                                                I didn't click the Community Wiki box. Why is this answer Community Wiki? Is it because there was no box this time because it's better for me to post it as Community Wiki because I'm not that great at answering questions?
                                                                                                $endgroup$
                                                                                                – Timothy
                                                                                                Dec 23 '18 at 5:49






                                                                                              • 1




                                                                                                $begingroup$
                                                                                                I think it might be because the question is now set to Community Wiki—I've just checked through and every answer is set that way (including mine which I've been thinking about deleting).
                                                                                                $endgroup$
                                                                                                – timtfj
                                                                                                Dec 25 '18 at 1:07


















                                                                                              0












                                                                                              $begingroup$

                                                                                              Edit: I totally rewrote this answer in a completely different way than before. I decided to write it as another answer instead of undeleting my existing answer because it was posted as Community Wiki and Community Wiki answers are supposed to be pretty infrequent.



                                                                                              Edit: I see that it ended up being Community Wiki anyway without me purposely making it that way. Maybe the previous answer was marked as Community Wiki to limit the number of questions I answer not as Community Wiki because some of my answers weren't that great so it would have been better for me not to write another answer even if I could not undelete my other answer because it was Community Wiki.



                                                                                              Some people claim that $(mathbb{R}, 1, 0, +, times, leq)$ is a complete ordered field from which we can deduce that 0.999... = 1. Now how to we know that it's a complete ordered field? Because we can construct a complete ordered field in ZF and show that it's a complete ordered field which is unique up to isomorphism. First, we construct the natural numbers as finite ordinal numbers and define +, $times$, and $leq$ on them. 0 is not the successor of any number so we invent the number -1. -1 is still not the successor of any number so we invent the number -2 and keep going to get the integers and then redefine +, $times$, and $leq$ on them in the intuitive way. Now each odd number $x$ is not a solution to $2 times y = x$ so we invent a number $y$ to be the solution. Let's call those newly invented numbers the half integers. Now each half integer $y$ is still not a solution to $2 times z = y$ so we invent solutions to them and keep going forever to get the dyadic rationals and again redefine +, $times$, and $leq$ on them. Take any subset of the set of dyadic rationals such that that set and its complement are nonempty and for any dyadic rational in the subset, all smaller dyadic rationals are in the subset. Some of them have the additional property that they have no maximal element nor does their complement have a minimal element. For each one where that's the case, we can invent a number that's larger than all dyadic rationals in the subset and smaller than all dyadic rationals in its complement. We can call the set of all real numbers the set of all numbers constructed in this way and then redefine +, $times$, and $leq$ on that set in the intuitive way and show that $(mathbb{R}, text{{{}}}, text{{}}, +, times, leq)$ is a complete ordered field.



                                                                                              Some people might not want to use the fact that there exists a complete ordered field which is unique up to isomprphism. Some might prefer to use the fact that there exists a set with operations satisfying a different property which is unique up to isomorphism as I will describe. Take any totally ordered field with the property that if you take any subset of it such that it's nonempty and its complement is nonempty and for any number in the subset, all smaller numbers in the set and you can take a number in the set and a number not in the set arbitrarily close, then either the set has a maximal element or its complement has a minimal element. The real number system satisfies that property but so does the hyperreal number system. Some people might instead want to use the fact that if you add the criterion that there are no infinite numbers in the system, there is a set with operations satisfying those criteria which is unique up to isomporphism. It turns out that given the criteria for a hyperreal system I described, the system is complete if and only if it has no infinite numbers. Any set with those operations satisfying one set of criteria satisfies the other set of criteria so we don't need to disagree on the structure of the real numbers.



                                                                                              I've shown only how to construct the real numbers themselves. Inventing a decimal notation for all of them is another thing. It can be show that in that system all Cauchy sequences approach a real number. A decimal notation can be defined as a limit and since the limit exists for all notations, all notations represent a number. It can also be shown that all real numbers have a decimal notation according to that definition and some of them have 2 notations according to that definition. I read that some authors forbid a notation with a string of trailing 9's. You can also construct a set with +, $times$, and $leq$ a similar way to the way I just did except that you replace the number 2 with 10 and show that it's a complete ordered field and therefore isomorphic to the one I constructed. That also gives an intuitive way to define a decimal notation for each real number but it defines only one notation for each number and forbids a string of trailing 9's. It can also be shown that this notation for each real number is also one of the notations using the other definition of a decimal notation.






                                                                                              share|cite|improve this answer











                                                                                              $endgroup$













                                                                                              • $begingroup$
                                                                                                I didn't click the Community Wiki box. Why is this answer Community Wiki? Is it because there was no box this time because it's better for me to post it as Community Wiki because I'm not that great at answering questions?
                                                                                                $endgroup$
                                                                                                – Timothy
                                                                                                Dec 23 '18 at 5:49






                                                                                              • 1




                                                                                                $begingroup$
                                                                                                I think it might be because the question is now set to Community Wiki—I've just checked through and every answer is set that way (including mine which I've been thinking about deleting).
                                                                                                $endgroup$
                                                                                                – timtfj
                                                                                                Dec 25 '18 at 1:07
















                                                                                              0












                                                                                              0








                                                                                              0





                                                                                              $begingroup$

                                                                                              Edit: I totally rewrote this answer in a completely different way than before. I decided to write it as another answer instead of undeleting my existing answer because it was posted as Community Wiki and Community Wiki answers are supposed to be pretty infrequent.



                                                                                              Edit: I see that it ended up being Community Wiki anyway without me purposely making it that way. Maybe the previous answer was marked as Community Wiki to limit the number of questions I answer not as Community Wiki because some of my answers weren't that great so it would have been better for me not to write another answer even if I could not undelete my other answer because it was Community Wiki.



                                                                                              Some people claim that $(mathbb{R}, 1, 0, +, times, leq)$ is a complete ordered field from which we can deduce that 0.999... = 1. Now how to we know that it's a complete ordered field? Because we can construct a complete ordered field in ZF and show that it's a complete ordered field which is unique up to isomorphism. First, we construct the natural numbers as finite ordinal numbers and define +, $times$, and $leq$ on them. 0 is not the successor of any number so we invent the number -1. -1 is still not the successor of any number so we invent the number -2 and keep going to get the integers and then redefine +, $times$, and $leq$ on them in the intuitive way. Now each odd number $x$ is not a solution to $2 times y = x$ so we invent a number $y$ to be the solution. Let's call those newly invented numbers the half integers. Now each half integer $y$ is still not a solution to $2 times z = y$ so we invent solutions to them and keep going forever to get the dyadic rationals and again redefine +, $times$, and $leq$ on them. Take any subset of the set of dyadic rationals such that that set and its complement are nonempty and for any dyadic rational in the subset, all smaller dyadic rationals are in the subset. Some of them have the additional property that they have no maximal element nor does their complement have a minimal element. For each one where that's the case, we can invent a number that's larger than all dyadic rationals in the subset and smaller than all dyadic rationals in its complement. We can call the set of all real numbers the set of all numbers constructed in this way and then redefine +, $times$, and $leq$ on that set in the intuitive way and show that $(mathbb{R}, text{{{}}}, text{{}}, +, times, leq)$ is a complete ordered field.



                                                                                              Some people might not want to use the fact that there exists a complete ordered field which is unique up to isomprphism. Some might prefer to use the fact that there exists a set with operations satisfying a different property which is unique up to isomorphism as I will describe. Take any totally ordered field with the property that if you take any subset of it such that it's nonempty and its complement is nonempty and for any number in the subset, all smaller numbers in the set and you can take a number in the set and a number not in the set arbitrarily close, then either the set has a maximal element or its complement has a minimal element. The real number system satisfies that property but so does the hyperreal number system. Some people might instead want to use the fact that if you add the criterion that there are no infinite numbers in the system, there is a set with operations satisfying those criteria which is unique up to isomporphism. It turns out that given the criteria for a hyperreal system I described, the system is complete if and only if it has no infinite numbers. Any set with those operations satisfying one set of criteria satisfies the other set of criteria so we don't need to disagree on the structure of the real numbers.



                                                                                              I've shown only how to construct the real numbers themselves. Inventing a decimal notation for all of them is another thing. It can be show that in that system all Cauchy sequences approach a real number. A decimal notation can be defined as a limit and since the limit exists for all notations, all notations represent a number. It can also be shown that all real numbers have a decimal notation according to that definition and some of them have 2 notations according to that definition. I read that some authors forbid a notation with a string of trailing 9's. You can also construct a set with +, $times$, and $leq$ a similar way to the way I just did except that you replace the number 2 with 10 and show that it's a complete ordered field and therefore isomorphic to the one I constructed. That also gives an intuitive way to define a decimal notation for each real number but it defines only one notation for each number and forbids a string of trailing 9's. It can also be shown that this notation for each real number is also one of the notations using the other definition of a decimal notation.






                                                                                              share|cite|improve this answer











                                                                                              $endgroup$



                                                                                              Edit: I totally rewrote this answer in a completely different way than before. I decided to write it as another answer instead of undeleting my existing answer because it was posted as Community Wiki and Community Wiki answers are supposed to be pretty infrequent.



                                                                                              Edit: I see that it ended up being Community Wiki anyway without me purposely making it that way. Maybe the previous answer was marked as Community Wiki to limit the number of questions I answer not as Community Wiki because some of my answers weren't that great so it would have been better for me not to write another answer even if I could not undelete my other answer because it was Community Wiki.



                                                                                              Some people claim that $(mathbb{R}, 1, 0, +, times, leq)$ is a complete ordered field from which we can deduce that 0.999... = 1. Now how to we know that it's a complete ordered field? Because we can construct a complete ordered field in ZF and show that it's a complete ordered field which is unique up to isomorphism. First, we construct the natural numbers as finite ordinal numbers and define +, $times$, and $leq$ on them. 0 is not the successor of any number so we invent the number -1. -1 is still not the successor of any number so we invent the number -2 and keep going to get the integers and then redefine +, $times$, and $leq$ on them in the intuitive way. Now each odd number $x$ is not a solution to $2 times y = x$ so we invent a number $y$ to be the solution. Let's call those newly invented numbers the half integers. Now each half integer $y$ is still not a solution to $2 times z = y$ so we invent solutions to them and keep going forever to get the dyadic rationals and again redefine +, $times$, and $leq$ on them. Take any subset of the set of dyadic rationals such that that set and its complement are nonempty and for any dyadic rational in the subset, all smaller dyadic rationals are in the subset. Some of them have the additional property that they have no maximal element nor does their complement have a minimal element. For each one where that's the case, we can invent a number that's larger than all dyadic rationals in the subset and smaller than all dyadic rationals in its complement. We can call the set of all real numbers the set of all numbers constructed in this way and then redefine +, $times$, and $leq$ on that set in the intuitive way and show that $(mathbb{R}, text{{{}}}, text{{}}, +, times, leq)$ is a complete ordered field.



                                                                                              Some people might not want to use the fact that there exists a complete ordered field which is unique up to isomprphism. Some might prefer to use the fact that there exists a set with operations satisfying a different property which is unique up to isomorphism as I will describe. Take any totally ordered field with the property that if you take any subset of it such that it's nonempty and its complement is nonempty and for any number in the subset, all smaller numbers in the set and you can take a number in the set and a number not in the set arbitrarily close, then either the set has a maximal element or its complement has a minimal element. The real number system satisfies that property but so does the hyperreal number system. Some people might instead want to use the fact that if you add the criterion that there are no infinite numbers in the system, there is a set with operations satisfying those criteria which is unique up to isomporphism. It turns out that given the criteria for a hyperreal system I described, the system is complete if and only if it has no infinite numbers. Any set with those operations satisfying one set of criteria satisfies the other set of criteria so we don't need to disagree on the structure of the real numbers.



                                                                                              I've shown only how to construct the real numbers themselves. Inventing a decimal notation for all of them is another thing. It can be show that in that system all Cauchy sequences approach a real number. A decimal notation can be defined as a limit and since the limit exists for all notations, all notations represent a number. It can also be shown that all real numbers have a decimal notation according to that definition and some of them have 2 notations according to that definition. I read that some authors forbid a notation with a string of trailing 9's. You can also construct a set with +, $times$, and $leq$ a similar way to the way I just did except that you replace the number 2 with 10 and show that it's a complete ordered field and therefore isomorphic to the one I constructed. That also gives an intuitive way to define a decimal notation for each real number but it defines only one notation for each number and forbids a string of trailing 9's. It can also be shown that this notation for each real number is also one of the notations using the other definition of a decimal notation.







                                                                                              share|cite|improve this answer














                                                                                              share|cite|improve this answer



                                                                                              share|cite|improve this answer








                                                                                              edited Dec 23 '18 at 6:00


























                                                                                              community wiki





                                                                                              2 revs
                                                                                              Timothy













                                                                                              • $begingroup$
                                                                                                I didn't click the Community Wiki box. Why is this answer Community Wiki? Is it because there was no box this time because it's better for me to post it as Community Wiki because I'm not that great at answering questions?
                                                                                                $endgroup$
                                                                                                – Timothy
                                                                                                Dec 23 '18 at 5:49






                                                                                              • 1




                                                                                                $begingroup$
                                                                                                I think it might be because the question is now set to Community Wiki—I've just checked through and every answer is set that way (including mine which I've been thinking about deleting).
                                                                                                $endgroup$
                                                                                                – timtfj
                                                                                                Dec 25 '18 at 1:07




















                                                                                              • $begingroup$
                                                                                                I didn't click the Community Wiki box. Why is this answer Community Wiki? Is it because there was no box this time because it's better for me to post it as Community Wiki because I'm not that great at answering questions?
                                                                                                $endgroup$
                                                                                                – Timothy
                                                                                                Dec 23 '18 at 5:49






                                                                                              • 1




                                                                                                $begingroup$
                                                                                                I think it might be because the question is now set to Community Wiki—I've just checked through and every answer is set that way (including mine which I've been thinking about deleting).
                                                                                                $endgroup$
                                                                                                – timtfj
                                                                                                Dec 25 '18 at 1:07


















                                                                                              $begingroup$
                                                                                              I didn't click the Community Wiki box. Why is this answer Community Wiki? Is it because there was no box this time because it's better for me to post it as Community Wiki because I'm not that great at answering questions?
                                                                                              $endgroup$
                                                                                              – Timothy
                                                                                              Dec 23 '18 at 5:49




                                                                                              $begingroup$
                                                                                              I didn't click the Community Wiki box. Why is this answer Community Wiki? Is it because there was no box this time because it's better for me to post it as Community Wiki because I'm not that great at answering questions?
                                                                                              $endgroup$
                                                                                              – Timothy
                                                                                              Dec 23 '18 at 5:49




                                                                                              1




                                                                                              1




                                                                                              $begingroup$
                                                                                              I think it might be because the question is now set to Community Wiki—I've just checked through and every answer is set that way (including mine which I've been thinking about deleting).
                                                                                              $endgroup$
                                                                                              – timtfj
                                                                                              Dec 25 '18 at 1:07






                                                                                              $begingroup$
                                                                                              I think it might be because the question is now set to Community Wiki—I've just checked through and every answer is set that way (including mine which I've been thinking about deleting).
                                                                                              $endgroup$
                                                                                              – timtfj
                                                                                              Dec 25 '18 at 1:07













                                                                                              -1












                                                                                              $begingroup$

                                                                                              The direct proof:




                                                                                              $$0.9999999999...=lim_{ntoinfty} left( 1-frac {1}{10^n}right)=1-0=1$$




                                                                                              Q.E.D.






                                                                                              share|cite|improve this answer











                                                                                              $endgroup$


















                                                                                                -1












                                                                                                $begingroup$

                                                                                                The direct proof:




                                                                                                $$0.9999999999...=lim_{ntoinfty} left( 1-frac {1}{10^n}right)=1-0=1$$




                                                                                                Q.E.D.






                                                                                                share|cite|improve this answer











                                                                                                $endgroup$
















                                                                                                  -1












                                                                                                  -1








                                                                                                  -1





                                                                                                  $begingroup$

                                                                                                  The direct proof:




                                                                                                  $$0.9999999999...=lim_{ntoinfty} left( 1-frac {1}{10^n}right)=1-0=1$$




                                                                                                  Q.E.D.






                                                                                                  share|cite|improve this answer











                                                                                                  $endgroup$



                                                                                                  The direct proof:




                                                                                                  $$0.9999999999...=lim_{ntoinfty} left( 1-frac {1}{10^n}right)=1-0=1$$




                                                                                                  Q.E.D.







                                                                                                  share|cite|improve this answer














                                                                                                  share|cite|improve this answer



                                                                                                  share|cite|improve this answer








                                                                                                  answered Sep 4 '18 at 9:22


























                                                                                                  community wiki





                                                                                                  Math
























                                                                                                      -2












                                                                                                      $begingroup$

                                                                                                      [Note: this is my original answer, but completely rewritten to clarify its purpose.]



                                                                                                      This answer takes up Trevor Richards' point that people asking this question often aren't convinced by rigorous mathematical proofs and instead feel tricked by them. In this situation one thing that might help is a convincing visible demonstration that $0.999999 . . . =1$ has some chance of being true.



                                                                                                      The usual demonstration consists of getting someone to agree that $frac13=0.33333 . . . $ and then multiply it by $3$ to get $0.99999 . . . $. At this point they might be convinced, but might equally feel puzzled or duped.



                                                                                                      This, I think, is where more examples come in. We have to see that $frac13$ isn't some sort of special case that can be used to trick us.



                                                                                                      When I first encountered $0.999999. . .$, I found looking at multiples of $frac19$ helpful. Once you've convinced yourself that this can be represented by an infinite string of $1$'s, it's easy to see that repeatedly adding it gives $0.222222. . .$, $0.333333. . .$, $0.444444. . .$ all the way up to $0.999999. . .$



                                                                                                      There's a complete inevitability about this process, especially if you write it out on paper. But . . . maybe it's still just a trick with a repeating digit?



                                                                                                      OK then: let's try multiples of $frac17=0.142857 . . . $. This one's fun because of the way the cycle of digits behaves:



                                                                                                      $frac17=0.142857 . . .$
                                                                                                      $frac27=0.285714 . . .$
                                                                                                      $frac37=0.428571 . . .$



                                                                                                      and the pattern continues nicely, and soon it's "obvious" that the digits will just keep rotating round. But then, all of a sudden, they don't:



                                                                                                      $frac67=0.857142 . . .$
                                                                                                      $frac77=0. 999999 . . .$



                                                                                                      There it is again!



                                                                                                      We can try with other fractions too, like $frac{1}{13}$ and $frac{1}{37}$, that recur after a manageable number of digits. Always we end up at $0.999999 . . .$.



                                                                                                      At this stage, it should seem clear (but not formally proved) that accepting the idea of infinitely recurring decimals entails accepting that $0.999999. . . =1$.



                                                                                                      The remaining issue, of course, is the acceptance of infinitely recurring decimals. That's addressed in other answers.






                                                                                                      share|cite|improve this answer











                                                                                                      $endgroup$













                                                                                                      • $begingroup$
                                                                                                        I think this answer could be improved to explain more clearly and be like Noldorin's answer but Noldorin's answer already exists to this answer doesn't add anything to it.
                                                                                                        $endgroup$
                                                                                                        – Timothy
                                                                                                        Dec 24 '18 at 0:15










                                                                                                      • $begingroup$
                                                                                                        @Timothy My point was the psychology of it: by the time someone has worked their way up all the way from $frac19$ to $frac89$ it'll be obvious to them what's going to happen. I believe the block is more psychological than mathematical.
                                                                                                        $endgroup$
                                                                                                        – timtfj
                                                                                                        Dec 24 '18 at 0:28


















                                                                                                      -2












                                                                                                      $begingroup$

                                                                                                      [Note: this is my original answer, but completely rewritten to clarify its purpose.]



                                                                                                      This answer takes up Trevor Richards' point that people asking this question often aren't convinced by rigorous mathematical proofs and instead feel tricked by them. In this situation one thing that might help is a convincing visible demonstration that $0.999999 . . . =1$ has some chance of being true.



                                                                                                      The usual demonstration consists of getting someone to agree that $frac13=0.33333 . . . $ and then multiply it by $3$ to get $0.99999 . . . $. At this point they might be convinced, but might equally feel puzzled or duped.



                                                                                                      This, I think, is where more examples come in. We have to see that $frac13$ isn't some sort of special case that can be used to trick us.



                                                                                                      When I first encountered $0.999999. . .$, I found looking at multiples of $frac19$ helpful. Once you've convinced yourself that this can be represented by an infinite string of $1$'s, it's easy to see that repeatedly adding it gives $0.222222. . .$, $0.333333. . .$, $0.444444. . .$ all the way up to $0.999999. . .$



                                                                                                      There's a complete inevitability about this process, especially if you write it out on paper. But . . . maybe it's still just a trick with a repeating digit?



                                                                                                      OK then: let's try multiples of $frac17=0.142857 . . . $. This one's fun because of the way the cycle of digits behaves:



                                                                                                      $frac17=0.142857 . . .$
                                                                                                      $frac27=0.285714 . . .$
                                                                                                      $frac37=0.428571 . . .$



                                                                                                      and the pattern continues nicely, and soon it's "obvious" that the digits will just keep rotating round. But then, all of a sudden, they don't:



                                                                                                      $frac67=0.857142 . . .$
                                                                                                      $frac77=0. 999999 . . .$



                                                                                                      There it is again!



                                                                                                      We can try with other fractions too, like $frac{1}{13}$ and $frac{1}{37}$, that recur after a manageable number of digits. Always we end up at $0.999999 . . .$.



                                                                                                      At this stage, it should seem clear (but not formally proved) that accepting the idea of infinitely recurring decimals entails accepting that $0.999999. . . =1$.



                                                                                                      The remaining issue, of course, is the acceptance of infinitely recurring decimals. That's addressed in other answers.






                                                                                                      share|cite|improve this answer











                                                                                                      $endgroup$













                                                                                                      • $begingroup$
                                                                                                        I think this answer could be improved to explain more clearly and be like Noldorin's answer but Noldorin's answer already exists to this answer doesn't add anything to it.
                                                                                                        $endgroup$
                                                                                                        – Timothy
                                                                                                        Dec 24 '18 at 0:15










                                                                                                      • $begingroup$
                                                                                                        @Timothy My point was the psychology of it: by the time someone has worked their way up all the way from $frac19$ to $frac89$ it'll be obvious to them what's going to happen. I believe the block is more psychological than mathematical.
                                                                                                        $endgroup$
                                                                                                        – timtfj
                                                                                                        Dec 24 '18 at 0:28
















                                                                                                      -2












                                                                                                      -2








                                                                                                      -2





                                                                                                      $begingroup$

                                                                                                      [Note: this is my original answer, but completely rewritten to clarify its purpose.]



                                                                                                      This answer takes up Trevor Richards' point that people asking this question often aren't convinced by rigorous mathematical proofs and instead feel tricked by them. In this situation one thing that might help is a convincing visible demonstration that $0.999999 . . . =1$ has some chance of being true.



                                                                                                      The usual demonstration consists of getting someone to agree that $frac13=0.33333 . . . $ and then multiply it by $3$ to get $0.99999 . . . $. At this point they might be convinced, but might equally feel puzzled or duped.



                                                                                                      This, I think, is where more examples come in. We have to see that $frac13$ isn't some sort of special case that can be used to trick us.



                                                                                                      When I first encountered $0.999999. . .$, I found looking at multiples of $frac19$ helpful. Once you've convinced yourself that this can be represented by an infinite string of $1$'s, it's easy to see that repeatedly adding it gives $0.222222. . .$, $0.333333. . .$, $0.444444. . .$ all the way up to $0.999999. . .$



                                                                                                      There's a complete inevitability about this process, especially if you write it out on paper. But . . . maybe it's still just a trick with a repeating digit?



                                                                                                      OK then: let's try multiples of $frac17=0.142857 . . . $. This one's fun because of the way the cycle of digits behaves:



                                                                                                      $frac17=0.142857 . . .$
                                                                                                      $frac27=0.285714 . . .$
                                                                                                      $frac37=0.428571 . . .$



                                                                                                      and the pattern continues nicely, and soon it's "obvious" that the digits will just keep rotating round. But then, all of a sudden, they don't:



                                                                                                      $frac67=0.857142 . . .$
                                                                                                      $frac77=0. 999999 . . .$



                                                                                                      There it is again!



                                                                                                      We can try with other fractions too, like $frac{1}{13}$ and $frac{1}{37}$, that recur after a manageable number of digits. Always we end up at $0.999999 . . .$.



                                                                                                      At this stage, it should seem clear (but not formally proved) that accepting the idea of infinitely recurring decimals entails accepting that $0.999999. . . =1$.



                                                                                                      The remaining issue, of course, is the acceptance of infinitely recurring decimals. That's addressed in other answers.






                                                                                                      share|cite|improve this answer











                                                                                                      $endgroup$



                                                                                                      [Note: this is my original answer, but completely rewritten to clarify its purpose.]



                                                                                                      This answer takes up Trevor Richards' point that people asking this question often aren't convinced by rigorous mathematical proofs and instead feel tricked by them. In this situation one thing that might help is a convincing visible demonstration that $0.999999 . . . =1$ has some chance of being true.



                                                                                                      The usual demonstration consists of getting someone to agree that $frac13=0.33333 . . . $ and then multiply it by $3$ to get $0.99999 . . . $. At this point they might be convinced, but might equally feel puzzled or duped.



                                                                                                      This, I think, is where more examples come in. We have to see that $frac13$ isn't some sort of special case that can be used to trick us.



                                                                                                      When I first encountered $0.999999. . .$, I found looking at multiples of $frac19$ helpful. Once you've convinced yourself that this can be represented by an infinite string of $1$'s, it's easy to see that repeatedly adding it gives $0.222222. . .$, $0.333333. . .$, $0.444444. . .$ all the way up to $0.999999. . .$



                                                                                                      There's a complete inevitability about this process, especially if you write it out on paper. But . . . maybe it's still just a trick with a repeating digit?



                                                                                                      OK then: let's try multiples of $frac17=0.142857 . . . $. This one's fun because of the way the cycle of digits behaves:



                                                                                                      $frac17=0.142857 . . .$
                                                                                                      $frac27=0.285714 . . .$
                                                                                                      $frac37=0.428571 . . .$



                                                                                                      and the pattern continues nicely, and soon it's "obvious" that the digits will just keep rotating round. But then, all of a sudden, they don't:



                                                                                                      $frac67=0.857142 . . .$
                                                                                                      $frac77=0. 999999 . . .$



                                                                                                      There it is again!



                                                                                                      We can try with other fractions too, like $frac{1}{13}$ and $frac{1}{37}$, that recur after a manageable number of digits. Always we end up at $0.999999 . . .$.



                                                                                                      At this stage, it should seem clear (but not formally proved) that accepting the idea of infinitely recurring decimals entails accepting that $0.999999. . . =1$.



                                                                                                      The remaining issue, of course, is the acceptance of infinitely recurring decimals. That's addressed in other answers.







                                                                                                      share|cite|improve this answer














                                                                                                      share|cite|improve this answer



                                                                                                      share|cite|improve this answer








                                                                                                      edited Dec 25 '18 at 4:24


























                                                                                                      community wiki





                                                                                                      4 revs
                                                                                                      timtfj













                                                                                                      • $begingroup$
                                                                                                        I think this answer could be improved to explain more clearly and be like Noldorin's answer but Noldorin's answer already exists to this answer doesn't add anything to it.
                                                                                                        $endgroup$
                                                                                                        – Timothy
                                                                                                        Dec 24 '18 at 0:15










                                                                                                      • $begingroup$
                                                                                                        @Timothy My point was the psychology of it: by the time someone has worked their way up all the way from $frac19$ to $frac89$ it'll be obvious to them what's going to happen. I believe the block is more psychological than mathematical.
                                                                                                        $endgroup$
                                                                                                        – timtfj
                                                                                                        Dec 24 '18 at 0:28




















                                                                                                      • $begingroup$
                                                                                                        I think this answer could be improved to explain more clearly and be like Noldorin's answer but Noldorin's answer already exists to this answer doesn't add anything to it.
                                                                                                        $endgroup$
                                                                                                        – Timothy
                                                                                                        Dec 24 '18 at 0:15










                                                                                                      • $begingroup$
                                                                                                        @Timothy My point was the psychology of it: by the time someone has worked their way up all the way from $frac19$ to $frac89$ it'll be obvious to them what's going to happen. I believe the block is more psychological than mathematical.
                                                                                                        $endgroup$
                                                                                                        – timtfj
                                                                                                        Dec 24 '18 at 0:28


















                                                                                                      $begingroup$
                                                                                                      I think this answer could be improved to explain more clearly and be like Noldorin's answer but Noldorin's answer already exists to this answer doesn't add anything to it.
                                                                                                      $endgroup$
                                                                                                      – Timothy
                                                                                                      Dec 24 '18 at 0:15




                                                                                                      $begingroup$
                                                                                                      I think this answer could be improved to explain more clearly and be like Noldorin's answer but Noldorin's answer already exists to this answer doesn't add anything to it.
                                                                                                      $endgroup$
                                                                                                      – Timothy
                                                                                                      Dec 24 '18 at 0:15












                                                                                                      $begingroup$
                                                                                                      @Timothy My point was the psychology of it: by the time someone has worked their way up all the way from $frac19$ to $frac89$ it'll be obvious to them what's going to happen. I believe the block is more psychological than mathematical.
                                                                                                      $endgroup$
                                                                                                      – timtfj
                                                                                                      Dec 24 '18 at 0:28






                                                                                                      $begingroup$
                                                                                                      @Timothy My point was the psychology of it: by the time someone has worked their way up all the way from $frac19$ to $frac89$ it'll be obvious to them what's going to happen. I believe the block is more psychological than mathematical.
                                                                                                      $endgroup$
                                                                                                      – timtfj
                                                                                                      Dec 24 '18 at 0:28





                                                                                                      Popular posts from this blog

                                                                                                      Biblatex bibliography style without URLs when DOI exists (in Overleaf with Zotero bibliography)

                                                                                                      ComboBox Display Member on multiple fields

                                                                                                      Is it possible to collect Nectar points via Trainline?